anal zis 2 - széchenyi istván universityhorvathg/bsc/gazdmath2.pdfha f es g gra konja, ak ar t...

276
Anal´ ızis 2 Horv´ ath G´ abor SzE-Gy˝or A tant´ argyi k¨ ovetelm´ enyrendszer ´ es oktat´ asi seg´ edanyagok megtal´ alhat´ ok a http://rs1.szif.hu/~horvathg/horvathg.html ´ es a http://math.sze.hu webhelyeken. Horv´ ath G´ abor Anal´ ızis 2

Upload: others

Post on 07-Mar-2020

4 views

Category:

Documents


0 download

TRANSCRIPT

Page 1: Anal zis 2 - Széchenyi István Universityhorvathg/bsc/gazdmath2.pdfHa f es g gra konja, ak ar t obbsz or is, atmeszi egym ast [a;b]-n, az a-ban es b-ben all tott fugg} oleges egyenesnek,

Analızis 2

Horvath Gabor

SzE-Gyor

A tantargyi kovetelmenyrendszer es oktatasi segedanyagokmegtalalhatok a

http://rs1.szif.hu/~horvathg/horvathg.html

es a

http://math.sze.hu

webhelyeken.Horvath Gabor Analızis 2

Page 2: Anal zis 2 - Széchenyi István Universityhorvathg/bsc/gazdmath2.pdfHa f es g gra konja, ak ar t obbsz or is, atmeszi egym ast [a;b]-n, az a-ban es b-ben all tott fugg} oleges egyenesnek,

Hatarozott integral

Definıcio 1 (Felosztas)

Tekintsuk az [a, b] intervallumot es az intervallumba esoa = x0 < x1 < x2 < . . . < xn = b szamokat. Az [a, b] intervallum nreszre valo felosztasan az [x0, x1], [x1, x2], . . . , [xn−1, xn]intervallumok rendszeret ertjuk.

1. abra. Intervallum felosztasa

Definıcio 2 (Minden hataron tul finomodo felosztassorozat)

Ha minden n pozitıv egesz szamhoz hozzarendeljuk az [a, b]intervallum egy n reszre valo felosztasat, akkor egyfelosztassorozatot definialtunk. Ez a felosztassorozat mindenhataron tul finomodo, ha minden egyes felosztas leghosszabbreszintervallumanak a hosszabol alkotott szamsorozat nullahoz tart.

Horvath Gabor Analızis 2

Page 3: Anal zis 2 - Széchenyi István Universityhorvathg/bsc/gazdmath2.pdfHa f es g gra konja, ak ar t obbsz or is, atmeszi egym ast [a;b]-n, az a-ban es b-ben all tott fugg} oleges egyenesnek,

Hatarozott integral

Definıcio 3 (Kozelıto osszeg)

Tekintsuk az [a, b] intervallumon korlatos f fuggvenyt es az intervallum

egy n reszre valo felosztasat. Minden [xi−1, xi ] reszintervallumban

valasztunk egy ξi kozbulso helyet. Ekkor az f fuggveny adott

felosztashoz es kozbulso helyekhez tartozo kozelıto osszege

σn =n∑

i=1

f (ξi )(xi − xi−1).

2. abra. Kozelıto osszegHorvath Gabor Analızis 2

Page 4: Anal zis 2 - Széchenyi István Universityhorvathg/bsc/gazdmath2.pdfHa f es g gra konja, ak ar t obbsz or is, atmeszi egym ast [a;b]-n, az a-ban es b-ben all tott fugg} oleges egyenesnek,

Hatarozott integral

Definıcio 4 (Hatarozott integral)

Az [a, b] intervallumon korlatos f fuggveny [a, b] intervallum felettihatarozott integralja az I szam, ha tetszoleges minden hataron tulfinomodo felosztassorozathoz es kozbulso helyekhez tartozo σn

kozelıto osszegekreσn −→ I .

Ekkor f integralhato [a, b]-n, es ezt az I szamot ezentul ıgy jeloljuk:

b∫a

f (x) dx .

Cel

A tovabbiakban szeretnenk minel tobb fuggveny minel tobbintervallum feletti hatarozott integraljat kiszamolni.

Horvath Gabor Analızis 2

Page 5: Anal zis 2 - Széchenyi István Universityhorvathg/bsc/gazdmath2.pdfHa f es g gra konja, ak ar t obbsz or is, atmeszi egym ast [a;b]-n, az a-ban es b-ben all tott fugg} oleges egyenesnek,

Hatarozott integral

Mar csak a definıcio bonyolultsaga miatt sem varhato, hogyminden fuggvenynek van minden [a, b] intervallum felett hatarozottintegralja. Megis igen sok fuggveny eseten ez a helyzet.

Tetel 1

Ha f folytonos az [a, b] intervallumon, akkor letezik azb∫a

f (x) dx

hatarozott integralja.

A definıcio alapjan eleg korulmenyes kiszamolni egy fuggvenyhatarozott integraljat. Szerencsere bebizonyıthato egy alapvetotetel.

Horvath Gabor Analızis 2

Page 6: Anal zis 2 - Széchenyi István Universityhorvathg/bsc/gazdmath2.pdfHa f es g gra konja, ak ar t obbsz or is, atmeszi egym ast [a;b]-n, az a-ban es b-ben all tott fugg} oleges egyenesnek,

Newton-Leibniz tetel

Tetel 2 (Newton-Leibniz tetel)

Ha letezik az [a, b] intervallumon korlatos f fuggveny hatarozottintegralja, es a F fuggveny a f primitıv fuggvenye, azaz∫

f (x) dx = F (x) + C , akkor

b∫a

f (x) dx = [F (x)]ba := F (b)− F (a).

Feladat 1

Szamoljuk ki az2∫

0

x2 dx integralt.

Megoldas: Mivel, mint tudjuk,∫

x2 dx = x3

3 + C ,

2∫0

x2 dx =

[x3

3

]2

0

=23

3− 03

3=

8

3.

Horvath Gabor Analızis 2

Page 7: Anal zis 2 - Széchenyi István Universityhorvathg/bsc/gazdmath2.pdfHa f es g gra konja, ak ar t obbsz or is, atmeszi egym ast [a;b]-n, az a-ban es b-ben all tott fugg} oleges egyenesnek,

Newton-Leibniz tetel

Tetel 2 (Newton-Leibniz tetel)

Ha letezik az [a, b] intervallumon korlatos f fuggveny hatarozottintegralja, es a F fuggveny a f primitıv fuggvenye, azaz∫

f (x) dx = F (x) + C , akkor

b∫a

f (x) dx = [F (x)]ba := F (b)− F (a).

Feladat 1

Szamoljuk ki az2∫

0

x2 dx integralt.

Megoldas: Mivel, mint tudjuk,∫

x2 dx = x3

3 + C ,

2∫0

x2 dx =

[x3

3

]2

0

=23

3− 03

3=

8

3.

Horvath Gabor Analızis 2

Page 8: Anal zis 2 - Széchenyi István Universityhorvathg/bsc/gazdmath2.pdfHa f es g gra konja, ak ar t obbsz or is, atmeszi egym ast [a;b]-n, az a-ban es b-ben all tott fugg} oleges egyenesnek,

Hatarozott integral

A hatarozott integralokra vonatkozo szamos tetel kozul alegfontosabbakat foglalja ossze a kovetkezo.

Tetel 3

Legyen f es g integralhato [a, b]-n, α ∈ R, es c ∈ (a, b). Ekkor

1

b∫aα · f (x) dx = α ·

b∫a

f (x) dx,

2

b∫a

(f (x) + g(x)) dx =b∫a

f (x) dx +b∫a

g(x) dx,

3

b∫a

f (x) dx =c∫a

f (x) dx +b∫c

f (x) dx.

Az elso ketto fejezi ki a hatarozott integral linearitasat, ez analog ahatarozatlan integralokra vonatkozo tetellel. A linearitas egy fontoses elonyos tulajdonsag.

Horvath Gabor Analızis 2

Page 9: Anal zis 2 - Széchenyi István Universityhorvathg/bsc/gazdmath2.pdfHa f es g gra konja, ak ar t obbsz or is, atmeszi egym ast [a;b]-n, az a-ban es b-ben all tott fugg} oleges egyenesnek,

Teruletszamıtas

A hatarozott integralnak rengeteg alkalmazesa van a matematikanbelul es mas termeszettudomanyokban is. Mi csak egy ilyennelfogunk foglalkozunk, a teruletszımıtassal.Eddig tanulmanyaink soran megtanultuk nehany ”szabalyos”sıkidom teruletet kiszamolni, ilyen peldaul a teglalap, a haromszog,a sokszog, a kor, a korcikk, a korszelet es esetleg az ellipszis.Tovabba olyan alakzatok teruletet is ki tudjuk szamolni, amelyekszetdarabolhatok a fenti alakzatokra.Felmerul a kerdes: hogyan lehet egy ”szabalytalan” alakzatteruletet kiszamolni?Ennek a vizsgalata soran kiderult, hogy eloszor is az a kerdes, hogyhogyan kell egy sıkbeli alakzat teruletet definialni.Mindenesetre akarmilyen definıciot is konstrualunk, aztmegkoveteljuk, hogy az elobbi alakzatoknak legyen, es ugyanazlegyen a terulete, mint eddig.

Horvath Gabor Analızis 2

Page 10: Anal zis 2 - Széchenyi István Universityhorvathg/bsc/gazdmath2.pdfHa f es g gra konja, ak ar t obbsz or is, atmeszi egym ast [a;b]-n, az a-ban es b-ben all tott fugg} oleges egyenesnek,

Teruletszamıtas

Az is vilagos, hogy a teruletre gondolhatunk ugy is, hogy az egyfuggveny, amelyik a sık reszhalmazaihoz rendel egy szamot, azalakzat teruletet, es rendelkezik az alabbi tulajdonsagokkal:

1 egy alakzat terulete nem negatıv,

2 az a es b oldalu teglalap terulete ab,

3 ket diszjunkt halmaz uniojanak terulete a reszek teruletenekosszege,

4 egybevago alakzatok terulete egyenlo.

Ezutan egy meglepo teny derult ki: nem lehet ugy definialni ateruletet, hogy ezek a tulajdonsagok teljesuljonek, es a sık mindenreszhalmazanak legyen terulete. (Lenyegeben azert, mert a sıknaknagyon sok, nagyon bonyolult szerkezetu reszhalmaza van.)

Horvath Gabor Analızis 2

Page 11: Anal zis 2 - Széchenyi István Universityhorvathg/bsc/gazdmath2.pdfHa f es g gra konja, ak ar t obbsz or is, atmeszi egym ast [a;b]-n, az a-ban es b-ben all tott fugg} oleges egyenesnek,

Teruletszamıtas

Mi csak specialis, de azert eleg sok halmazt magaba foglalohalmaztıpusra definialjuk a teruletet.

Definıcio 5 (Gorbeoldalu trapez)

Legyen f az [a, b] intervallumon ertelmezett nemnegatıv folytonosfuggveny. Ekkor az f fuggveny [a, b] feletti gorbeoldalu trapeza az

N = (x , y) ∈ R2|a ≤ x ≤ b, 0 ≤ y ≤ f (x)

halmaz.

3. abra. Gorbeoldalu trapezHorvath Gabor Analızis 2

Page 12: Anal zis 2 - Széchenyi István Universityhorvathg/bsc/gazdmath2.pdfHa f es g gra konja, ak ar t obbsz or is, atmeszi egym ast [a;b]-n, az a-ban es b-ben all tott fugg} oleges egyenesnek,

Teruletszamıtas

Most mar megfogalmazhatjuk a teruletszamıtasban alapvetodefinıciot.

Definıcio 6 (Gorbeoldalu trapez terulete)

Legyen f az [a, b] intervallumon ertelmezett nemnegatıv folytonosfuggveny. Ekkor az f fuggveny [a, b] feletti gorbeoldalu trapezanaka terulete

T =

b∫a

f (x) dx .

Ebbol, es az integral linearitasabol kovetkezik az alabbi

Tetel 4

Ha f folytonos es nem pozitıv az [a, b] intervallumon, akkor

T = −b∫a

f (x) dx.

Horvath Gabor Analızis 2

Page 13: Anal zis 2 - Széchenyi István Universityhorvathg/bsc/gazdmath2.pdfHa f es g gra konja, ak ar t obbsz or is, atmeszi egym ast [a;b]-n, az a-ban es b-ben all tott fugg} oleges egyenesnek,

Teruletszamıtas

Definıcio 7

Abban az esetben, ha f , akar tobbszor is, elojelet valt [a, b]-n, aza-ban es b-ben allıtott fuggoleges egyenesnek, az x tengely, es afuggveny grafikonja kozotti alakzatot is gorbeoldalu trapeznakhıvjuk.

Ennek a Tetel 3 es a Tetel 4 segıtsegevel lehet kiszamolni ateruletet, a kovetkezo modon.

Eggyel tobb integralt kell kiszamolni, mint ahanyszor f elojeletvalt. Ezek az integralok eppen azok folott a maximalis hosszureszintervallumok folotti integralok, ahol f nem valt elojelet. Ezenintegralok abszolut ertekenek az osszege adja ilyenkor agorbeoldalu trapez teruletet.

Horvath Gabor Analızis 2

Page 14: Anal zis 2 - Széchenyi István Universityhorvathg/bsc/gazdmath2.pdfHa f es g gra konja, ak ar t obbsz or is, atmeszi egym ast [a;b]-n, az a-ban es b-ben all tott fugg} oleges egyenesnek,

Normaltartomany

A gorbevonalu trapez eseten az a lenyeg, hogy ezt az alakzatot aza-ban es b-ben allıtott fuggoleges egyenesnek, az x tengely, es afuggveny grafikonja hatarolja.A gorbevonalu trapez fogalmat erdemes picit altalanosıtani.

Definıcio 8 (Normaltartomany)

Legyen az f es a g az [a, b] intervallumon ertelmezett ket folytonosfuggveny, ugy, hogy g(x) ≥ f (x) teljesul minden x ∈ [a, b] eseten.Ekkor az

N = (x , y) ∈ R2|a ≤ x ≤ b, f (x) ≤ y ≤ g(x)

halmazt normaltartomanynak hıvjuk.

A normaltartomanyt tehat az a-ban es b-ben allıtott fuggolegesegyenesek, alulrol az f fuggveny grafikonja, felulrol a g fuggvenygrafikonja hatarolja.

Horvath Gabor Analızis 2

Page 15: Anal zis 2 - Széchenyi István Universityhorvathg/bsc/gazdmath2.pdfHa f es g gra konja, ak ar t obbsz or is, atmeszi egym ast [a;b]-n, az a-ban es b-ben all tott fugg} oleges egyenesnek,

Normaltartomany

Tetel 5

A normaltartomany terulete

T =

b∫a

(g(x)− f (x)) dx .

Jegyezzuk meg, hogy normaltartomany eseten nem szamıt, hogy fvagy g elojelet valt-e.

Definıcio 9

Ha f es g grafikonja, akar tobbszor is, atmeszi egymast [a, b]-n, aza-ban es b-ben allıtott fuggoleges egyenesnek, es a fuggvenyekgrafikonja kozotti alakzatot is normaltartomany hıvjuk.

Horvath Gabor Analızis 2

Page 16: Anal zis 2 - Széchenyi István Universityhorvathg/bsc/gazdmath2.pdfHa f es g gra konja, ak ar t obbsz or is, atmeszi egym ast [a;b]-n, az a-ban es b-ben all tott fugg} oleges egyenesnek,

Normaltartomany

Ennek a Tetel 3 es a Tetel 5 segıtsegevel lehet kiszamolni ateruletet, a kovetkezo modon.

Eggyel tobb integralt kell kiszamolni, mint ahanyszor g − f elojeletvalt. Ezek az integralok eppen azok folott a maximalis hosszureszintervallumok folotti integralok, ahol g − f nem valt elojelet.Ezen integralok abszolut ertekenek az osszege adja ilyenkor anormaltartomany teruletet.

Az eddigiek szamos alakzat teruletenek kiszamıtasat lehetoveteszik. Az alakzatokat gyakran ugy definialjuk, hogy megmondjuk,hogy milyen vonalak hataroljak. Ezekbol kell kihamozni, hogymilyen intervallumokrol, fuggvenyekrol, gorbeoldalu trapezokrolvagy normaltartomanyokrol van szo a feladatban. Ehhez a korabbifuggvenytani ismeretekre, a grafikonok felrajzolasara van szukseg.Lassunk nehany feladatot!

Horvath Gabor Analızis 2

Page 17: Anal zis 2 - Széchenyi István Universityhorvathg/bsc/gazdmath2.pdfHa f es g gra konja, ak ar t obbsz or is, atmeszi egym ast [a;b]-n, az a-ban es b-ben all tott fugg} oleges egyenesnek,

Feladat 2

Szamoljuk ki az f (x) = x2 + 2 fuggveny [−1, 2] feletti gorbevonalutrapezanak a teruletet.

Megoldas: Teruletszamıtas eseten mindig rajzoljuk fel azalakzatot!

Latjuk, hogy egy gorbeoldalu trapez teruleterol van szo. Igy

T =

2∫−1

(x2 + 2) dx =

[x3

3+ 2x

]2

−1

=8

3+ 4−

(−1

3+ (−2)

)= 9.

Horvath Gabor Analızis 2

Page 18: Anal zis 2 - Széchenyi István Universityhorvathg/bsc/gazdmath2.pdfHa f es g gra konja, ak ar t obbsz or is, atmeszi egym ast [a;b]-n, az a-ban es b-ben all tott fugg} oleges egyenesnek,

Feladat 2

Szamoljuk ki az f (x) = x2 + 2 fuggveny [−1, 2] feletti gorbevonalutrapezanak a teruletet.Megoldas: Teruletszamıtas eseten mindig rajzoljuk fel azalakzatot!

Latjuk, hogy egy gorbeoldalu trapez teruleterol van szo. Igy

T =

2∫−1

(x2 + 2) dx =

[x3

3+ 2x

]2

−1

=8

3+ 4−

(−1

3+ (−2)

)= 9.

Horvath Gabor Analızis 2

Page 19: Anal zis 2 - Széchenyi István Universityhorvathg/bsc/gazdmath2.pdfHa f es g gra konja, ak ar t obbsz or is, atmeszi egym ast [a;b]-n, az a-ban es b-ben all tott fugg} oleges egyenesnek,

Feladat 2

Szamoljuk ki az f (x) = x2 + 2 fuggveny [−1, 2] feletti gorbevonalutrapezanak a teruletet.Megoldas: Teruletszamıtas eseten mindig rajzoljuk fel azalakzatot!

Latjuk, hogy egy gorbeoldalu trapez teruleterol van szo. Igy

T =

2∫−1

(x2 + 2) dx =

[x3

3+ 2x

]2

−1

=8

3+ 4−

(−1

3+ (−2)

)= 9.

Horvath Gabor Analızis 2

Page 20: Anal zis 2 - Széchenyi István Universityhorvathg/bsc/gazdmath2.pdfHa f es g gra konja, ak ar t obbsz or is, atmeszi egym ast [a;b]-n, az a-ban es b-ben all tott fugg} oleges egyenesnek,

Feladat 2

Szamoljuk ki az f (x) = x2 + 2 fuggveny [−1, 2] feletti gorbevonalutrapezanak a teruletet.Megoldas: Teruletszamıtas eseten mindig rajzoljuk fel azalakzatot!

Latjuk, hogy egy gorbeoldalu trapez teruleterol van szo. Igy

T =

2∫−1

(x2 + 2) dx =

[x3

3+ 2x

]2

−1

=8

3+ 4−

(−1

3+ (−2)

)= 9.

Horvath Gabor Analızis 2

Page 21: Anal zis 2 - Széchenyi István Universityhorvathg/bsc/gazdmath2.pdfHa f es g gra konja, ak ar t obbsz or is, atmeszi egym ast [a;b]-n, az a-ban es b-ben all tott fugg} oleges egyenesnek,

Feladat 3Szamoljuk ki az f (x) = 3x − x2 − 2 fuggveny [0, 4] feletti gorbevonalu trapezanak a teruletet.

Megoldas: Most is az abraval kezdjuk.

Latjuk, hogy egy gorbeoldalu trapez teruleterol van szo, de f ketszer is elojelet valt a [0, 4] intervallumon.

T =

∣∣∣∣∣∣∣1∫

0

(3x − x2 − 2) dx

∣∣∣∣∣∣∣ +

2∫1

(3x − x2 − 2) dx +

∣∣∣∣∣∣∣4∫

2

(3x − x2 − 2) dx

∣∣∣∣∣∣∣ =

=

∣∣∣∣∣[

3x2

2−

x3

3− 2x

]1

0

∣∣∣∣∣ +

[3x2

2−

x3

3− 2x

]2

1

+

∣∣∣∣∣[

3x2

2−

x3

3− 2x

]4

2

∣∣∣∣∣ =5

6+

1

6+

28

6=

34

6.

Horvath Gabor Analızis 2

Page 22: Anal zis 2 - Széchenyi István Universityhorvathg/bsc/gazdmath2.pdfHa f es g gra konja, ak ar t obbsz or is, atmeszi egym ast [a;b]-n, az a-ban es b-ben all tott fugg} oleges egyenesnek,

Feladat 3Szamoljuk ki az f (x) = 3x − x2 − 2 fuggveny [0, 4] feletti gorbevonalu trapezanak a teruletet.Megoldas: Most is az abraval kezdjuk.

Latjuk, hogy egy gorbeoldalu trapez teruleterol van szo, de f ketszer is elojelet valt a [0, 4] intervallumon.

T =

∣∣∣∣∣∣∣1∫

0

(3x − x2 − 2) dx

∣∣∣∣∣∣∣ +

2∫1

(3x − x2 − 2) dx +

∣∣∣∣∣∣∣4∫

2

(3x − x2 − 2) dx

∣∣∣∣∣∣∣ =

=

∣∣∣∣∣[

3x2

2−

x3

3− 2x

]1

0

∣∣∣∣∣ +

[3x2

2−

x3

3− 2x

]2

1

+

∣∣∣∣∣[

3x2

2−

x3

3− 2x

]4

2

∣∣∣∣∣ =5

6+

1

6+

28

6=

34

6.

Horvath Gabor Analızis 2

Page 23: Anal zis 2 - Széchenyi István Universityhorvathg/bsc/gazdmath2.pdfHa f es g gra konja, ak ar t obbsz or is, atmeszi egym ast [a;b]-n, az a-ban es b-ben all tott fugg} oleges egyenesnek,

Feladat 3Szamoljuk ki az f (x) = 3x − x2 − 2 fuggveny [0, 4] feletti gorbevonalu trapezanak a teruletet.Megoldas: Most is az abraval kezdjuk.

Latjuk, hogy egy gorbeoldalu trapez teruleterol van szo, de f ketszer is elojelet valt a [0, 4] intervallumon.

T =

∣∣∣∣∣∣∣1∫

0

(3x − x2 − 2) dx

∣∣∣∣∣∣∣ +

2∫1

(3x − x2 − 2) dx +

∣∣∣∣∣∣∣4∫

2

(3x − x2 − 2) dx

∣∣∣∣∣∣∣ =

=

∣∣∣∣∣[

3x2

2−

x3

3− 2x

]1

0

∣∣∣∣∣ +

[3x2

2−

x3

3− 2x

]2

1

+

∣∣∣∣∣[

3x2

2−

x3

3− 2x

]4

2

∣∣∣∣∣ =5

6+

1

6+

28

6=

34

6.

Horvath Gabor Analızis 2

Page 24: Anal zis 2 - Széchenyi István Universityhorvathg/bsc/gazdmath2.pdfHa f es g gra konja, ak ar t obbsz or is, atmeszi egym ast [a;b]-n, az a-ban es b-ben all tott fugg} oleges egyenesnek,

Feladat 3Szamoljuk ki az f (x) = 3x − x2 − 2 fuggveny [0, 4] feletti gorbevonalu trapezanak a teruletet.Megoldas: Most is az abraval kezdjuk.

Latjuk, hogy egy gorbeoldalu trapez teruleterol van szo, de f ketszer is elojelet valt a [0, 4] intervallumon.

T =

∣∣∣∣∣∣∣1∫

0

(3x − x2 − 2) dx

∣∣∣∣∣∣∣ +

2∫1

(3x − x2 − 2) dx +

∣∣∣∣∣∣∣4∫

2

(3x − x2 − 2) dx

∣∣∣∣∣∣∣ =

=

∣∣∣∣∣[

3x2

2−

x3

3− 2x

]1

0

∣∣∣∣∣ +

[3x2

2−

x3

3− 2x

]2

1

+

∣∣∣∣∣[

3x2

2−

x3

3− 2x

]4

2

∣∣∣∣∣ =5

6+

1

6+

28

6=

34

6.

Horvath Gabor Analızis 2

Page 25: Anal zis 2 - Széchenyi István Universityhorvathg/bsc/gazdmath2.pdfHa f es g gra konja, ak ar t obbsz or is, atmeszi egym ast [a;b]-n, az a-ban es b-ben all tott fugg} oleges egyenesnek,

Feladat 4Szamoljuk ki az f (x) = 2− x es a g(x) = 4− 2x − x2 fuggvenyek grafikonjai altal kozrezart teruletet.

Megoldas: Felrajzoljuk az alakzatunkat hatarolo vonalakat.

Az abrarol latjuk, hogy normaltartomannyal van dolgunk, felulrol a g grafikonja, alulrol az f grafikonja hatarol. Aza es b azoknak a pontoknak az elso koordinatai, amelyekben f es g grafikonja metszi egymast. Ezek eppen azf (x) = g(x) egyenlet megoldasai. 2− x = 4− 2x − x2 pontosan akkor teljesul, ha x1 = −2, vagy x2 = 1, tehata = −2, b = 1. Az alakzatunk terulete pedig

T =

b∫a

(g(x)− f (x)) dx =

1∫−2

((4− 2x − x2)− (2− x)) dx =

1∫−2

(2− x − x2) dx =

=

[2x −

x2

2−

x3

3

]1

−2

=

(2−

1

2−

1

3

)−(−4− 2 +

8

3

)=

9

2.

Horvath Gabor Analızis 2

Page 26: Anal zis 2 - Széchenyi István Universityhorvathg/bsc/gazdmath2.pdfHa f es g gra konja, ak ar t obbsz or is, atmeszi egym ast [a;b]-n, az a-ban es b-ben all tott fugg} oleges egyenesnek,

Feladat 4Szamoljuk ki az f (x) = 2− x es a g(x) = 4− 2x − x2 fuggvenyek grafikonjai altal kozrezart teruletet.Megoldas: Felrajzoljuk az alakzatunkat hatarolo vonalakat.

Az abrarol latjuk, hogy normaltartomannyal van dolgunk, felulrol a g grafikonja, alulrol az f grafikonja hatarol. Aza es b azoknak a pontoknak az elso koordinatai, amelyekben f es g grafikonja metszi egymast. Ezek eppen azf (x) = g(x) egyenlet megoldasai. 2− x = 4− 2x − x2 pontosan akkor teljesul, ha x1 = −2, vagy x2 = 1, tehata = −2, b = 1. Az alakzatunk terulete pedig

T =

b∫a

(g(x)− f (x)) dx =

1∫−2

((4− 2x − x2)− (2− x)) dx =

1∫−2

(2− x − x2) dx =

=

[2x −

x2

2−

x3

3

]1

−2

=

(2−

1

2−

1

3

)−(−4− 2 +

8

3

)=

9

2.

Horvath Gabor Analızis 2

Page 27: Anal zis 2 - Széchenyi István Universityhorvathg/bsc/gazdmath2.pdfHa f es g gra konja, ak ar t obbsz or is, atmeszi egym ast [a;b]-n, az a-ban es b-ben all tott fugg} oleges egyenesnek,

Feladat 4Szamoljuk ki az f (x) = 2− x es a g(x) = 4− 2x − x2 fuggvenyek grafikonjai altal kozrezart teruletet.Megoldas: Felrajzoljuk az alakzatunkat hatarolo vonalakat.

Az abrarol latjuk, hogy normaltartomannyal van dolgunk, felulrol a g grafikonja, alulrol az f grafikonja hatarol. Aza es b azoknak a pontoknak az elso koordinatai, amelyekben f es g grafikonja metszi egymast. Ezek eppen azf (x) = g(x) egyenlet megoldasai. 2− x = 4− 2x − x2 pontosan akkor teljesul, ha x1 = −2, vagy x2 = 1, tehata = −2, b = 1. Az alakzatunk terulete pedig

T =

b∫a

(g(x)− f (x)) dx =

1∫−2

((4− 2x − x2)− (2− x)) dx =

1∫−2

(2− x − x2) dx =

=

[2x −

x2

2−

x3

3

]1

−2

=

(2−

1

2−

1

3

)−(−4− 2 +

8

3

)=

9

2.

Horvath Gabor Analızis 2

Page 28: Anal zis 2 - Széchenyi István Universityhorvathg/bsc/gazdmath2.pdfHa f es g gra konja, ak ar t obbsz or is, atmeszi egym ast [a;b]-n, az a-ban es b-ben all tott fugg} oleges egyenesnek,

Feladat 4Szamoljuk ki az f (x) = 2− x es a g(x) = 4− 2x − x2 fuggvenyek grafikonjai altal kozrezart teruletet.Megoldas: Felrajzoljuk az alakzatunkat hatarolo vonalakat.

Az abrarol latjuk, hogy normaltartomannyal van dolgunk, felulrol a g grafikonja, alulrol az f grafikonja hatarol. Aza es b azoknak a pontoknak az elso koordinatai, amelyekben f es g grafikonja metszi egymast. Ezek eppen azf (x) = g(x) egyenlet megoldasai. 2− x = 4− 2x − x2 pontosan akkor teljesul, ha x1 = −2, vagy x2 = 1, tehata = −2, b = 1. Az alakzatunk terulete pedig

T =

b∫a

(g(x)− f (x)) dx =

1∫−2

((4− 2x − x2)− (2− x)) dx =

1∫−2

(2− x − x2) dx =

=

[2x −

x2

2−

x3

3

]1

−2

=

(2−

1

2−

1

3

)−(−4− 2 +

8

3

)=

9

2.

Horvath Gabor Analızis 2

Page 29: Anal zis 2 - Széchenyi István Universityhorvathg/bsc/gazdmath2.pdfHa f es g gra konja, ak ar t obbsz or is, atmeszi egym ast [a;b]-n, az a-ban es b-ben all tott fugg} oleges egyenesnek,

Feladat 5Szamoljuk ki az f (x) = x2 − 2x + 3 es a g(x) = 3− x2 fuggvenyek [−1, 2] feletti normaltartomanyanak teruletet.

Megoldas: Most is felrajzoljuk az alakzatunkat hatarolo vonalakat.

Az abrarol latjuk, hogy ismet normaltartomannyal van dolgunk, de f es g grafikonja ketszer is atmetszi egymast amegadott intervallumon. A metszespontok az x2 − 2x + 3 = 3− x2, azaz 2x2 − 2x = 0 egyenlet megoldasabolx1 = 0, vagy x2 = 1. Az alakzatunk terulete tehat

T =

0∫−1

((x2 − 2x + 3)− (3− x2)) dx +

1∫0

((3− x2)− (x2 − 2x + 3)) dx +

2∫1

((x2 − 2x + 3)− (3− x2)) dx =

=

[2x3

3− x2

]0

−1

+

[x2 −

2x3

3

]1

0

+

[2x3

3− x2

]2

1

=5

3+

1

3+

5

3=

11

3.

Horvath Gabor Analızis 2

Page 30: Anal zis 2 - Széchenyi István Universityhorvathg/bsc/gazdmath2.pdfHa f es g gra konja, ak ar t obbsz or is, atmeszi egym ast [a;b]-n, az a-ban es b-ben all tott fugg} oleges egyenesnek,

Feladat 5Szamoljuk ki az f (x) = x2 − 2x + 3 es a g(x) = 3− x2 fuggvenyek [−1, 2] feletti normaltartomanyanak teruletet.Megoldas: Most is felrajzoljuk az alakzatunkat hatarolo vonalakat.

Az abrarol latjuk, hogy ismet normaltartomannyal van dolgunk, de f es g grafikonja ketszer is atmetszi egymast amegadott intervallumon. A metszespontok az x2 − 2x + 3 = 3− x2, azaz 2x2 − 2x = 0 egyenlet megoldasabolx1 = 0, vagy x2 = 1. Az alakzatunk terulete tehat

T =

0∫−1

((x2 − 2x + 3)− (3− x2)) dx +

1∫0

((3− x2)− (x2 − 2x + 3)) dx +

2∫1

((x2 − 2x + 3)− (3− x2)) dx =

=

[2x3

3− x2

]0

−1

+

[x2 −

2x3

3

]1

0

+

[2x3

3− x2

]2

1

=5

3+

1

3+

5

3=

11

3.

Horvath Gabor Analızis 2

Page 31: Anal zis 2 - Széchenyi István Universityhorvathg/bsc/gazdmath2.pdfHa f es g gra konja, ak ar t obbsz or is, atmeszi egym ast [a;b]-n, az a-ban es b-ben all tott fugg} oleges egyenesnek,

Feladat 5Szamoljuk ki az f (x) = x2 − 2x + 3 es a g(x) = 3− x2 fuggvenyek [−1, 2] feletti normaltartomanyanak teruletet.Megoldas: Most is felrajzoljuk az alakzatunkat hatarolo vonalakat.

Az abrarol latjuk, hogy ismet normaltartomannyal van dolgunk, de f es g grafikonja ketszer is atmetszi egymast amegadott intervallumon. A metszespontok az x2 − 2x + 3 = 3− x2, azaz 2x2 − 2x = 0 egyenlet megoldasabolx1 = 0, vagy x2 = 1. Az alakzatunk terulete tehat

T =

0∫−1

((x2 − 2x + 3)− (3− x2)) dx +

1∫0

((3− x2)− (x2 − 2x + 3)) dx +

2∫1

((x2 − 2x + 3)− (3− x2)) dx =

=

[2x3

3− x2

]0

−1

+

[x2 −

2x3

3

]1

0

+

[2x3

3− x2

]2

1

=5

3+

1

3+

5

3=

11

3.

Horvath Gabor Analızis 2

Page 32: Anal zis 2 - Széchenyi István Universityhorvathg/bsc/gazdmath2.pdfHa f es g gra konja, ak ar t obbsz or is, atmeszi egym ast [a;b]-n, az a-ban es b-ben all tott fugg} oleges egyenesnek,

Feladat 5Szamoljuk ki az f (x) = x2 − 2x + 3 es a g(x) = 3− x2 fuggvenyek [−1, 2] feletti normaltartomanyanak teruletet.Megoldas: Most is felrajzoljuk az alakzatunkat hatarolo vonalakat.

Az abrarol latjuk, hogy ismet normaltartomannyal van dolgunk, de f es g grafikonja ketszer is atmetszi egymast amegadott intervallumon. A metszespontok az x2 − 2x + 3 = 3− x2, azaz 2x2 − 2x = 0 egyenlet megoldasabolx1 = 0, vagy x2 = 1. Az alakzatunk terulete tehat

T =

0∫−1

((x2 − 2x + 3)− (3− x2)) dx +

1∫0

((3− x2)− (x2 − 2x + 3)) dx +

2∫1

((x2 − 2x + 3)− (3− x2)) dx =

=

[2x3

3− x2

]0

−1

+

[x2 −

2x3

3

]1

0

+

[2x3

3− x2

]2

1

=5

3+

1

3+

5

3=

11

3.

Horvath Gabor Analızis 2

Page 33: Anal zis 2 - Széchenyi István Universityhorvathg/bsc/gazdmath2.pdfHa f es g gra konja, ak ar t obbsz or is, atmeszi egym ast [a;b]-n, az a-ban es b-ben all tott fugg} oleges egyenesnek,

Feladat 6Szamoljuk ki az f (x) =

√2− x es a g(x) = −x fuggvenyek grafikonja, es az x tengely altal hatarolt sıkidom

teruletet.

Megoldas: Felrajzoljuk az alakzatunkat hatarolo vonalakat.

Alakzatunk most is ket fuggoleges koze szorıthato, a√

2− x = −x egyenlet megoldasabol a = −2. Az alakzatnormaltartomany, de az alulrol hatarolo fuggveny szakaszonkent definialt. Ezert celszeru a sıkidomot a c = 0-banallıtott fuggoleges egyenessel ket reszre vegni, es alkalmazni a Tetel 3 3. reszet. Igy

T =

0∫−2

(√

2− x − (−x)) dx +

2∫0

√2− x dx =

[−

2

3

√(2− x)3 +

x2

2

]0

−2

+

[−

2

3

√(2− x)3

]2

0=

=10

3−

4√

2

3+

4√

2

3=

10

3.

Horvath Gabor Analızis 2

Page 34: Anal zis 2 - Széchenyi István Universityhorvathg/bsc/gazdmath2.pdfHa f es g gra konja, ak ar t obbsz or is, atmeszi egym ast [a;b]-n, az a-ban es b-ben all tott fugg} oleges egyenesnek,

Feladat 6Szamoljuk ki az f (x) =

√2− x es a g(x) = −x fuggvenyek grafikonja, es az x tengely altal hatarolt sıkidom

teruletet.Megoldas: Felrajzoljuk az alakzatunkat hatarolo vonalakat.

Alakzatunk most is ket fuggoleges koze szorıthato, a√

2− x = −x egyenlet megoldasabol a = −2. Az alakzatnormaltartomany, de az alulrol hatarolo fuggveny szakaszonkent definialt. Ezert celszeru a sıkidomot a c = 0-banallıtott fuggoleges egyenessel ket reszre vegni, es alkalmazni a Tetel 3 3. reszet. Igy

T =

0∫−2

(√

2− x − (−x)) dx +

2∫0

√2− x dx =

[−

2

3

√(2− x)3 +

x2

2

]0

−2

+

[−

2

3

√(2− x)3

]2

0=

=10

3−

4√

2

3+

4√

2

3=

10

3.

Horvath Gabor Analızis 2

Page 35: Anal zis 2 - Széchenyi István Universityhorvathg/bsc/gazdmath2.pdfHa f es g gra konja, ak ar t obbsz or is, atmeszi egym ast [a;b]-n, az a-ban es b-ben all tott fugg} oleges egyenesnek,

Feladat 6Szamoljuk ki az f (x) =

√2− x es a g(x) = −x fuggvenyek grafikonja, es az x tengely altal hatarolt sıkidom

teruletet.Megoldas: Felrajzoljuk az alakzatunkat hatarolo vonalakat.

Alakzatunk most is ket fuggoleges koze szorıthato, a√

2− x = −x egyenlet megoldasabol a = −2. Az alakzatnormaltartomany, de az alulrol hatarolo fuggveny szakaszonkent definialt. Ezert celszeru a sıkidomot a c = 0-banallıtott fuggoleges egyenessel ket reszre vegni, es alkalmazni a Tetel 3 3. reszet. Igy

T =

0∫−2

(√

2− x − (−x)) dx +

2∫0

√2− x dx =

[−

2

3

√(2− x)3 +

x2

2

]0

−2

+

[−

2

3

√(2− x)3

]2

0=

=10

3−

4√

2

3+

4√

2

3=

10

3.

Horvath Gabor Analızis 2

Page 36: Anal zis 2 - Széchenyi István Universityhorvathg/bsc/gazdmath2.pdfHa f es g gra konja, ak ar t obbsz or is, atmeszi egym ast [a;b]-n, az a-ban es b-ben all tott fugg} oleges egyenesnek,

Feladat 6Szamoljuk ki az f (x) =

√2− x es a g(x) = −x fuggvenyek grafikonja, es az x tengely altal hatarolt sıkidom

teruletet.Megoldas: Felrajzoljuk az alakzatunkat hatarolo vonalakat.

Alakzatunk most is ket fuggoleges koze szorıthato, a√

2− x = −x egyenlet megoldasabol a = −2. Az alakzatnormaltartomany, de az alulrol hatarolo fuggveny szakaszonkent definialt. Ezert celszeru a sıkidomot a c = 0-banallıtott fuggoleges egyenessel ket reszre vegni, es alkalmazni a Tetel 3 3. reszet. Igy

T =

0∫−2

(√

2− x − (−x)) dx +

2∫0

√2− x dx =

[−

2

3

√(2− x)3 +

x2

2

]0

−2

+

[−

2

3

√(2− x)3

]2

0=

=10

3−

4√

2

3+

4√

2

3=

10

3.

Horvath Gabor Analızis 2

Page 37: Anal zis 2 - Széchenyi István Universityhorvathg/bsc/gazdmath2.pdfHa f es g gra konja, ak ar t obbsz or is, atmeszi egym ast [a;b]-n, az a-ban es b-ben all tott fugg} oleges egyenesnek,

Improprius integralokAz

b∫a

f (x) dx

hatarozott integralban az [a, b] intervallum korlatos intervallum, es f korlatos fuggveny [a, b]-n.Az improprius integralok a kozonseges hatarozott integral altalanosıtasai, amelyeket ugy kapunk, hogy a fenti ketfeltetel valamelyiket vagy mindkettot elejtjuk.

Definıcio 10 (Vegtelen intervallunom korlatos fuggveny improprius integralja)

Legyen f folytonos az [a,∞) vegtelen intervallumon. Az f fuggveny [a,∞) intervallumon vett improprius integralja

∞∫a

f (x) dx = limu→∞

u∫a

f (x) dx,

ha ez a limesz letezik es veges. Ilyenkor azt is mondjuk, hogy az∞∫a

f (x) dx improprius integral konvergens, mas

esetekben pedig divergens.

Tetel 6

Ha F a f fuggveny primitıv fuggvenye az [a,∞) intervallumon, akkor

∞∫a

f (x) dx = limx→∞

F (x)− limx→a+

F (x) = limx→∞

F (x)− F (a).

Horvath Gabor Analızis 2

Page 38: Anal zis 2 - Széchenyi István Universityhorvathg/bsc/gazdmath2.pdfHa f es g gra konja, ak ar t obbsz or is, atmeszi egym ast [a;b]-n, az a-ban es b-ben all tott fugg} oleges egyenesnek,

Geometriai interpretacio

Legyen f folytonos es pozitıv az [a,∞) halmazon. Ekkorb∫a

f (x)dx az x tengely es a fuggvenygorbe kozotti alakzat

teruletet szamolja ki az [a, b] intervallum felett. Ha b-vel tartunk a vegtelenbe, akkor ez az alakzat kozeledik aza-ban allıtott fuggoleges, az x tengely es a fuggvenygorbe kozotti vegtelenben nyulo alakzathoz. Tehat azt

mondhatjuk, hogy az∞∫a

f (x)dx improprius integral, ha letezik es veges, akkor ennek a vegtelenben nyulo

alakzatnak a teruletet szamolja ki.Ilyenkor a fuggveny hatarerteke a vegtelenben nulla kell, hogy legyen, hiszen, ha mondjuk mindig 0.02-nal nagyobberteket venne fel, akkor az alakzatba beırhato lenne egy 0.01 szazad magassagu es tetszoleges szelessegu teglalap,aminek persze barmekkora lehet a terulete. Igy a nala biztosan nagyobb teruletu alakzat terulete sem lehetne veges.Erdekes azonban, hogy az, hogy a fuggveny hatarerteke a vegtelenben nulla, nem eleg ahhoz, hogy a vegtelenbenyulo alakzat terulete veges legyen, az is kell, hogy a fuggveny eleg gyorsan tartson nullahoz. Peldaul

∞∫1

1

xdx =∞, de

∞∫1

1

x2dx = 1.

4. abra. Vegtelenbe nyulo alakzatokHorvath Gabor Analızis 2

Page 39: Anal zis 2 - Széchenyi István Universityhorvathg/bsc/gazdmath2.pdfHa f es g gra konja, ak ar t obbsz or is, atmeszi egym ast [a;b]-n, az a-ban es b-ben all tott fugg} oleges egyenesnek,

Improprius integralok

Definıcio 11 (Vegtelen intervallunom korlatos fuggveny improprius integralja)

Legyen f folytonos a (−∞, a] vegtelen intervallumon. Az f fuggveny (−∞, a] intervallumon vett impropriusintegralja

a∫−∞

f (x) dx = limu→−∞

a∫u

f (x) dx,

ha ez a limesz letezik es veges. Ilyenkor azt is mondjuk, hogy az∞∫a

f (x) dx improprius integral konvergens, mas

esetekben pedig divergens.

Tetel 7

Ha F a f fuggveny primitıv fuggvenye a (−∞, a] intervallumon, akkor

a∫−∞

f (x)dx = limx→a−

F (x)− limx→−∞

F (x) = F (a)− limx→−∞

F (x).

Horvath Gabor Analızis 2

Page 40: Anal zis 2 - Széchenyi István Universityhorvathg/bsc/gazdmath2.pdfHa f es g gra konja, ak ar t obbsz or is, atmeszi egym ast [a;b]-n, az a-ban es b-ben all tott fugg} oleges egyenesnek,

Improprius integralok

Definıcio 12 (Vegtelen intervallunom korlatos fuggveny improprius integralja)

Ha f a (−∞,∞) intervallumon folytonos fuggveny, akkor a (−∞,∞) intervallumon vett improprius integralja

∞∫−∞

f (x) dx =

a∫−∞

f (x) dx +

∞∫a

f (x) dx,

ha a jobb oldalon mindket improprius integral konvergens valamilyen tetszoleges, de rogzıtett a ∈ R eseten.

Tetel 8

Ha F a f primitıv fuggvenye a (−∞,∞) intervallumon, akkor

∞∫−∞

f (x)dx = limx→∞

F (x)− limx→−∞

F (x).

Horvath Gabor Analızis 2

Page 41: Anal zis 2 - Széchenyi István Universityhorvathg/bsc/gazdmath2.pdfHa f es g gra konja, ak ar t obbsz or is, atmeszi egym ast [a;b]-n, az a-ban es b-ben all tott fugg} oleges egyenesnek,

Feladat 7

Szamıtsuk ki az∞∫0

e−x+2 dx improprius integralt, ha az letezik.

Megoldas: A [0,∞) intervallumon folytonos az integrandus, es∫e−x+2 dx = −e−x+2︸ ︷︷ ︸

=F (x)

+C .

Ezert

∞∫0

e−x+2 dx = limx→∞

(−e−x+2)− (−e2) = 0 + e2 = e2.

Horvath Gabor Analızis 2

Page 42: Anal zis 2 - Széchenyi István Universityhorvathg/bsc/gazdmath2.pdfHa f es g gra konja, ak ar t obbsz or is, atmeszi egym ast [a;b]-n, az a-ban es b-ben all tott fugg} oleges egyenesnek,

Feladat 7

Szamıtsuk ki az∞∫0

e−x+2 dx improprius integralt, ha az letezik.

Megoldas: A [0,∞) intervallumon folytonos az integrandus, es∫e−x+2 dx = −e−x+2︸ ︷︷ ︸

=F (x)

+C .

Ezert

∞∫0

e−x+2 dx = limx→∞

(−e−x+2)− (−e2) = 0 + e2 = e2.

Horvath Gabor Analızis 2

Page 43: Anal zis 2 - Széchenyi István Universityhorvathg/bsc/gazdmath2.pdfHa f es g gra konja, ak ar t obbsz or is, atmeszi egym ast [a;b]-n, az a-ban es b-ben all tott fugg} oleges egyenesnek,

Feladat 8

Szamıtsuk ki az−1∫−∞

1x3 dx improprius integralt, ha az letezik.

Megoldas: A (−∞,−1] intervallumon folytonos az integrandus, es∫1

x3dx = − 1

2x2︸ ︷︷ ︸=F (x)

+C .

Ezert−1∫−∞

1

x3dx = −1

2− lim

x→−∞

(− 1

2x2

)= −1

2.

Horvath Gabor Analızis 2

Page 44: Anal zis 2 - Széchenyi István Universityhorvathg/bsc/gazdmath2.pdfHa f es g gra konja, ak ar t obbsz or is, atmeszi egym ast [a;b]-n, az a-ban es b-ben all tott fugg} oleges egyenesnek,

Feladat 8

Szamıtsuk ki az−1∫−∞

1x3 dx improprius integralt, ha az letezik.

Megoldas: A (−∞,−1] intervallumon folytonos az integrandus, es∫1

x3dx = − 1

2x2︸ ︷︷ ︸=F (x)

+C .

Ezert−1∫−∞

1

x3dx = −1

2− lim

x→−∞

(− 1

2x2

)= −1

2.

Horvath Gabor Analızis 2

Page 45: Anal zis 2 - Széchenyi István Universityhorvathg/bsc/gazdmath2.pdfHa f es g gra konja, ak ar t obbsz or is, atmeszi egym ast [a;b]-n, az a-ban es b-ben all tott fugg} oleges egyenesnek,

Feladat 9

Szamıtsuk ki az∞∫−∞

2x2+2x+2

dx improprius integralt, ha az letezik.

Megoldas: A (−∞,∞) intervallumon folytonos az integrandus, es∫2

x2 + 2x + 2dx =

∫2

1 + (x + 1)2dx = 2arctg(x + 1)︸ ︷︷ ︸

=F (x)

+C .

Ezert

∞∫−∞

2

x2 + 2x + 2dx = lim

x→−∞(2arctg(x+1))− lim

x→∞(2arctg(x+1)) =

= 2 · π2−(−2 · π

2

)= 2π.

Horvath Gabor Analızis 2

Page 46: Anal zis 2 - Széchenyi István Universityhorvathg/bsc/gazdmath2.pdfHa f es g gra konja, ak ar t obbsz or is, atmeszi egym ast [a;b]-n, az a-ban es b-ben all tott fugg} oleges egyenesnek,

Feladat 9

Szamıtsuk ki az∞∫−∞

2x2+2x+2

dx improprius integralt, ha az letezik.

Megoldas: A (−∞,∞) intervallumon folytonos az integrandus, es∫2

x2 + 2x + 2dx =

∫2

1 + (x + 1)2dx = 2arctg(x + 1)︸ ︷︷ ︸

=F (x)

+C .

Ezert

∞∫−∞

2

x2 + 2x + 2dx = lim

x→−∞(2arctg(x+1))− lim

x→∞(2arctg(x+1)) =

= 2 · π2−(−2 · π

2

)= 2π.

Horvath Gabor Analızis 2

Page 47: Anal zis 2 - Széchenyi István Universityhorvathg/bsc/gazdmath2.pdfHa f es g gra konja, ak ar t obbsz or is, atmeszi egym ast [a;b]-n, az a-ban es b-ben all tott fugg} oleges egyenesnek,

Feladat 10

Szamıtsuk ki az∞∫0

xe−x dx improprius integralt, ha az letezik.

Megoldas: Tudjuk, hogy az integrandus folytonos a [0,∞) intervallumon. A primitıv fuggvenyere van szuksegunk.Mivel az integrandus egy polinom es egy exponencialis fuggveny szorzata, parcialis integralast alakalmazunk, ugy,

hogy a polinomot kelljen derivalni, azaz u(x) = x, v′(x) = e−x . Ekkor u′(x) = 1, v(x) = −e−x . Tehat

∫x︸︷︷︸

u(x)

e−x︸︷︷︸v′(x)

dx = x(−e−x )−∫

(−e−x ) dx = x(−e−x )− e−x + C = −(x + 1)e−x︸ ︷︷ ︸F (x)

+C .

Ezt felhasznalva

∞∫0

xe−x dx = limx→∞

(−(x + 1)e−x )− (−1)︸ ︷︷ ︸=F (0)

= limx→∞

(−(x + 1)e−x ) + 1.

A limesz hatarozatlan alaku, ”0 · ∞” tıpusu. Miutan a szorzatot tortkent felırtuk, alkalmazhatjuk a L’Hospitalszabalyt a kiszamolasahoz.

limx→∞

(−(x + 1)e−x ) = limx→∞

(−(x + 1)

ex

)︸ ︷︷ ︸

”∞∞ ”

= limx→∞

(−1

ex

)= 0.

Igy∞∫0

xe−x dx = 1.

Horvath Gabor Analızis 2

Page 48: Anal zis 2 - Széchenyi István Universityhorvathg/bsc/gazdmath2.pdfHa f es g gra konja, ak ar t obbsz or is, atmeszi egym ast [a;b]-n, az a-ban es b-ben all tott fugg} oleges egyenesnek,

Feladat 10

Szamıtsuk ki az∞∫0

xe−x dx improprius integralt, ha az letezik.

Megoldas: Tudjuk, hogy az integrandus folytonos a [0,∞) intervallumon. A primitıv fuggvenyere van szuksegunk.Mivel az integrandus egy polinom es egy exponencialis fuggveny szorzata, parcialis integralast alakalmazunk, ugy,

hogy a polinomot kelljen derivalni, azaz u(x) = x, v′(x) = e−x . Ekkor u′(x) = 1, v(x) = −e−x . Tehat

∫x︸︷︷︸

u(x)

e−x︸︷︷︸v′(x)

dx = x(−e−x )−∫

(−e−x ) dx = x(−e−x )− e−x + C = −(x + 1)e−x︸ ︷︷ ︸F (x)

+C .

Ezt felhasznalva

∞∫0

xe−x dx = limx→∞

(−(x + 1)e−x )− (−1)︸ ︷︷ ︸=F (0)

= limx→∞

(−(x + 1)e−x ) + 1.

A limesz hatarozatlan alaku, ”0 · ∞” tıpusu. Miutan a szorzatot tortkent felırtuk, alkalmazhatjuk a L’Hospitalszabalyt a kiszamolasahoz.

limx→∞

(−(x + 1)e−x ) = limx→∞

(−(x + 1)

ex

)︸ ︷︷ ︸

”∞∞ ”

= limx→∞

(−1

ex

)= 0.

Igy∞∫0

xe−x dx = 1.

Horvath Gabor Analızis 2

Page 49: Anal zis 2 - Széchenyi István Universityhorvathg/bsc/gazdmath2.pdfHa f es g gra konja, ak ar t obbsz or is, atmeszi egym ast [a;b]-n, az a-ban es b-ben all tott fugg} oleges egyenesnek,

Feladat 11

Szamıtsuk ki az0∫−∞

ex

3−2ex dx improprius integralt, ha az letezik.

Megoldas: Az integrandus nevezoje ex = 1.5⇐⇒ x ≈ 0.41 esetenlenne nulla, ez nem esik az integracios intervallumba, ıgy azon azintegrandus folytonos. A primitıv fuggveny meghatarozasahoz aztkell eszrevenni, hogy a nevezo derivaltja csak konstans szorzobankulonbozik a szamlalotol, az

∫f ′

f szabaly alkalmazhato.∫ex

3− 2exdx = −1

2·∫−2ex

3− 2ex︸ ︷︷ ︸f ′f

dx = −1

2· ln |3− 2ex |︸ ︷︷ ︸

F (x)

+C .

Igy tehat

0∫−∞

ex

3− 2exdx = 0︸︷︷︸

F (0)=ln 1

− limx→−∞

(− ln |3− 2ex |

2

)=

ln 3

2≈ 0.56.

Horvath Gabor Analızis 2

Page 50: Anal zis 2 - Széchenyi István Universityhorvathg/bsc/gazdmath2.pdfHa f es g gra konja, ak ar t obbsz or is, atmeszi egym ast [a;b]-n, az a-ban es b-ben all tott fugg} oleges egyenesnek,

Feladat 11

Szamıtsuk ki az0∫−∞

ex

3−2ex dx improprius integralt, ha az letezik.

Megoldas: Az integrandus nevezoje ex = 1.5⇐⇒ x ≈ 0.41 esetenlenne nulla, ez nem esik az integracios intervallumba, ıgy azon azintegrandus folytonos. A primitıv fuggveny meghatarozasahoz aztkell eszrevenni, hogy a nevezo derivaltja csak konstans szorzobankulonbozik a szamlalotol, az

∫f ′

f szabaly alkalmazhato.∫ex

3− 2exdx = −1

2·∫−2ex

3− 2ex︸ ︷︷ ︸f ′f

dx = −1

2· ln |3− 2ex |︸ ︷︷ ︸

F (x)

+C .

Igy tehat

0∫−∞

ex

3− 2exdx = 0︸︷︷︸

F (0)=ln 1

− limx→−∞

(− ln |3− 2ex |

2

)=

ln 3

2≈ 0.56.

Horvath Gabor Analızis 2

Page 51: Anal zis 2 - Széchenyi István Universityhorvathg/bsc/gazdmath2.pdfHa f es g gra konja, ak ar t obbsz or is, atmeszi egym ast [a;b]-n, az a-ban es b-ben all tott fugg} oleges egyenesnek,

Feladat 12

Szamıtsuk ki az∞∫−∞

x(x2+3)2 dx improprius integralt, ha az letezik.

Megoldas: Az integrandus az egesz R-en folytonos. Ugyan tort alaku, de a nevezo derivaltja egy harmadfokupolinom, az elobbi szabaly nem alkalmazhato az integralasara. Ezert felırjuk nem tortkent az integrandust.

∫x

(x2 + 3)2dx =

∫x · (x2 + 3)−2 dx.

Itt felbukkan egy f fuggveny α-dik hatvanya, ami konstans szorzot nem tekintve, az f ′-vel van szorozva, tehat az∫f ′ · fα szabaly alkalmazhato.

∫x · (x2 + 3)−2 dx =

1

2

∫2x · (x2 + 3)−2︸ ︷︷ ︸

f ′·fα

dx = −1

2(x2 + 3)−1 + C = −

1

2(x2 + 3)︸ ︷︷ ︸F (x)

+C .

Igy vegul

∞∫−∞

x

(x2 + 3)2dx = lim

x→∞

(−

1

2(x2 + 3)

)− lim

x→−∞

(−

1

2(x2 + 3)

)= 0− 0 = 0.

Horvath Gabor Analızis 2

Page 52: Anal zis 2 - Széchenyi István Universityhorvathg/bsc/gazdmath2.pdfHa f es g gra konja, ak ar t obbsz or is, atmeszi egym ast [a;b]-n, az a-ban es b-ben all tott fugg} oleges egyenesnek,

Feladat 12

Szamıtsuk ki az∞∫−∞

x(x2+3)2 dx improprius integralt, ha az letezik.

Megoldas: Az integrandus az egesz R-en folytonos. Ugyan tort alaku, de a nevezo derivaltja egy harmadfokupolinom, az elobbi szabaly nem alkalmazhato az integralasara. Ezert felırjuk nem tortkent az integrandust.

∫x

(x2 + 3)2dx =

∫x · (x2 + 3)−2 dx.

Itt felbukkan egy f fuggveny α-dik hatvanya, ami konstans szorzot nem tekintve, az f ′-vel van szorozva, tehat az∫f ′ · fα szabaly alkalmazhato.

∫x · (x2 + 3)−2 dx =

1

2

∫2x · (x2 + 3)−2︸ ︷︷ ︸

f ′·fα

dx = −1

2(x2 + 3)−1 + C = −

1

2(x2 + 3)︸ ︷︷ ︸F (x)

+C .

Igy vegul

∞∫−∞

x

(x2 + 3)2dx = lim

x→∞

(−

1

2(x2 + 3)

)− lim

x→−∞

(−

1

2(x2 + 3)

)= 0− 0 = 0.

Horvath Gabor Analızis 2

Page 53: Anal zis 2 - Széchenyi István Universityhorvathg/bsc/gazdmath2.pdfHa f es g gra konja, ak ar t obbsz or is, atmeszi egym ast [a;b]-n, az a-ban es b-ben all tott fugg} oleges egyenesnek,

Matrixok

A matrixok taglalap alaku adattablazatok, legtobbszorszamtablazatok. Nagyon sok helyzetben celszeru az adatok ilyenelrendezese.

Definıcio 13 (Valos n × m-es matrix)

Valos n × m tıpusu matrixon egy olyan szamtablazatot ertunk, amelynek n sora es m oszlopa van, es az elemei

valos szamok. Azt, hogy A valos n × m tıpusu matrix ezentul A ∈ Rn×m jeloli. Az A matrix eseten az i-edik sorj-edik elemet ai,j jeloli. Az elemeket szogletes zarojelek kozott adjuk meg. A matrixokat vastag latin nagybetukkel,az elemeiket a megfelelo indexelt latin kisbetuvel jeloljuk.

A =

a1,1 a1,2 · · · a1,m−1 a1,m

a2,1 a2,2 · · · a2,m−1 a2,m...

.... . .

......

an−1,1 an−1,2 · · · an−1,m−1 an−1,m

an,1 an,2 · · · an,m−1 an,m

A rengeteg valos matrix kozott vannak specialis szerkezetuek, ezekkozul kovetkezik nehany. Elnevezeseket vezetunk be, illetvemuveleteket definialunk a matrixok kozott.

Horvath Gabor Analızis 2

Page 54: Anal zis 2 - Széchenyi István Universityhorvathg/bsc/gazdmath2.pdfHa f es g gra konja, ak ar t obbsz or is, atmeszi egym ast [a;b]-n, az a-ban es b-ben all tott fugg} oleges egyenesnek,

Matrixok

Definıcio 14

1 Az n × n tıpusu matrixokat negyzetes matrixoknak hıvjuk.

2 Az 1× n tıpusu matrixokat n dimenzios sormatrixoknak hıvjuk.

3 Az n × 1 tıpusu matrixokat n dimenzios oszlopmatrixoknakhıvjuk.

4 Ha egy matrix minden eleme nulla, akkor nullmatrixnak hıvjuk.

5 Egy negyzetes matrix foatloja az azonos sor- es oszlopindexelrendelkezo elemekbol all.

6 Egy negyzetes matrix diagonalis, ha a foatlojan kıvul mindenelem nulla.

7 Az In n× n-es egysegmatrix olyan diagonalis matrix, amelyneka foatlojaban csupa 1-es all.

8 Az A negyzetes matrix szimmetrikus, ha ai ,j = aj ,i minden i , jindexparra.

Horvath Gabor Analızis 2

Page 55: Anal zis 2 - Széchenyi István Universityhorvathg/bsc/gazdmath2.pdfHa f es g gra konja, ak ar t obbsz or is, atmeszi egym ast [a;b]-n, az a-ban es b-ben all tott fugg} oleges egyenesnek,

Matrixok osszeadasa

Definıcio 15

Legyen A,B ∈ Rn×m. Ekkor az A es B matrixok osszege az aszinten n ×m tıpusu C = A + B matrix, amelyre ci ,j = ai ,j + bi ,j

minden szobajovo i , j index eseten.

Osszeadni tehat csak azonos tıpusu matrixokat lehet, es az azonospozıcioju elemeket kell osszeadni.

Tetel 9

Az osszeadas kommutatıv es asszociatıv muvelet, azaz

1 A + B = B + A, (kommutativitas),

2 (A + B) + C = A + (B + C) = A + B + C, (asszociativitas).

Horvath Gabor Analızis 2

Page 56: Anal zis 2 - Széchenyi István Universityhorvathg/bsc/gazdmath2.pdfHa f es g gra konja, ak ar t obbsz or is, atmeszi egym ast [a;b]-n, az a-ban es b-ben all tott fugg} oleges egyenesnek,

Skalarral valo szorzas

Definıcio 16

Legyen A ∈ Rn×m es α ∈ R. Ekkor az A matrix α szorosa az aszinten n ×m tıpusu B = αA matrix, amelyre bi ,j = αai ,j mindenszobajovo i , j index eseten.

Szammal tehat barmilyen matrix megszorozhato, es szammal valoszorzaskor minden elemet meg kell szorozni.

Tetel 10

1 α(A + B) = αA + αB,

2 (α + β)A = αA + βA.

Definıcio 17

Legyen A,B ∈ Rn×m. Ekkor az A es B matrixok A−B kulonbsegeaz A + (−1)B matrix.

Horvath Gabor Analızis 2

Page 57: Anal zis 2 - Széchenyi István Universityhorvathg/bsc/gazdmath2.pdfHa f es g gra konja, ak ar t obbsz or is, atmeszi egym ast [a;b]-n, az a-ban es b-ben all tott fugg} oleges egyenesnek,

Feladat 13

Legyen A =

[0 −23 2

], B =

[1 1−2 −1

]. Szamıtsuk ki a

2A− 3B matrixot.

Megoldas: Mind a ket matrix 2× 2 tıpusu, ilyenek tetszolegesszamszorosaik is, azonos tıpusu matrixok pedig osszeadhatok eskivonhatok, ıgy a feladatban szereplo muveletek elvegezhetok.A definıciok szerint matrixok osszeadasat, kivonasat es szammalvalo szorzasat elemenkent kell elvegezni, ezert

2A− 3B =

[−3 −712 7

].

Horvath Gabor Analızis 2

Page 58: Anal zis 2 - Széchenyi István Universityhorvathg/bsc/gazdmath2.pdfHa f es g gra konja, ak ar t obbsz or is, atmeszi egym ast [a;b]-n, az a-ban es b-ben all tott fugg} oleges egyenesnek,

Feladat 13

Legyen A =

[0 −23 2

], B =

[1 1−2 −1

]. Szamıtsuk ki a

2A− 3B matrixot.Megoldas: Mind a ket matrix 2× 2 tıpusu, ilyenek tetszolegesszamszorosaik is, azonos tıpusu matrixok pedig osszeadhatok eskivonhatok, ıgy a feladatban szereplo muveletek elvegezhetok.A definıciok szerint matrixok osszeadasat, kivonasat es szammalvalo szorzasat elemenkent kell elvegezni, ezert

2A− 3B =

[−3 −712 7

].

Horvath Gabor Analızis 2

Page 59: Anal zis 2 - Széchenyi István Universityhorvathg/bsc/gazdmath2.pdfHa f es g gra konja, ak ar t obbsz or is, atmeszi egym ast [a;b]-n, az a-ban es b-ben all tott fugg} oleges egyenesnek,

Transzponalt

Definıcio 18

Legyen A ∈ Rn×m matrix. A transzponaltja az az m× n tıpusu AT

matrix, amelyre minden i = 1, . . . ,m, j = 1, . . . , n eseten aTij = aji .

A transzponalas tehat felcsereli a matrix sorait es oszlopait, azeredeti matrix soraibol lesznek a transzponalt oszlopai.A transzponalas es az eddigi muveletek kapcsolatat foglalja ossze akovetkezo tetel.

Tetel 11

Legyen A,B ∈ Rn×m. Ekkor

1 (AT )T = A,

2 (αA)T = α(AT ),

3 (A + B)T = AT + BT .

Horvath Gabor Analızis 2

Page 60: Anal zis 2 - Széchenyi István Universityhorvathg/bsc/gazdmath2.pdfHa f es g gra konja, ak ar t obbsz or is, atmeszi egym ast [a;b]-n, az a-ban es b-ben all tott fugg} oleges egyenesnek,

Feladat 14

Legyen A =

[1 0 22 1 −1

], B =

[1 −1 1−2 −1 0

]. Szamıtsuk ki a BT − AT matrixot.

Megoldas: Az A es a B matrix is 2× 3-as, a transzponaltjaik 3× 2-es matrixok, tehat elvegezheto a feladatbanszereplo muvelet. Eloszor is

BT =

1 −2−1 −1

1 0

, AT =

1 20 12 −1

.Igy

BT − AT =

0 −4−1 −2−1 1

.Kiszamolhattuk volna eloszor a

B− A =

[0 −1 −1−4 −2 1

]matrixot, majd a Tetel 11 masodik es harmadik resze alapjan ennek transzponaltja eppen BT − AT , hiszen

(B− A)T = (B + (−1)A)T = BT + ((−1)A)T = BT + (−1)AT = BT − AT.

Persze ugyanazt az eredmenyt kapjuk, mint elobb.

Horvath Gabor Analızis 2

Page 61: Anal zis 2 - Széchenyi István Universityhorvathg/bsc/gazdmath2.pdfHa f es g gra konja, ak ar t obbsz or is, atmeszi egym ast [a;b]-n, az a-ban es b-ben all tott fugg} oleges egyenesnek,

Feladat 14

Legyen A =

[1 0 22 1 −1

], B =

[1 −1 1−2 −1 0

]. Szamıtsuk ki a BT − AT matrixot.

Megoldas: Az A es a B matrix is 2× 3-as, a transzponaltjaik 3× 2-es matrixok, tehat elvegezheto a feladatbanszereplo muvelet. Eloszor is

BT =

1 −2−1 −1

1 0

, AT =

1 20 12 −1

.Igy

BT − AT =

0 −4−1 −2−1 1

.Kiszamolhattuk volna eloszor a

B− A =

[0 −1 −1−4 −2 1

]matrixot, majd a Tetel 11 masodik es harmadik resze alapjan ennek transzponaltja eppen BT − AT , hiszen

(B− A)T = (B + (−1)A)T = BT + ((−1)A)T = BT + (−1)AT = BT − AT.

Persze ugyanazt az eredmenyt kapjuk, mint elobb.

Horvath Gabor Analızis 2

Page 62: Anal zis 2 - Széchenyi István Universityhorvathg/bsc/gazdmath2.pdfHa f es g gra konja, ak ar t obbsz or is, atmeszi egym ast [a;b]-n, az a-ban es b-ben all tott fugg} oleges egyenesnek,

Matrixok szorzasa

Definıcio 19

Legyen A ∈ Rn×k , B ∈ Rk×m matrix. Ekkor A es B szorzata az azn ×m tıpusu C = AB matrix, amelyre minden i = 1, . . . , n,j = 1, . . . ,m eseten

ci ,j = ai ,1b1,j + ai ,2b2,j + · · ·+ ai ,k−1bk−1,j + ai ,k bk,j =k∑

s=1

ai ,sbs,j .

Tehat a szorzat matrix i-edik soranak j-edik elemet ugy kapjuk,hogy az elol levo matrix i-edik soranak elemeit rendremegszorozzuk a hatul allo matrix j-edik oszlopanak elemeivel, esezeket a szorzatokat osszeadjuk. Azt a feltetelt, hogy a szorzatbanelol allo matrixnak annyi oszlopa kell legyen, mint ahany sora ahatul allo matrixnak van, kompatibilitasi feltetelnek hıvjuk. Ezgarantalja, hogy a sorban es az oszlopban ugyanannyi elem van.

Horvath Gabor Analızis 2

Page 63: Anal zis 2 - Széchenyi István Universityhorvathg/bsc/gazdmath2.pdfHa f es g gra konja, ak ar t obbsz or is, atmeszi egym ast [a;b]-n, az a-ban es b-ben all tott fugg} oleges egyenesnek,

Matrixok szorzasa

Az eddig muveletek es a szorzas kapcsolatat foglalja ossze akovetkezo tetel.

Tetel 12

Tegyuk fel, hogy a matrixok tıpusaira vonatkozo szuksegesfeltetelek teljesulnek. Ekkor

1 AB 6= BA,

2 A(BC) = (AB)C = ABC,

3 (A + B)C = AC + BC,

4 A(B + C) = AB + AC,

5 (AB)T = BT AT .

Horvath Gabor Analızis 2

Page 64: Anal zis 2 - Széchenyi István Universityhorvathg/bsc/gazdmath2.pdfHa f es g gra konja, ak ar t obbsz or is, atmeszi egym ast [a;b]-n, az a-ban es b-ben all tott fugg} oleges egyenesnek,

Feladat 15

Legyen A =

[1 1 −12 0 3

], B =

1 −1 1 0−2 −1 2 1

1 2 3 4

. Szamıtsuk ki az AB matrixot, ha letezik.

Megoldas: Az A 2× 3-as, a B 3× 4-es, teljesul a kompatibilitasi feltetel, a matrixok ebben a sorrendbenosszeszorozhatok, es a szorzatuk egy 2× 4-es tıpusu C = AB matrix lesz. Ennek a nyolc elemet kell kiszamolni. Adefinıcio alapjan peldaul a szorzatmatrix masodik soranak harmadik elemet ugy kapjuk, hogy az elol allo matrixmasodik soranak elemeit megszorozzuk a hatul allo matrix harmadik oszlopanak az elemeivel, es ezeket aszorzatokat osszeadjuk. Tehat

c2,3 = a2,1b1,3 + a2,2b2,3 + a2,3b3,3 = 2 · 1 + 0 · 2 + 3 · 3 = 11.

Hasonloan szamolva a tobbi elemet is

C = AB =

[−2 −4 0 −3

5 4 11 12

].

Horvath Gabor Analızis 2

Page 65: Anal zis 2 - Széchenyi István Universityhorvathg/bsc/gazdmath2.pdfHa f es g gra konja, ak ar t obbsz or is, atmeszi egym ast [a;b]-n, az a-ban es b-ben all tott fugg} oleges egyenesnek,

Feladat 15

Legyen A =

[1 1 −12 0 3

], B =

1 −1 1 0−2 −1 2 1

1 2 3 4

. Szamıtsuk ki az AB matrixot, ha letezik.

Megoldas: Az A 2× 3-as, a B 3× 4-es, teljesul a kompatibilitasi feltetel, a matrixok ebben a sorrendbenosszeszorozhatok, es a szorzatuk egy 2× 4-es tıpusu C = AB matrix lesz. Ennek a nyolc elemet kell kiszamolni. Adefinıcio alapjan peldaul a szorzatmatrix masodik soranak harmadik elemet ugy kapjuk, hogy az elol allo matrixmasodik soranak elemeit megszorozzuk a hatul allo matrix harmadik oszlopanak az elemeivel, es ezeket aszorzatokat osszeadjuk. Tehat

c2,3 = a2,1b1,3 + a2,2b2,3 + a2,3b3,3 = 2 · 1 + 0 · 2 + 3 · 3 = 11.

Hasonloan szamolva a tobbi elemet is

C = AB =

[−2 −4 0 −3

5 4 11 12

].

Horvath Gabor Analızis 2

Page 66: Anal zis 2 - Széchenyi István Universityhorvathg/bsc/gazdmath2.pdfHa f es g gra konja, ak ar t obbsz or is, atmeszi egym ast [a;b]-n, az a-ban es b-ben all tott fugg} oleges egyenesnek,

Feladat 16

Legyen A =

−1 2 −12 1 01 3 −2

. Szamıtsuk ki az AAT matrixot, ha letezik.

Megoldas: Egy 3× 3-as matrix transzponaltja is ilyen tıpusu, az azonos meretu negyzetes matrixok pedigbarmilyen sorrendben szorozhatok, tehat letezik a kiszamıtando matrix. Mivel

AT =

−1 2 12 1 3−1 0 −2

,

AAT =

6 0 70 5 57 5 14

.Vegyuk eszre, hogy az eredmeny egy szimmetrikus matrix. Ez altalaban is ıgy van: tetszoleges A matrix eseten

letezik az AAT matrix es az AT A matrix, es mindketto szimmetrikus, de csak negyzetes A eseten egyenlok is.

Horvath Gabor Analızis 2

Page 67: Anal zis 2 - Széchenyi István Universityhorvathg/bsc/gazdmath2.pdfHa f es g gra konja, ak ar t obbsz or is, atmeszi egym ast [a;b]-n, az a-ban es b-ben all tott fugg} oleges egyenesnek,

Feladat 16

Legyen A =

−1 2 −12 1 01 3 −2

. Szamıtsuk ki az AAT matrixot, ha letezik.

Megoldas: Egy 3× 3-as matrix transzponaltja is ilyen tıpusu, az azonos meretu negyzetes matrixok pedigbarmilyen sorrendben szorozhatok, tehat letezik a kiszamıtando matrix. Mivel

AT =

−1 2 12 1 3−1 0 −2

,

AAT =

6 0 70 5 57 5 14

.Vegyuk eszre, hogy az eredmeny egy szimmetrikus matrix. Ez altalaban is ıgy van: tetszoleges A matrix eseten

letezik az AAT matrix es az AT A matrix, es mindketto szimmetrikus, de csak negyzetes A eseten egyenlok is.

Horvath Gabor Analızis 2

Page 68: Anal zis 2 - Széchenyi István Universityhorvathg/bsc/gazdmath2.pdfHa f es g gra konja, ak ar t obbsz or is, atmeszi egym ast [a;b]-n, az a-ban es b-ben all tott fugg} oleges egyenesnek,

Specialis oszlopmatrixok

Definıcio 20

1 1n jeloli azt az n dimenzios oszlopmatrixot, amelynek mideneleme 1-es.

2 en,k jeloli azt az n dimenzios oszlopmatrixot, amelynek, mideneleme nulla, kiveve a k-adik sorban levot, az pedig 1-es.

Peldaul

13 =

111

, e4,2 =

0100

.Vigyazzunk arra, hogy ne keverjuk ossze az 1n es az In matrixokat.

Horvath Gabor Analızis 2

Page 69: Anal zis 2 - Széchenyi István Universityhorvathg/bsc/gazdmath2.pdfHa f es g gra konja, ak ar t obbsz or is, atmeszi egym ast [a;b]-n, az a-ban es b-ben all tott fugg} oleges egyenesnek,

Feladat 17

Tekintsuk az A =

1 2 3 40 1 0 24 2 3 1

matrixot. Szamıtsuk ki es magyarazzuk meg az Ae4,2, az Ae4,4, az

eT3,1A es az eT

3,3A szorzatokat.

Megoldas: Az Ae4,2 szorzat letezik, mert teljesul a kompatibilitasi feltetel, es az eredmeny harom dimenziososzlopvektor. Elvegezve a szorzast, kapjuk, hogy

Ae4,2 =

1 2 3 40 1 0 24 2 3 1

0100

=

212

.

Vegyuk eszre, hogy ez nem mas, mint az A masodik oszlopa.Hasonloan,

Ae4,4 =

1 2 3 40 1 0 24 2 3 1

0001

=

421

,ami az A negyedik oszlopa.

Mivel eT3,1 =

[1 0 0

]az eT

3,1A szorzat letezik es egy negy dimenzios sorvektor.

eT3,1A =

[1 0 0

] 1 2 3 40 1 0 24 2 3 1

=[

1 2 3 4], ami A elso sora.

Hasonloan, eT3,3A =

[0 0 1

] 1 2 3 40 1 0 24 2 3 1

=[

4 2 3 1], ami A harmadik sora.

Horvath Gabor Analızis 2

Page 70: Anal zis 2 - Széchenyi István Universityhorvathg/bsc/gazdmath2.pdfHa f es g gra konja, ak ar t obbsz or is, atmeszi egym ast [a;b]-n, az a-ban es b-ben all tott fugg} oleges egyenesnek,

Feladat 17

Tekintsuk az A =

1 2 3 40 1 0 24 2 3 1

matrixot. Szamıtsuk ki es magyarazzuk meg az Ae4,2, az Ae4,4, az

eT3,1A es az eT

3,3A szorzatokat.

Megoldas: Az Ae4,2 szorzat letezik, mert teljesul a kompatibilitasi feltetel, es az eredmeny harom dimenziososzlopvektor. Elvegezve a szorzast, kapjuk, hogy

Ae4,2 =

1 2 3 40 1 0 24 2 3 1

0100

=

212

.

Vegyuk eszre, hogy ez nem mas, mint az A masodik oszlopa.Hasonloan,

Ae4,4 =

1 2 3 40 1 0 24 2 3 1

0001

=

421

,ami az A negyedik oszlopa.

Mivel eT3,1 =

[1 0 0

]az eT

3,1A szorzat letezik es egy negy dimenzios sorvektor.

eT3,1A =

[1 0 0

] 1 2 3 40 1 0 24 2 3 1

=[

1 2 3 4], ami A elso sora.

Hasonloan, eT3,3A =

[0 0 1

] 1 2 3 40 1 0 24 2 3 1

=[

4 2 3 1], ami A harmadik sora.

Horvath Gabor Analızis 2

Page 71: Anal zis 2 - Széchenyi István Universityhorvathg/bsc/gazdmath2.pdfHa f es g gra konja, ak ar t obbsz or is, atmeszi egym ast [a;b]-n, az a-ban es b-ben all tott fugg} oleges egyenesnek,

Feladat 18

Tekintsuk az A =

1 2 3 40 1 0 24 2 3 1

matrixot. Szamıtsuk ki es magyarazzuk meg az A14 es az 1T3 A

szorzatokat.

Megoldas: Az A14 szorzat letezik, mert teljesul a kompatibilitasi feltetel, es az eredmeny harom dimenziososzlopvektor. Elvegezve a szorzast, kapjuk, hogy

A14 =

1 2 3 40 1 0 24 2 3 1

1111

=

103

10

.

Vegyuk eszre, hogy ennek a sorai eppen az A megfelelo soraiban levo szamok osszegei.

Az 1T3 A szorzat is letezik, es egy negy dimenzios sorvektor.

eT3,1A =

[1 1 1

] 1 2 3 40 1 0 24 2 3 1

=[

5 5 6 7]. Ennek az oszlopai eppen az A

megfelelo oszlopaiban levo szamok osszegei.

Horvath Gabor Analızis 2

Page 72: Anal zis 2 - Széchenyi István Universityhorvathg/bsc/gazdmath2.pdfHa f es g gra konja, ak ar t obbsz or is, atmeszi egym ast [a;b]-n, az a-ban es b-ben all tott fugg} oleges egyenesnek,

Feladat 18

Tekintsuk az A =

1 2 3 40 1 0 24 2 3 1

matrixot. Szamıtsuk ki es magyarazzuk meg az A14 es az 1T3 A

szorzatokat.Megoldas: Az A14 szorzat letezik, mert teljesul a kompatibilitasi feltetel, es az eredmeny harom dimenziososzlopvektor. Elvegezve a szorzast, kapjuk, hogy

A14 =

1 2 3 40 1 0 24 2 3 1

1111

=

103

10

.

Vegyuk eszre, hogy ennek a sorai eppen az A megfelelo soraiban levo szamok osszegei.

Az 1T3 A szorzat is letezik, es egy negy dimenzios sorvektor.

eT3,1A =

[1 1 1

] 1 2 3 40 1 0 24 2 3 1

=[

5 5 6 7]. Ennek az oszlopai eppen az A

megfelelo oszlopaiban levo szamok osszegei.

Horvath Gabor Analızis 2

Page 73: Anal zis 2 - Széchenyi István Universityhorvathg/bsc/gazdmath2.pdfHa f es g gra konja, ak ar t obbsz or is, atmeszi egym ast [a;b]-n, az a-ban es b-ben all tott fugg} oleges egyenesnek,

Technologiai matrix

Tegyuk fel, hogy egy uzem negy fele termeket gyart harom felealapanyagbol. Ekkor az egyes termekek alapanyagszukseglete egyA, ugynevezett technologiai matrix segıtsegevel adhato meg:

A =

1 2 2 11 1 0 22 2 1 0

.A matrix oszlopai tartalmazzak az egyes termekekalapanyagszuksegletet, valamilyen egysegben.Az egyes alapanyagok arat a kT =

[1 2 3

]matrix ırja le, az

egy napi termeles darabszamait tT =[

2 1 3 4]

matrix adjameg. (Azert a matrixok transzponaltjat ırtuk fel, mert az kisebbhelyet foglal el.)

Horvath Gabor Analızis 2

Page 74: Anal zis 2 - Széchenyi István Universityhorvathg/bsc/gazdmath2.pdfHa f es g gra konja, ak ar t obbsz or is, atmeszi egym ast [a;b]-n, az a-ban es b-ben all tott fugg} oleges egyenesnek,

Technologiai matrix

Szamıtsuk ki es magyarazzuk meg, hogy mit jelentenek ekkor

1 az eT3,2A14,

2 a kT Ae4,2,

3 es a kT At

matrixok.

¶ Mivel a matrixok szorzasa asszociatıv, ezert eT3,2A14 =

(eT

3,2A)

14.

Azt is tudjuk, hogy eT3,2A nem mas, mint az A matrix masodik sora, eT

3,2A =[

1 1 0 2].

Ennek a sornak az a jelentese, hogy minden termekbol egy darabot gyartva az egyes termekekhez hanyegyseg kell a masodik alapanyagbol.Ha ezt a sormatrixot megszorozzuk az 14 oszlopmatrixal, akkor osszeadjuk a sormatrix elemeit.(

eT3,2A

)14 = 4.

Ennek a szamnak pedig az a jelentese, hogy minden termekbol egy darabot gyartva osszesen hany egysegrelesz szuksegunk a masodik alapanyagbol.

Horvath Gabor Analızis 2

Page 75: Anal zis 2 - Széchenyi István Universityhorvathg/bsc/gazdmath2.pdfHa f es g gra konja, ak ar t obbsz or is, atmeszi egym ast [a;b]-n, az a-ban es b-ben all tott fugg} oleges egyenesnek,

Technologiai matrix

· Ismet felhasznaljuk, hogy kT Ae4,2 =(

kT A)

e4,2. Az vilagos, hogy kT A =[

9 12 5 5]. Mit

jelentenek ennek a sorvektornak a szamai? Peldaul

9 = 1 · 1 + 2 · 1 + 3 · 2 = kT1,1 · a1,1 + kT

1,2 · a2,1 + kT1,3 · a3,1,

azaz egy darab elso termek legyartasahoz az 1 koltsegu elso alapanyagbol felhasznaltunk 1 egyseget, meg a2 koltsegu masodik alapanyagbol 1 egyseget, meg a 3 koltsegu harmadik alapanyagbol 2 egyseget. Ez aszam tehat egy darab elso termek legyartasanak alapanyag koltsege.

A kT A =[

9 12 5 5]

sormatrix tehat az egyes termekek egy darabjanak alapanyagkoltsegetadja meg.

A kT Ae4,2 = 12 szorzat ennek a sormatrixnak a masodik eleme, ez tehat azt jelenti, hogy a masodiktermek egy darabjanak mennyi az alapanyagkoltsege.

¸ A kT A =[

9 12 5 5]

matrix jelenteset mar ismerjuk.

kT A14 azt adna meg, hogy minden termekbol egy darabot gyartva mennyi az alapanyagkoltseg.

kT A (2 · 14) azt adna meg, hogy minden termekbol ket darabot gyartva mennyi az alapanyagkoltseg.

Igy vegul kT At = 65 azt adja meg, hogy az egyes termekekbol rendre a t matrixban megadott darabotgyartva mennyi az alapanyagkoltseg.

Horvath Gabor Analızis 2

Page 76: Anal zis 2 - Széchenyi István Universityhorvathg/bsc/gazdmath2.pdfHa f es g gra konja, ak ar t obbsz or is, atmeszi egym ast [a;b]-n, az a-ban es b-ben all tott fugg} oleges egyenesnek,

Feladat 19

Egy csukraszda kınalataban negyfele torta szerepel: csokolade, dobos, turo es gyumolcs torta. Egy hetvege ket

napjan a tortakbol eladott mennyisegeket tartalmazza az A =

[12 18 16 2130 17 21 19

]matrix, a tortak ara a

fenti sorrendben pT =[

180 200 280 250].

1 Szamıtsuk ki matrixok segıtsegevel a vasarnapi tortaeladasbol szarmazo arbevetelt.

2 Szamıtsuk ki matrixok segıtsegevel szombati turo torta eladasbol szarmazo arbevetelt.

Megoldas:

1 Az A masodik sorat kell megszorozni a p oszlopvektorral.

eT2,2Ap =

(eT

2,2A)

p = 19430.

Itt, mint tudjuk, a zarojelben levo szorzat az A masodik sora.

2 A elso soranak harmadik elemet kell megszorozni p harmadik elemevel.

(eT2,1Ae4,3)(eT

4,3p) = (eT2,1Ae4,3)(pT e4,3) = 16 · 280 = 4480.

Horvath Gabor Analızis 2

Page 77: Anal zis 2 - Széchenyi István Universityhorvathg/bsc/gazdmath2.pdfHa f es g gra konja, ak ar t obbsz or is, atmeszi egym ast [a;b]-n, az a-ban es b-ben all tott fugg} oleges egyenesnek,

Feladat 19

Egy csukraszda kınalataban negyfele torta szerepel: csokolade, dobos, turo es gyumolcs torta. Egy hetvege ket

napjan a tortakbol eladott mennyisegeket tartalmazza az A =

[12 18 16 2130 17 21 19

]matrix, a tortak ara a

fenti sorrendben pT =[

180 200 280 250].

1 Szamıtsuk ki matrixok segıtsegevel a vasarnapi tortaeladasbol szarmazo arbevetelt.

2 Szamıtsuk ki matrixok segıtsegevel szombati turo torta eladasbol szarmazo arbevetelt.

Megoldas:

1 Az A masodik sorat kell megszorozni a p oszlopvektorral.

eT2,2Ap =

(eT

2,2A)

p = 19430.

Itt, mint tudjuk, a zarojelben levo szorzat az A masodik sora.

2 A elso soranak harmadik elemet kell megszorozni p harmadik elemevel.

(eT2,1Ae4,3)(eT

4,3p) = (eT2,1Ae4,3)(pT e4,3) = 16 · 280 = 4480.

Horvath Gabor Analızis 2

Page 78: Anal zis 2 - Széchenyi István Universityhorvathg/bsc/gazdmath2.pdfHa f es g gra konja, ak ar t obbsz or is, atmeszi egym ast [a;b]-n, az a-ban es b-ben all tott fugg} oleges egyenesnek,

A kombinatorika alapfeladata

A kombinatorika a matematika egy onallo fejezete.Mi ennek a nagy tudomanyteruletnek csak egy kis reszet fogjukerinteni.

Alapfeladat

A kombinatorika alapfeladata: megszamolni, hogy egy vegeshalmaznak hany eleme van.

Ez egyszerunek tunhet, de hamarosan ki fog derulni, hogy ez afeladat nagyon nehez is lehet.

Horvath Gabor Analızis 2

Page 79: Anal zis 2 - Széchenyi István Universityhorvathg/bsc/gazdmath2.pdfHa f es g gra konja, ak ar t obbsz or is, atmeszi egym ast [a;b]-n, az a-ban es b-ben all tott fugg} oleges egyenesnek,

Veges halmazok megadasa

Egy veges halmazt ket modon definialhatunk, azaz adhatunk meg.

Definıcio 21

Egy veges halmazt megadhatunk ugy, hogy kapcsos zarojelekkozott vesszovel elvalasztva felsoroljuk az elemeit.

Pelda

A = 1, a, ∅, .

Nem nehez megszamolni, hogy ennek az A halmaznak 4 eleme van.

Definıcio 22

Az A veges halmaz elemszamat |A| jeloli.

Horvath Gabor Analızis 2

Page 80: Anal zis 2 - Széchenyi István Universityhorvathg/bsc/gazdmath2.pdfHa f es g gra konja, ak ar t obbsz or is, atmeszi egym ast [a;b]-n, az a-ban es b-ben all tott fugg} oleges egyenesnek,

Veges halmazok megadasa

Ha az A halmaznak nagyon sok eleme van, akkor ez a modja ahalmaz megadasanak nem celszeru.

Egy halmazt nagyon gyakran megadhatunk az elemeinek a”tulajdonsagaval” is.Peldaul legyen A = 1, 2, 3, 4, 5, 6, 7, 8, 9, es tekintsuk azt a Bhalmazt, amelyik A harommal oszthato elemeibol all.Ezt a halmazt ıgy jeloljuk: B = x ∈ A| x oszthato harommal.Vilagos, hogy most B = 3, 6, 9.

Definıcio 23

Az A halmaz azon elemeinek a halmazat, amelyek rendelkeznek aΦ tulajdonsaggal

x ∈ A|Φ(x)

fogja jelolni.

Horvath Gabor Analızis 2

Page 81: Anal zis 2 - Széchenyi István Universityhorvathg/bsc/gazdmath2.pdfHa f es g gra konja, ak ar t obbsz or is, atmeszi egym ast [a;b]-n, az a-ban es b-ben all tott fugg} oleges egyenesnek,

Veges halmazok megadasa

A Φ tulajdonsag lehet barmilyen ”ertelmes matematikaitulajdonsag”, az a lenyeg, hogy az A halmaz minden elemerolegyertelmuen el tudjuk donteni, hogy rendelkezik-e az adotttulajdonsaggal vagy nem.

Fontos az is, hogy ıgy csak egy adott A halmaz elemeibol tudunkkivalasztani bizonyosakat. (Kell az A halmaz is, nem csak a Φtulajdonsag.)

Horvath Gabor Analızis 2

Page 82: Anal zis 2 - Széchenyi István Universityhorvathg/bsc/gazdmath2.pdfHa f es g gra konja, ak ar t obbsz or is, atmeszi egym ast [a;b]-n, az a-ban es b-ben all tott fugg} oleges egyenesnek,

Alaptetelek

Nagyon sokat fogjuk hasznalni a kovetkezo negy szamolasi szabalyt(tetelt).

Tetel 13 (Osszeadas)

Ha A es B diszjunkt veges halmazok, akkor

|A ∪ B| = |A|+ |B|.

Altalanosabban, ha A1, A2,. . . , An paronket diszjunkt vegeshalmazok, akkor

|A1 ∪ A2 ∪ · · · ∪ An| = |A1|+ |A2|+ · · ·+ |An|.

Horvath Gabor Analızis 2

Page 83: Anal zis 2 - Széchenyi István Universityhorvathg/bsc/gazdmath2.pdfHa f es g gra konja, ak ar t obbsz or is, atmeszi egym ast [a;b]-n, az a-ban es b-ben all tott fugg} oleges egyenesnek,

Alaptetelek

Nagyon sokat fogjuk hasznalni a kovetkezo negy szamolasi szabalyt(tetelt).

Tetel 13 (Osszeadas)

Ha A es B diszjunkt veges halmazok, akkor

|A ∪ B| = |A|+ |B|.

Altalanosabban, ha A1, A2,. . . , An paronket diszjunkt vegeshalmazok, akkor

|A1 ∪ A2 ∪ · · · ∪ An| = |A1|+ |A2|+ · · ·+ |An|.

Horvath Gabor Analızis 2

Page 84: Anal zis 2 - Széchenyi István Universityhorvathg/bsc/gazdmath2.pdfHa f es g gra konja, ak ar t obbsz or is, atmeszi egym ast [a;b]-n, az a-ban es b-ben all tott fugg} oleges egyenesnek,

Alaptetelek

Tetel 14 (Kivonas)

Ha A veges halmaz, es B ⊂ A, akkor

|A \ B| = |A| − |B|.

Tetel 15 (Szorzas)

Ha A es B veges halmazok, akkor azoknak az (a, b) rendezettparoknak a szama, amelyekre a ∈ A es b ∈ B

|A| · |B|.

Altalanosabban, ha A1, A2,. . . , An veges halmazok, akkor azoknakaz (a1, a2, . . . , an) rendezett n-eseknek a szama, amelyekrea1 ∈ A1, a2 ∈ A2,. . . ,an ∈ An

|A1| · |A2| · . . . · |An|.

Horvath Gabor Analızis 2

Page 85: Anal zis 2 - Széchenyi István Universityhorvathg/bsc/gazdmath2.pdfHa f es g gra konja, ak ar t obbsz or is, atmeszi egym ast [a;b]-n, az a-ban es b-ben all tott fugg} oleges egyenesnek,

Alaptetelek

Tetel 14 (Kivonas)

Ha A veges halmaz, es B ⊂ A, akkor

|A \ B| = |A| − |B|.

Tetel 15 (Szorzas)

Ha A es B veges halmazok, akkor azoknak az (a, b) rendezettparoknak a szama, amelyekre a ∈ A es b ∈ B

|A| · |B|.

Altalanosabban, ha A1, A2,. . . , An veges halmazok, akkor azoknakaz (a1, a2, . . . , an) rendezett n-eseknek a szama, amelyekrea1 ∈ A1, a2 ∈ A2,. . . ,an ∈ An

|A1| · |A2| · . . . · |An|.Horvath Gabor Analızis 2

Page 86: Anal zis 2 - Széchenyi István Universityhorvathg/bsc/gazdmath2.pdfHa f es g gra konja, ak ar t obbsz or is, atmeszi egym ast [a;b]-n, az a-ban es b-ben all tott fugg} oleges egyenesnek,

Alaptetelek

Definıcio 24

Legyenek A es B veges halmazok. Az f : A −→ B fuggvenybijektıv vagy kolcsonosen egyertelmu, ha

1 injektıv, azaz x , y ∈ A, x 6= y eseten f (x) 6= f (y),

2 rakepezes, azaz minden y ∈ B eseten letezik x ∈ A, ugy, hogyf (x) = y.

Tetel 16 (Ekvivalens halmazok)

Ha letezik f : A −→ B bijekcio, akkor

|A| = |B|.

Ez talan a leghatekonyabb tetel veges halmazok elemszamanakmeghatarozasara. Ha ki akarjuk szamolni az A halmaz elemszamat,de az A elemeit nehez megszamolni, akkor bijektıven lekepezzukegy B halmazra, aminek konnyebb megszamolni az elemeit.

Horvath Gabor Analızis 2

Page 87: Anal zis 2 - Széchenyi István Universityhorvathg/bsc/gazdmath2.pdfHa f es g gra konja, ak ar t obbsz or is, atmeszi egym ast [a;b]-n, az a-ban es b-ben all tott fugg} oleges egyenesnek,

Feladat 20

Hany darab olyan ketjegyu szam van, amelyik 30-nal kisebb, es aszamjegyei novekvoek?

Megoldas: Ha egy ketjegyu szam 30-nal kisebb, akkor az elsoszamjegye egyes vagy kettes. Jeloljuk A-val a 30-nal kisebb esegyessel kezdodo, novekvo szamjegyu, B-vel a 30-nal kisebb eskettessel kezdodo, novekvo szamjegyu szamok halmazat.Eloszor is vilagos, hogy A es B diszjunkt, es amit ki akarunkszamolni az |A ∪ B|.Tovabba |A| = 8, hiszen a masodik szamjegy lehet2, 3, 4, 5, 6, 7, 8, 9, ez nyolc lehetoseg. Hasonloan |B| = 7.Igy a (Tetel 13) alapjan a keresett ketjegyu szamok szama

|A ∪ B| = |A|+ |B| = 8 + 7 = 15.

Horvath Gabor Analızis 2

Page 88: Anal zis 2 - Széchenyi István Universityhorvathg/bsc/gazdmath2.pdfHa f es g gra konja, ak ar t obbsz or is, atmeszi egym ast [a;b]-n, az a-ban es b-ben all tott fugg} oleges egyenesnek,

Feladat 20

Hany darab olyan ketjegyu szam van, amelyik 30-nal kisebb, es aszamjegyei novekvoek?Megoldas: Ha egy ketjegyu szam 30-nal kisebb, akkor az elsoszamjegye egyes vagy kettes. Jeloljuk A-val a 30-nal kisebb esegyessel kezdodo, novekvo szamjegyu, B-vel a 30-nal kisebb eskettessel kezdodo, novekvo szamjegyu szamok halmazat.Eloszor is vilagos, hogy A es B diszjunkt, es amit ki akarunkszamolni az |A ∪ B|.Tovabba |A| = 8, hiszen a masodik szamjegy lehet2, 3, 4, 5, 6, 7, 8, 9, ez nyolc lehetoseg. Hasonloan |B| = 7.Igy a (Tetel 13) alapjan a keresett ketjegyu szamok szama

|A ∪ B| = |A|+ |B| = 8 + 7 = 15.

Horvath Gabor Analızis 2

Page 89: Anal zis 2 - Széchenyi István Universityhorvathg/bsc/gazdmath2.pdfHa f es g gra konja, ak ar t obbsz or is, atmeszi egym ast [a;b]-n, az a-ban es b-ben all tott fugg} oleges egyenesnek,

Feladat 21

Hany darab olyan ezernel nem nagyobb pozitıv egesz szam van,amelyik legalabb ket kulonbozo szamjegyet tartalmaz?

Megoldas: Az ezernel nem nagyobb pozitıv egesz szamokhalmazat jelolje A. Pesze |A| = 1000. Ha egy szamra nem igaz az,hogy legalabb ket kulonbozo szamjegyet tartalmaz, akkor csakegyetlen szamjegyet tartalmaz. Jelolje B az ezernel nem nagyobbpozitıv egeszek kozul azok halmazat, amelyek csak egy szamjegyettartalmaznak. Ekkor B ⊂ A, es A \ B az ezernel nem nagyobb,legalabb ket kulonbozo szamjegyet tartalmazo pozitıv egeszekhalmaza. Tehat amit keresunk, az |A \ B|. NyilvanvaloanB = B1 ∪ B2 ∪ B3, ahol B1 az egyjegyu, B2 a ketjegyu es B3 aharomjegyu, csak egyfele szamjegyet tartalmazo szamok halmaza.Ezek a Bi -k paronkent diszjunktak, es|B| = |B1|+ |B2|+ |B3| = 9 + 9 + 9 = 27, a (Tetel 13) miatt. Igy,a (Tetel 14) felhasznalasaval

|A \ B| = |A| − |B| = 1000− 27 = 973.

Horvath Gabor Analızis 2

Page 90: Anal zis 2 - Széchenyi István Universityhorvathg/bsc/gazdmath2.pdfHa f es g gra konja, ak ar t obbsz or is, atmeszi egym ast [a;b]-n, az a-ban es b-ben all tott fugg} oleges egyenesnek,

Feladat 21

Hany darab olyan ezernel nem nagyobb pozitıv egesz szam van,amelyik legalabb ket kulonbozo szamjegyet tartalmaz?Megoldas: Az ezernel nem nagyobb pozitıv egesz szamokhalmazat jelolje A. Pesze |A| = 1000. Ha egy szamra nem igaz az,hogy legalabb ket kulonbozo szamjegyet tartalmaz, akkor csakegyetlen szamjegyet tartalmaz. Jelolje B az ezernel nem nagyobbpozitıv egeszek kozul azok halmazat, amelyek csak egy szamjegyettartalmaznak. Ekkor B ⊂ A, es A \ B az ezernel nem nagyobb,legalabb ket kulonbozo szamjegyet tartalmazo pozitıv egeszekhalmaza. Tehat amit keresunk, az |A \ B|. NyilvanvaloanB = B1 ∪ B2 ∪ B3, ahol B1 az egyjegyu, B2 a ketjegyu es B3 aharomjegyu, csak egyfele szamjegyet tartalmazo szamok halmaza.Ezek a Bi -k paronkent diszjunktak, es|B| = |B1|+ |B2|+ |B3| = 9 + 9 + 9 = 27, a (Tetel 13) miatt. Igy,a (Tetel 14) felhasznalasaval

|A \ B| = |A| − |B| = 1000− 27 = 973.

Horvath Gabor Analızis 2

Page 91: Anal zis 2 - Széchenyi István Universityhorvathg/bsc/gazdmath2.pdfHa f es g gra konja, ak ar t obbsz or is, atmeszi egym ast [a;b]-n, az a-ban es b-ben all tott fugg} oleges egyenesnek,

Feladat 22

Egy cukraszdaban negy fele fagylalt kaphato: csokolade, vanılia,puncs es citrom. Ket fele tolcsert kerhetunk: hagyomanyosat eskakaost. Hanyfele egy gombocos fagylaltot vasarolhatunk?

Megoldas: Azt, hogy peldaul kakaos tolcserbe vettunk vanıliafagylaltot ıgy jelolhetjuk:

(kakaos, vanılia).Altalaban is, a vasarlasunk egy rendezett par segıtsegevel ırhato le,az elso tag azt adja meg, hogy milyen tolcsert valasztottunk, amasodik, hogy milyen fagylaltot. Ha

A1 = hagyomanyos, kakaos,

A2 = csokolade, vanılia,puncs, citrom,

akkor |A1| = 2, |A2| = 4, es a (Tetel 15) szerint a vasarlasra

|A1| · |A2| = 2 · 4 = 8

lehetosegunk van.

Horvath Gabor Analızis 2

Page 92: Anal zis 2 - Széchenyi István Universityhorvathg/bsc/gazdmath2.pdfHa f es g gra konja, ak ar t obbsz or is, atmeszi egym ast [a;b]-n, az a-ban es b-ben all tott fugg} oleges egyenesnek,

Feladat 22

Egy cukraszdaban negy fele fagylalt kaphato: csokolade, vanılia,puncs es citrom. Ket fele tolcsert kerhetunk: hagyomanyosat eskakaost. Hanyfele egy gombocos fagylaltot vasarolhatunk?Megoldas: Azt, hogy peldaul kakaos tolcserbe vettunk vanıliafagylaltot ıgy jelolhetjuk:

(kakaos, vanılia).Altalaban is, a vasarlasunk egy rendezett par segıtsegevel ırhato le,az elso tag azt adja meg, hogy milyen tolcsert valasztottunk, amasodik, hogy milyen fagylaltot. Ha

A1 = hagyomanyos, kakaos,

A2 = csokolade, vanılia, puncs, citrom,

akkor |A1| = 2, |A2| = 4, es a (Tetel 15) szerint a vasarlasra

|A1| · |A2| = 2 · 4 = 8

lehetosegunk van.Horvath Gabor Analızis 2

Page 93: Anal zis 2 - Széchenyi István Universityhorvathg/bsc/gazdmath2.pdfHa f es g gra konja, ak ar t obbsz or is, atmeszi egym ast [a;b]-n, az a-ban es b-ben all tott fugg} oleges egyenesnek,

Permutaciok

Tekintsuk az A = a, b, c , d halmazt. Ezeknek az elemeknek egypermutacioja

cabd ,

egy masikcdab.

Definıcio 25

n darab kulonbozo elem egy permutaciojan az n elem egysorbarendezeset ertjuk. Az n elem osszes permutacioinak szamatPn jeloli.

Tetel 17

Pn = n!,

ahol n! = 1 · 2 · . . . · n. Megallapodas szerint 0! = 1.

Horvath Gabor Analızis 2

Page 94: Anal zis 2 - Széchenyi István Universityhorvathg/bsc/gazdmath2.pdfHa f es g gra konja, ak ar t obbsz or is, atmeszi egym ast [a;b]-n, az a-ban es b-ben all tott fugg} oleges egyenesnek,

Ismetleses permutaciok

Az a, a, b, c , c , c, d elemek egy ismetleses permutacioja bacdacc ,egy masik cadcbac.

Definıcio 26

Adott n nem feltetlenul kulonbozo elem ugy, hogy az elso tıpusbolk1 darab, a masodik tıpusbol k2 darab, es ıgy tovabb, az s-ediktıpusbol ks darab van. Nyilvan k1 + k2 + · · ·+ ks = n. Ezeknek azelemeknek egy ismetleses permutaciojan az n elem egysorbarendezeset ertjuk. Az n elem osszes ismetlesespermutacioinak szamat Pk1,k2,...,ks

n jeloli.

Tetel 18

Pk1,k2,...,ksn =

n!

k1! · k2! · . . . · ks !.

Horvath Gabor Analızis 2

Page 95: Anal zis 2 - Széchenyi István Universityhorvathg/bsc/gazdmath2.pdfHa f es g gra konja, ak ar t obbsz or is, atmeszi egym ast [a;b]-n, az a-ban es b-ben all tott fugg} oleges egyenesnek,

Kombinaciok

Definıcio 27

Ha kivalasztunk n kulonbozo elem kozul k darabot ugy, hogy akivalasztott elemek sorrendje nem szamıt, akkor eloallıtottuk az nelem egy k-ad osztalyu ismetles nelkuli kombinaciojat. Az nkulonbozo elem osszes lehetseges k-ad osztalyu ismetles nelkulikombinacioinak szamat C k

n jeloli.

Vilagos, hogy 0 ≤ k ≤ n, es egy k-ad osztalyu ismetles nelkulikombinacio azonosıthato az n elem egy k elemu reszhalmazaval.

Tetel 19

C kn =

n!

k! · (n − k)!=

n · (n − 1) · . . . · (n − k + 1)

k!.

Konnyu meggyozodni rola, hogy C 0n = 1, C 1

n = n, C n−1n = n,

C nn = 1, sot altalaban is C k

n = C n−kn .

Horvath Gabor Analızis 2

Page 96: Anal zis 2 - Széchenyi István Universityhorvathg/bsc/gazdmath2.pdfHa f es g gra konja, ak ar t obbsz or is, atmeszi egym ast [a;b]-n, az a-ban es b-ben all tott fugg} oleges egyenesnek,

Binomialis egyutthatok

Definıcio 28

Az(n

k

)binomialis egyutthato az a + b osszeg - binom - n-edik

hatvanyanak a kifejteseben az an−k bk tag egyutthatoja.

Tetel 20 (Binomialis tetel)

(a + b)n =

=(n

0

)an +

(n

1

)an−1b +

(n

2

)an−2b2 + . . .+

( n

n − 1

)abn−1 +

(n

n

)bn =

=n∑

k=0

(n

k

)an−k bk .

Tetel 21 (n

k

)= C k

n .

Horvath Gabor Analızis 2

Page 97: Anal zis 2 - Széchenyi István Universityhorvathg/bsc/gazdmath2.pdfHa f es g gra konja, ak ar t obbsz or is, atmeszi egym ast [a;b]-n, az a-ban es b-ben all tott fugg} oleges egyenesnek,

Pascal haromszog

Tetel 22(n

k

)=

(n

n − k

),

(n + 1

k

)=

(n

k − 1

)+

(n

k

).

,

5. abra. Pascal haromszog.

Horvath Gabor Analızis 2

Page 98: Anal zis 2 - Széchenyi István Universityhorvathg/bsc/gazdmath2.pdfHa f es g gra konja, ak ar t obbsz or is, atmeszi egym ast [a;b]-n, az a-ban es b-ben all tott fugg} oleges egyenesnek,

Ismetleses kombinaciok

Definıcio 29

Ha kivalasztunk n kulonbozo elem kozul k darabot ugy, hogy akivalasztott elemek sorrendje nem szamıt, de az elemekismetlodhetnek is, akkor eloallıtottuk az n elem egy k-ad osztalyuismetleses kombinaciojat. Az n kulonbozo elem osszes lehetsegesk-ad osztalyu ismetleses kombinacioinak szamat C k,i

n jeloli.

Most persze k lehet nagyobb is, mint n.

Tetel 23

C k,in = C k

n+k−1 =

(n + k − 1

k

).

Horvath Gabor Analızis 2

Page 99: Anal zis 2 - Széchenyi István Universityhorvathg/bsc/gazdmath2.pdfHa f es g gra konja, ak ar t obbsz or is, atmeszi egym ast [a;b]-n, az a-ban es b-ben all tott fugg} oleges egyenesnek,

Variaciok

Definıcio 30

Ha kivalasztunk n kulonbozo elem kozul k darabot ugy, hogy akivalasztott elemek sorrendje is szamıt, akkor eloallıtottuk az nelem egy k-ad osztalyu ismetles nelkuli variaciojat. Az n kulonbozoelem osszes lehetseges k-ad osztalyu ismetles nelkuli variacioinakszamat V k

n jeloli.

Most tehat 0 ≤ k ≤ n.Peldaul az a, b, c, d elemek egy masod osztalyu ismetles nelkulivariacioja cd , egy masik dc .

Tetel 24

V kn = k! · C k

n = n · (n − 1) · . . . · (n − k + 1)︸ ︷︷ ︸k darab tenyezo

.

Horvath Gabor Analızis 2

Page 100: Anal zis 2 - Széchenyi István Universityhorvathg/bsc/gazdmath2.pdfHa f es g gra konja, ak ar t obbsz or is, atmeszi egym ast [a;b]-n, az a-ban es b-ben all tott fugg} oleges egyenesnek,

Ismetleses variaciok

Definıcio 31

Ha kivalasztunk n kulonbozo elem kozul k darabot ugy, hogy akivalasztott elemek sorrendje is szamıt es az elemek ismetlodhetnekis, akkor eloallıtottuk az n elem egy k-ad osztalyu ismetlesesvariaciojat. Az n kulonbozo elem osszes lehetseges k-ad osztalyuismetleses variacioinak szamat V k,i

n jeloli.

A k most lehet nagyobb is, mint n.Peldaul az a, b, c, d elemek egy masod osztalyu ismetlesesvariacioja cd , egy masik cc .

Tetel 25

V k,in = nk .

Horvath Gabor Analızis 2

Page 101: Anal zis 2 - Széchenyi István Universityhorvathg/bsc/gazdmath2.pdfHa f es g gra konja, ak ar t obbsz or is, atmeszi egym ast [a;b]-n, az a-ban es b-ben all tott fugg} oleges egyenesnek,

Feladat 23

A szazmeteres sıkfutas olimpiai dontojenek az osszes helyezesttekintve hany vegeredmenye lehet?

Megoldas: A resztvevoket azonosıthatjuk a palyajuk sorszamaval.Egy lehetseges befutasi sorrend az alabbi:

4 5 3 2 6 7 8 1

Ez az 1, 2, 3, 4, 5, 6, 7, 8 elemek egy ismetles nelkuli permutacioja.Vilagos, hogy befutasi sorrendbol pontosan annyi van, mint ilyenpermutaciobol. Ezert a feladat kerdesere a valasz:

P8 = 8! = 40320.

Horvath Gabor Analızis 2

Page 102: Anal zis 2 - Széchenyi István Universityhorvathg/bsc/gazdmath2.pdfHa f es g gra konja, ak ar t obbsz or is, atmeszi egym ast [a;b]-n, az a-ban es b-ben all tott fugg} oleges egyenesnek,

Feladat 23

A szazmeteres sıkfutas olimpiai dontojenek az osszes helyezesttekintve hany vegeredmenye lehet?Megoldas: A resztvevoket azonosıthatjuk a palyajuk sorszamaval.Egy lehetseges befutasi sorrend az alabbi:

4 5 3 2 6 7 8 1

Ez az 1, 2, 3, 4, 5, 6, 7, 8 elemek egy ismetles nelkuli permutacioja.Vilagos, hogy befutasi sorrendbol pontosan annyi van, mint ilyenpermutaciobol. Ezert a feladat kerdesere a valasz:

P8 = 8! = 40320.

Horvath Gabor Analızis 2

Page 103: Anal zis 2 - Széchenyi István Universityhorvathg/bsc/gazdmath2.pdfHa f es g gra konja, ak ar t obbsz or is, atmeszi egym ast [a;b]-n, az a-ban es b-ben all tott fugg} oleges egyenesnek,

Feladat 24

A MATEMATIKA szo betuinek egymas melle ırasaval hanykulonbozo, tobbnyire ertelmetlen ”szo” alkothato?

Megoldas: Peldaul egy lehetseges ”szo”:

E T A M A M I T A K .

Ez nyilvan a 10 felhasznalhato betu egy ismetleses permutacioja.Megszamoljuk, hogy hany elemtıpus van, es az egyes tıpusokbolhany elem van:

M betubol van 2 darab k1 = 2,A betubol van 3 darab k2 = 3,T betubol van 2 darab k3 = 2,E betubol van 1 darab k4 = 1,I betubol van 1 darab k5 = 1,K betubol van 1 darab k6 = 1.

Igy Pk1,k2,k3,k4,k5,k610 = 10!

2!·3!·2!·1!·1!·1! = 151200 halandzsa szo van.

Horvath Gabor Analızis 2

Page 104: Anal zis 2 - Széchenyi István Universityhorvathg/bsc/gazdmath2.pdfHa f es g gra konja, ak ar t obbsz or is, atmeszi egym ast [a;b]-n, az a-ban es b-ben all tott fugg} oleges egyenesnek,

Feladat 24

A MATEMATIKA szo betuinek egymas melle ırasaval hanykulonbozo, tobbnyire ertelmetlen ”szo” alkothato?Megoldas: Peldaul egy lehetseges ”szo”:

E T A M A M I T A K .

Ez nyilvan a 10 felhasznalhato betu egy ismetleses permutacioja.Megszamoljuk, hogy hany elemtıpus van, es az egyes tıpusokbolhany elem van:

M betubol van 2 darab k1 = 2,A betubol van 3 darab k2 = 3,T betubol van 2 darab k3 = 2,E betubol van 1 darab k4 = 1,I betubol van 1 darab k5 = 1,K betubol van 1 darab k6 = 1.

Igy Pk1,k2,k3,k4,k5,k610 = 10!

2!·3!·2!·1!·1!·1! = 151200 halandzsa szo van.

Horvath Gabor Analızis 2

Page 105: Anal zis 2 - Széchenyi István Universityhorvathg/bsc/gazdmath2.pdfHa f es g gra konja, ak ar t obbsz or is, atmeszi egym ast [a;b]-n, az a-ban es b-ben all tott fugg} oleges egyenesnek,

Feladat 25

Egy BKV buszon a jegyet ervenyesıto automata ugy van beallıtva,hogy negy mezot lukaszt a lehetseges kilenc kozul. Hanyfelebeallıtasa lehet a lukasztonak?

Megoldas: Egy lehetseges beallıtast mutat az alabbi abra.

l

l

l l

A lukaszto beallıtasakor, ha mar eldontottuk, hogy negy mezot foglukasztani, csak az szamıt, hogy melyik negy mezot valasztjuk ki.Ezert egy beallıtas azonosıthato a kilenc lehetseges pozıcio egynegyed osztalyu ismetles nelkuli kombinaciojaval. Lukasztobeallıtas tehat annyi van, ahany ilyen kombinacio, azaz

C 49 =

9!

4! · 5!= 126.

Horvath Gabor Analızis 2

Page 106: Anal zis 2 - Széchenyi István Universityhorvathg/bsc/gazdmath2.pdfHa f es g gra konja, ak ar t obbsz or is, atmeszi egym ast [a;b]-n, az a-ban es b-ben all tott fugg} oleges egyenesnek,

Feladat 25

Egy BKV buszon a jegyet ervenyesıto automata ugy van beallıtva,hogy negy mezot lukaszt a lehetseges kilenc kozul. Hanyfelebeallıtasa lehet a lukasztonak?Megoldas: Egy lehetseges beallıtast mutat az alabbi abra.

l

l

l l

A lukaszto beallıtasakor, ha mar eldontottuk, hogy negy mezot foglukasztani, csak az szamıt, hogy melyik negy mezot valasztjuk ki.Ezert egy beallıtas azonosıthato a kilenc lehetseges pozıcio egynegyed osztalyu ismetles nelkuli kombinaciojaval. Lukasztobeallıtas tehat annyi van, ahany ilyen kombinacio, azaz

C 49 =

9!

4! · 5!= 126.

Horvath Gabor Analızis 2

Page 107: Anal zis 2 - Széchenyi István Universityhorvathg/bsc/gazdmath2.pdfHa f es g gra konja, ak ar t obbsz or is, atmeszi egym ast [a;b]-n, az a-ban es b-ben all tott fugg} oleges egyenesnek,

Feladat 26

Egy cukraszdaban negy fele torta kaphato. Egy hat szeletbol allosutemeny csomagot szeretnenk vasarolni. Hanyfelekeppenallıthatjuk ossze a csomagot?

Megoldas: A tortaszeletek csomagon beluli sorrendje nyilvan nemszamıt, ıgy csak kombinaciokrol lehet szo. Mivel egy tortafajtaboltobb szeletet is valaszthatunk, ismetleses kombinacioval vandolgunk. Egy osszeallıtas tehat azonosıthato a negy elem egyhatod osztalyu ismetleses kombinaciojaval, tehat a valasz:

C 6,i4 =

(4 + 6− 1

6

)=

(9

6

)= 84.

Horvath Gabor Analızis 2

Page 108: Anal zis 2 - Széchenyi István Universityhorvathg/bsc/gazdmath2.pdfHa f es g gra konja, ak ar t obbsz or is, atmeszi egym ast [a;b]-n, az a-ban es b-ben all tott fugg} oleges egyenesnek,

Feladat 26

Egy cukraszdaban negy fele torta kaphato. Egy hat szeletbol allosutemeny csomagot szeretnenk vasarolni. Hanyfelekeppenallıthatjuk ossze a csomagot?Megoldas: A tortaszeletek csomagon beluli sorrendje nyilvan nemszamıt, ıgy csak kombinaciokrol lehet szo. Mivel egy tortafajtaboltobb szeletet is valaszthatunk, ismetleses kombinacioval vandolgunk. Egy osszeallıtas tehat azonosıthato a negy elem egyhatod osztalyu ismetleses kombinaciojaval, tehat a valasz:

C 6,i4 =

(4 + 6− 1

6

)=

(9

6

)= 84.

Horvath Gabor Analızis 2

Page 109: Anal zis 2 - Széchenyi István Universityhorvathg/bsc/gazdmath2.pdfHa f es g gra konja, ak ar t obbsz or is, atmeszi egym ast [a;b]-n, az a-ban es b-ben all tott fugg} oleges egyenesnek,

Feladat 27

A szazmeteres sıkfutas olimpiai dontojenek az ermeket tekintvehany vegeredmenye lehet?

Megoldas: A resztvevoket most is azonosıthatjuk a palyajuksorszamaval. Egy lehetseges befutasi sorrend az alabbi:

4 3 6

Ez az 1, 2, 3, 4, 5, 6, 7, 8 elemek egy harmad osztalyu ismetlesnelkuli variacioja. Vilagos, hogy vegeredmenybol pontosan annyivan, mint ilyen variaciobol. Tehat a valasz:

V 38 = 8 · 7 · 6 = 336.

Horvath Gabor Analızis 2

Page 110: Anal zis 2 - Széchenyi István Universityhorvathg/bsc/gazdmath2.pdfHa f es g gra konja, ak ar t obbsz or is, atmeszi egym ast [a;b]-n, az a-ban es b-ben all tott fugg} oleges egyenesnek,

Feladat 27

A szazmeteres sıkfutas olimpiai dontojenek az ermeket tekintvehany vegeredmenye lehet?Megoldas: A resztvevoket most is azonosıthatjuk a palyajuksorszamaval. Egy lehetseges befutasi sorrend az alabbi:

4 3 6

Ez az 1, 2, 3, 4, 5, 6, 7, 8 elemek egy harmad osztalyu ismetlesnelkuli variacioja. Vilagos, hogy vegeredmenybol pontosan annyivan, mint ilyen variaciobol. Tehat a valasz:

V 38 = 8 · 7 · 6 = 336.

Horvath Gabor Analızis 2

Page 111: Anal zis 2 - Széchenyi István Universityhorvathg/bsc/gazdmath2.pdfHa f es g gra konja, ak ar t obbsz or is, atmeszi egym ast [a;b]-n, az a-ban es b-ben all tott fugg} oleges egyenesnek,

Feladat 28

Hany olyan negyjegyu szam van amelyet csak az 1 es 2 szamjegyekhasznalataval felırhatunk?

Megoldas: Egy lehetseges ilyen negyjegyu szam:

1 2 2 1

Ez az 1, 2 elemek egy negyed osztalyu ismetleses variacioja.Vilagos, hogy a fenti feltetelnek eleget tevo negyjegyu szambolpontosan annyi van, mint ilyen ismetleses variaciobol. Tehat amegoldas:

V 42 = 24 = 16.

Horvath Gabor Analızis 2

Page 112: Anal zis 2 - Széchenyi István Universityhorvathg/bsc/gazdmath2.pdfHa f es g gra konja, ak ar t obbsz or is, atmeszi egym ast [a;b]-n, az a-ban es b-ben all tott fugg} oleges egyenesnek,

Feladat 28

Hany olyan negyjegyu szam van amelyet csak az 1 es 2 szamjegyekhasznalataval felırhatunk?Megoldas: Egy lehetseges ilyen negyjegyu szam:

1 2 2 1

Ez az 1, 2 elemek egy negyed osztalyu ismetleses variacioja.Vilagos, hogy a fenti feltetelnek eleget tevo negyjegyu szambolpontosan annyi van, mint ilyen ismetleses variaciobol. Tehat amegoldas:

V 42 = 24 = 16.

Horvath Gabor Analızis 2

Page 113: Anal zis 2 - Széchenyi István Universityhorvathg/bsc/gazdmath2.pdfHa f es g gra konja, ak ar t obbsz or is, atmeszi egym ast [a;b]-n, az a-ban es b-ben all tott fugg} oleges egyenesnek,

Ismetlodesnincs

Ismetlodesvan

Mindenelemet fel-hasznalunk

Permutaciok Ismetlesnelkulipermutacio

Ismetlesespermutacio

Nemhasznalunkfel mindenelemet

Sorrendnincs

Kombinaciok Ismetlesnelkulikom-binacio

Ismetleseskom-binacio

Sorrendvan

Variaciok Ismetlesnelkulivariacio

Ismetlesesvariacio

Persze nem szabad azt hinni, hogy minden kombinatorika feladatmegoldasa tisztan az eddig hat alapfogalom valamelyikenek azalkalmazasara vezet.

Horvath Gabor Analızis 2

Page 114: Anal zis 2 - Széchenyi István Universityhorvathg/bsc/gazdmath2.pdfHa f es g gra konja, ak ar t obbsz or is, atmeszi egym ast [a;b]-n, az a-ban es b-ben all tott fugg} oleges egyenesnek,

Feladat 29

Hanyfelekeppen ulhet le egy padra negy lany es negy fiu ugy, hogy sem ket lany, sem ket fiu nem ulhet egymasmellett?

Megoldas: A lanyokat jeloljuk az 1, 2, 3, 4 szamokkal, a fiukat az a, b, c, d betukkel. Felvaltva ketfeleelrendezesben ulhetnek: a bal szelso helyen lany ul vagy fiu:

L F L F L F L F vagy F L F L F L F L .

Egy lehetseges ultetes az elso tıpusbol:

3 a 1 c 2 d 4 b .

Ezt megkaphatjuk ugy, hogy eloszor leultetjuk csak a lanyokat a paratlan sorszamu helyekre:

3 1 2 4 .

Ezt nyilvan 4! modon tehetjuk meg. Minden lany leultetest kiegeszıthetunk egy teljes ultetesse azzal, hogy a parossorszamu helyekre minden lehetseges modon leultetjuk a fiukat. Erre is 4! lehetosegunk van. Mivel barmelyik lanyultetest barmelyik fiu ultetessel kiegeszıthetunk, az elso fajta ultetesbol 4! · 4! van.Ugyanez a helyzet a masodik tıpusu ultetesek eseten is, tehat a vegeredmeny:

4! · 4! + 4! · 4! = 1152.

Horvath Gabor Analızis 2

Page 115: Anal zis 2 - Széchenyi István Universityhorvathg/bsc/gazdmath2.pdfHa f es g gra konja, ak ar t obbsz or is, atmeszi egym ast [a;b]-n, az a-ban es b-ben all tott fugg} oleges egyenesnek,

Feladat 29

Hanyfelekeppen ulhet le egy padra negy lany es negy fiu ugy, hogy sem ket lany, sem ket fiu nem ulhet egymasmellett?Megoldas: A lanyokat jeloljuk az 1, 2, 3, 4 szamokkal, a fiukat az a, b, c, d betukkel. Felvaltva ketfeleelrendezesben ulhetnek: a bal szelso helyen lany ul vagy fiu:

L F L F L F L F vagy F L F L F L F L .

Egy lehetseges ultetes az elso tıpusbol:

3 a 1 c 2 d 4 b .

Ezt megkaphatjuk ugy, hogy eloszor leultetjuk csak a lanyokat a paratlan sorszamu helyekre:

3 1 2 4 .

Ezt nyilvan 4! modon tehetjuk meg. Minden lany leultetest kiegeszıthetunk egy teljes ultetesse azzal, hogy a parossorszamu helyekre minden lehetseges modon leultetjuk a fiukat. Erre is 4! lehetosegunk van. Mivel barmelyik lanyultetest barmelyik fiu ultetessel kiegeszıthetunk, az elso fajta ultetesbol 4! · 4! van.Ugyanez a helyzet a masodik tıpusu ultetesek eseten is, tehat a vegeredmeny:

4! · 4! + 4! · 4! = 1152.

Horvath Gabor Analızis 2

Page 116: Anal zis 2 - Széchenyi István Universityhorvathg/bsc/gazdmath2.pdfHa f es g gra konja, ak ar t obbsz or is, atmeszi egym ast [a;b]-n, az a-ban es b-ben all tott fugg} oleges egyenesnek,

Feladat 30

Hanyfelekeppen juthat el egy kiraly a sakktabla bal felso mezejerol a jobb also mezore, ha minden lepeskor csakjobbra vagy lefele lephet?

Megoldas:

A bal oldali abran berajzoltuk a kiraly egy lehetseges utvonalat. Ezeket a megengedett cikkcakkos utvonalakat kenemegszamolni. Ez eleg attekinthetetlennek tunik.Ezert minden utvonalat felbontunk lepesekre, es mivel minden lepes lefele vagy jobbra tett lepes, mindenutvonalhoz hozzarendelhetunk egy l es j betukbol allo jelsorozatot. Az elobbi utvonal eseten ezt latjuk a jobb oldaliabran, es az ehhez az utvonalhoz rendelt jelsorozat (kod): ljjjjjjjllllll .Minden ilyen jelsorozat 7 l betut es 7 j betut tartalmaz, tehat 14 betubol all, mert a nyolcadik sorbol 7 lepessel lekell jutni az elso sorba, es az elso oszlopbol 7 lepessel el kell jutni a nyolcadik oszlopba.Az is vilagos, hogy kulonbozo utvonalakhoz kulonbozo ilyen jelsorozat tartozik, es minden ilyen jelsorozat egyutvonalat kodol.Az ekvivalens halmazokrol szolo tetel ertelmeben tehat utvonal annyi van, mint ilyen jelsorozat.De konnyu felismerni, hogy egy ilyen jelsorozat egy ismetleses permutacio, ket tıpus van, mindegyik tıpusban hetszimbolum.Ezert a vegeredmeny:

14!

7! · 7!= 3432.

Horvath Gabor Analızis 2

Page 117: Anal zis 2 - Széchenyi István Universityhorvathg/bsc/gazdmath2.pdfHa f es g gra konja, ak ar t obbsz or is, atmeszi egym ast [a;b]-n, az a-ban es b-ben all tott fugg} oleges egyenesnek,

Feladat 30

Hanyfelekeppen juthat el egy kiraly a sakktabla bal felso mezejerol a jobb also mezore, ha minden lepeskor csakjobbra vagy lefele lephet?Megoldas:

A bal oldali abran berajzoltuk a kiraly egy lehetseges utvonalat. Ezeket a megengedett cikkcakkos utvonalakat kenemegszamolni. Ez eleg attekinthetetlennek tunik.Ezert minden utvonalat felbontunk lepesekre, es mivel minden lepes lefele vagy jobbra tett lepes, mindenutvonalhoz hozzarendelhetunk egy l es j betukbol allo jelsorozatot. Az elobbi utvonal eseten ezt latjuk a jobb oldaliabran, es az ehhez az utvonalhoz rendelt jelsorozat (kod): ljjjjjjjllllll .Minden ilyen jelsorozat 7 l betut es 7 j betut tartalmaz, tehat 14 betubol all, mert a nyolcadik sorbol 7 lepessel lekell jutni az elso sorba, es az elso oszlopbol 7 lepessel el kell jutni a nyolcadik oszlopba.Az is vilagos, hogy kulonbozo utvonalakhoz kulonbozo ilyen jelsorozat tartozik, es minden ilyen jelsorozat egyutvonalat kodol.Az ekvivalens halmazokrol szolo tetel ertelmeben tehat utvonal annyi van, mint ilyen jelsorozat.De konnyu felismerni, hogy egy ilyen jelsorozat egy ismetleses permutacio, ket tıpus van, mindegyik tıpusban hetszimbolum.Ezert a vegeredmeny:

14!

7! · 7!= 3432.

Horvath Gabor Analızis 2

Page 118: Anal zis 2 - Széchenyi István Universityhorvathg/bsc/gazdmath2.pdfHa f es g gra konja, ak ar t obbsz or is, atmeszi egym ast [a;b]-n, az a-ban es b-ben all tott fugg} oleges egyenesnek,

Feladat 31

Hanyfelekeppen rakhatunk be ket kicsi es harom nagy borıtekot egy negyfiokos asztal fiokjaiba?

Megoldas: Jogos a feltetelezes, hogy az azonos meretu borıtekokat nem tudjuk megkulonboztetni, de a fiokokatpersze igen. Az alabbi abran bemutatunk egy lehetseges elrendezest.

Ismet eljarhatunk ugy, hogy kulon megszamoljuk a kis borıtekok elhelyezkedesenek lehetosegeit, kulon a nagyborıtekoket; mivel minden kis borıtek elrendezeshez barmelyik nagy borıtek elrendezest tarsıthatjuk, ezek szorzataadja a vegeredmenyt.

A kis borıtekok elhelyezesenel ket esetet kell megkulonboztetnunk: mind a ket kis borıtek egy fiokba kerul, erre(

41

)lehetoseg van; a ket kis borıtek ket kulonbozo fiokba kerul, erre

(42

)lehetoseg van, hiszen a negy fiok kozul ki kell

valasztanunk azt a kettot, amelyekbe rakunk egy-egy kis borıtekot.A nagy borıtekok elhelyezesenel harom esetet kell megkulonboztetnunk: mind a harom nagy borıtek egy fiokba

kerul, erre(

41

)lehetoseg van; a harom nagy borıtek ket fiokba oszlik el, ekkor

(42

)felekepp kivalasztjuk azt a ket

fiokot, amelybe rakunk nagy borıtekot, kozulluk(

21

)felekeppen azt, amelybe kettot rakunk, ıgy

(42

)·(

21

)felekeppen oszolhatnak el a nagy borıtekok ket fiokba; vegul mind a harom nagy borıtek kulon fiokba kerul, erre

persze(

43

)lehetoseg van.

Tehat a vegeredmeny:

((4

1

)+

(4

2

))·((

4

1

)+

(4

2

)·(

2

1

)+

(4

3

))= (4 + 6) · (4 + 6 · 2 + 4) = 10 · 20 = 200.

Horvath Gabor Analızis 2

Page 119: Anal zis 2 - Széchenyi István Universityhorvathg/bsc/gazdmath2.pdfHa f es g gra konja, ak ar t obbsz or is, atmeszi egym ast [a;b]-n, az a-ban es b-ben all tott fugg} oleges egyenesnek,

Feladat 31

Hanyfelekeppen rakhatunk be ket kicsi es harom nagy borıtekot egy negyfiokos asztal fiokjaiba?Megoldas: Jogos a feltetelezes, hogy az azonos meretu borıtekokat nem tudjuk megkulonboztetni, de a fiokokatpersze igen. Az alabbi abran bemutatunk egy lehetseges elrendezest.

Ismet eljarhatunk ugy, hogy kulon megszamoljuk a kis borıtekok elhelyezkedesenek lehetosegeit, kulon a nagyborıtekoket; mivel minden kis borıtek elrendezeshez barmelyik nagy borıtek elrendezest tarsıthatjuk, ezek szorzataadja a vegeredmenyt.

A kis borıtekok elhelyezesenel ket esetet kell megkulonboztetnunk: mind a ket kis borıtek egy fiokba kerul, erre(

41

)lehetoseg van; a ket kis borıtek ket kulonbozo fiokba kerul, erre

(42

)lehetoseg van, hiszen a negy fiok kozul ki kell

valasztanunk azt a kettot, amelyekbe rakunk egy-egy kis borıtekot.A nagy borıtekok elhelyezesenel harom esetet kell megkulonboztetnunk: mind a harom nagy borıtek egy fiokba

kerul, erre(

41

)lehetoseg van; a harom nagy borıtek ket fiokba oszlik el, ekkor

(42

)felekepp kivalasztjuk azt a ket

fiokot, amelybe rakunk nagy borıtekot, kozulluk(

21

)felekeppen azt, amelybe kettot rakunk, ıgy

(42

)·(

21

)felekeppen oszolhatnak el a nagy borıtekok ket fiokba; vegul mind a harom nagy borıtek kulon fiokba kerul, erre

persze(

43

)lehetoseg van.

Tehat a vegeredmeny:

((4

1

)+

(4

2

))·((

4

1

)+

(4

2

)·(

2

1

)+

(4

3

))= (4 + 6) · (4 + 6 · 2 + 4) = 10 · 20 = 200.

Horvath Gabor Analızis 2

Page 120: Anal zis 2 - Széchenyi István Universityhorvathg/bsc/gazdmath2.pdfHa f es g gra konja, ak ar t obbsz or is, atmeszi egym ast [a;b]-n, az a-ban es b-ben all tott fugg} oleges egyenesnek,

Sztochasztikus es determinisztikus jelensegek

Determinisztikus jelenseg

Egy jelenseget determinisztikus jelensegnek hıvunk, ha adottkorulmenyek, feltetelek eseten a jelenseg mindig ugyanugy folyik le.Peldaul ilyenek a fizikai es kemiai kıserletek.

Sztochasztikus jelenseg

Egy jelenseget sztochasztikus jelensegnek hıvunk, ha adottkorulmenyek, feltetelek eseten a jelenseg lefolyasara tobb lehetosegis van. Peldaul ilyen, ha azt vizsgaljuk, hogy egy szabalyosdobokockaval dobva mennyi a dobott szam.

Azt gondolhatnank, hogy egy sztochasztikus jelensegdeterminisztikussa valik, ha a korulmenyeket, felteteleket elegpontosan ismerjuk, de ez nincs ıgy. Vannak eredendoensztochasztikus jelensegek, ilyen peldaul a radioaktıv bomlas. Nehapedig a korulmenyek pontossa tetelere nincs ido vagy anyagilagnem eri meg, celszerubb a jelenseget sztochasztikusnak tekinteni.

Horvath Gabor Analızis 2

Page 121: Anal zis 2 - Széchenyi István Universityhorvathg/bsc/gazdmath2.pdfHa f es g gra konja, ak ar t obbsz or is, atmeszi egym ast [a;b]-n, az a-ban es b-ben all tott fugg} oleges egyenesnek,

Kıserletek

Kıserlet

Legyen megadva korulmenyeknek, okonak, felteteleknek valamilyenK osszesege. Kıserleten K megvalosulasat ertjuk. Feltesszuk atovabbiakban, hogy a jelenseg a tekintetbe vett K korulmenykomplexumra nezve sztochasztikus. A kıserlet eredmenyet, amialtalaban valamilyen mennyiseg vagy tulajdonsag, mostantolkimenetelnek fogjuk hıvni.

Peldaul kockadobas eseten az a kıserlet, hogy feldobunk egyszabalyos kockat egy ures szobaban, a kocka hat oldala 1-tol 6-igmeg van szamozva. A fizika torvenyei miatt ez a kocka leesik apadlora es nyugalomba marad. A kıserlet kimenetelenek tekintsukazt a szamot, amely a kocka padloval erintkezo lapjaval szemkoztilapra van ırva.A valoszınusegszamıtas celja az ilyen kıserletek matematikaivizsgalata.

Horvath Gabor Analızis 2

Page 122: Anal zis 2 - Széchenyi István Universityhorvathg/bsc/gazdmath2.pdfHa f es g gra konja, ak ar t obbsz or is, atmeszi egym ast [a;b]-n, az a-ban es b-ben all tott fugg} oleges egyenesnek,

Veges esemenyalgebrak

Altalanos modell

Tekintsunk egy K kıserletet,amelynek veges sok kimenetelevan. A kıserlet kimeneteleitjelolje ω1, ω2, . . . ,ωn. A kıserletkimeneteleit elemi esemenyeknekhıvjuk, az elemi esemenyekhalmazat Ω jeloli. Tehat

Ω = ω1, ω2, . . . , ωn.

Konkret kıserlet

Legyen a kıserlet a kockadobas.Most a lehetseges kimenetelek:ω1: a dobott szam az 1-es,ω2: a dobott szam a 2-es,ω3: a dobott szam a 3-as,ω4: a dobott szam a 4-es,ω5: a dobott szam az 5-os,ω6: a dobott szam a 6-os.

De, meg egyszerubben, ωi -tjelolhetjuk i-vel is. Tehat most

Ω = 1, 2, 3, 4, 5, 6.

Horvath Gabor Analızis 2

Page 123: Anal zis 2 - Széchenyi István Universityhorvathg/bsc/gazdmath2.pdfHa f es g gra konja, ak ar t obbsz or is, atmeszi egym ast [a;b]-n, az a-ban es b-ben all tott fugg} oleges egyenesnek,

Veges esemenyalgebrak

Altalanos modell

De egy kıserlet soran nem csakaz lehet szamunkra fontos, hogymelyik elemi esemeny kovetkezikbe. Peldaul, ha a kıserlet egyfolyo vızszintejnek a megmeresenem az az igazan fontos, hogy azkonkretan mennyi, hamen, hogyaz jelent-e peldaul arvızveszelyt.Ezert definialjuk az esemenyfogalmat.

Konkret kıserlet

Peldaul lehet, hogy arra vagyunkkıvancsiak, hogy a dobott szamparos-e.

Horvath Gabor Analızis 2

Page 124: Anal zis 2 - Széchenyi István Universityhorvathg/bsc/gazdmath2.pdfHa f es g gra konja, ak ar t obbsz or is, atmeszi egym ast [a;b]-n, az a-ban es b-ben all tott fugg} oleges egyenesnek,

Veges esemenyalgebrak

Altalanos modell

Definıcio 32

Ezentul esemenynek nevezunkegy olyan logikai allıtast, amelyeta kıserlet bizonyos kimenetelei,azaz elemi esemenyei, igazzatesznek, a tobbi kimenetel pedighamissa.

Fontos, hogy a kıserlet minden kimenetele tegye igazzavagy hamissa az allıtast, kulonben nem tekintjuk ot akıserletre nezve esemenynek.

Ket logikai allıtast ugyanannak az esemenynek tekintunk,

ha mindkettot pontosan ugyanazok az elemi esemenyek

teszik igazza. Az esemenyek igazabol logikai allıtasok

halmazai.

Konkret kıserlet

A kockadobasra nezve esemenypeldaul az alabbi logikai allıtas:

A: ”a dobott szam 3-nal nagyobb”.

Peldaul ”a π egy irracionalisszam” logikai allıtas akockadobasra nezve nemesemeny, ennek igaz vagy hamisvolta nem attol fugg, hogymilyen kimenetel kovetkezik be.

Horvath Gabor Analızis 2

Page 125: Anal zis 2 - Széchenyi István Universityhorvathg/bsc/gazdmath2.pdfHa f es g gra konja, ak ar t obbsz or is, atmeszi egym ast [a;b]-n, az a-ban es b-ben all tott fugg} oleges egyenesnek,

Veges esemenyalgebrak

Altalanos modell

Legyen A egy esemeny. Aztmondjuk, hogy a kıserletvegrehajtasa soran az A esemenybekovetkezett, ha a kıserletnekolyan kinetele valosult meg,amelyik az A-val jelolt logikaiallıtast igazza teszi.

Konkret kıserlet

Peldaul, haA: ”a dobott szam 3-nal nagyobb”

esemeny, es a kıserletvegrehajtasa soran ”a dobottszam a 6-os” kimenetel valosultmeg, akkor bekovetkezett az Aesemeny.

Horvath Gabor Analızis 2

Page 126: Anal zis 2 - Széchenyi István Universityhorvathg/bsc/gazdmath2.pdfHa f es g gra konja, ak ar t obbsz or is, atmeszi egym ast [a;b]-n, az a-ban es b-ben all tott fugg} oleges egyenesnek,

Veges esemenyalgebrak

Altalanos modell

Definıcio 33

Azt a logikai allıtast, amelyetminden elemi esemeny igazzatesz biztos esemenynek hıvjuk.Azt, amelyet minden kimenetelhamissa tesz lehetetlenesemenynek hıvjuk.

Konkret kıserlet

Peldaul, azA: ”a dobott szam 7-nel nem nagyobb”

biztos esemeny,B: ”a dobott szam nagyobb, mint 8”

lehetetlen esemeny.

Horvath Gabor Analızis 2

Page 127: Anal zis 2 - Széchenyi István Universityhorvathg/bsc/gazdmath2.pdfHa f es g gra konja, ak ar t obbsz or is, atmeszi egym ast [a;b]-n, az a-ban es b-ben all tott fugg} oleges egyenesnek,

Veges esemenyalgebrak

Altalanos modell

Minden A esemeny jellemezhetoaz elemi esemenyek egyhalmazaval. Ebbe a halmazbapontosan azok az elemiesemenyek tartoznak bele,amelyek bekovetkezesekorbekovetkezik az A esemeny.Tehat gondolhatunk azesemenyekre ugy, hogy azok az Ωhalmaz reszhalmazai.

Konkret kıserlet

Tekintsuk azA: ”a dobott szam paratlan” esemenyt.

Ekkor A azonosıthato az 1, 3, 5elemi esemeny halmazzal:

A = 1, 3, 5.

Horvath Gabor Analızis 2

Page 128: Anal zis 2 - Széchenyi István Universityhorvathg/bsc/gazdmath2.pdfHa f es g gra konja, ak ar t obbsz or is, atmeszi egym ast [a;b]-n, az a-ban es b-ben all tott fugg} oleges egyenesnek,

Veges esemenyalgebrak

Altalanos modellMuveleteket definialunk az esemenyek kozott. Legyen A

es B ket esemeny (persze ugyanarra a kıserletre

vonatkozoan).

Definıcio 34

A + B-vel jeloljuk es ”A vagyB”-nek olvassuk azt azesemenyt, amelyik pontosanakkor kovetkezik be, habekovetkezik az A vagybekovetkezik a B esemeny.

Az A + B-t jellemzo elemiesemenyek halmaza az A-tjellemzo halmaz es a B-t jellemzohalmaz unioja.

Konkret kıserlet

Legyen A: ”a dobott szam paratlan”,

A = 1, 3, 5,

B: ”a dobott szam nagyobb, mint 4”

B = 5, 6.

EkkorA + B: ”a dobott szam paratlan vagy 4-nel nagyobb”

A + B = 1, 3, 5, 6.

Horvath Gabor Analızis 2

Page 129: Anal zis 2 - Széchenyi István Universityhorvathg/bsc/gazdmath2.pdfHa f es g gra konja, ak ar t obbsz or is, atmeszi egym ast [a;b]-n, az a-ban es b-ben all tott fugg} oleges egyenesnek,

Veges esemenyalgebrak

Altalanos modell

Definıcio 35

AB-vel jeloljuk es ”A es B”-nekolvassuk azt az esemenyt,amelyik pontosan akkorkovetkezik be, ha bekovetkezik azA es bekovetkezik a B esemeny.

Az AB-t jellemzo elemiesemenyek halmaza az A-tjellemzo halmaz es a B-t jellemzohalmaz metszete.

Konkret kıserletAB: ”a dobott szam paratlan es 4-nel nagyobb”

AB = 5.

Horvath Gabor Analızis 2

Page 130: Anal zis 2 - Széchenyi István Universityhorvathg/bsc/gazdmath2.pdfHa f es g gra konja, ak ar t obbsz or is, atmeszi egym ast [a;b]-n, az a-ban es b-ben all tott fugg} oleges egyenesnek,

Veges esemenyalgebrak

Altalanos modell

Definıcio 36

A-al jeloljuk es ”Aellentettje”-nek olvassuk azt azesemenyt, amelyik pontosanakkor kovetkezik be, ha nemkovetkezik be az A esemeny.

Az A-t jellemzo elemi esemenyek

halmaza az A-t jellemzo halmaz Ω-ra

vett komplementere, azaz Ω \ A. A

biztos esemenyhez az Ω halmazt, a

lehetetlen esemenyhez az ∅ halmazt,

es persze az ωi esemenyhez az ωihalmazt rendeljuk hozza.

Konkret kıserlet

A = 2, 4, 6.

Ez most az A: ”a dobott szam paros”

esemeny.

Horvath Gabor Analızis 2

Page 131: Anal zis 2 - Széchenyi István Universityhorvathg/bsc/gazdmath2.pdfHa f es g gra konja, ak ar t obbsz or is, atmeszi egym ast [a;b]-n, az a-ban es b-ben all tott fugg} oleges egyenesnek,

Veges esemenyalgebrak

Definıcio 37

Jeloljuk A-val a kıserlettel kapcsolatos osszes esemeny halmazat.

Azt kaptuk tehat, hogy, ha a K kıserletnek veges sok kimenetelevan, akkor a kıserlettel kapcsolatos osszes esemenyek halmaza nemmas, mint a Ω hatvanyhalmaza, amit P(Ω) fog jelolni. (Ωhatvanyhalmaza Ω osszes reszhalmazanak halmaza.)

Definıcio 38

Ha egy kıserlethez tartozo Ω esemenyter veges sok elemu, akkor az(Ω,A) rendezett part, ahol A = P(Ω), veges esemenyalgebranakhıvjuk.

Tetel 26

Ha egy veges esemenyalgebraban |Ω| = n, akkor |A| = 2n.

A tetel szerint kockadobas eseten 26 = 64 esemeny letezik.Horvath Gabor Analızis 2

Page 132: Anal zis 2 - Széchenyi István Universityhorvathg/bsc/gazdmath2.pdfHa f es g gra konja, ak ar t obbsz or is, atmeszi egym ast [a;b]-n, az a-ban es b-ben all tott fugg} oleges egyenesnek,

Veges esemenyalgebrak

Tetel 27

Legyen (Ω,A) veges esemenyalgebra, A,B,C ∈ A, (azaz legyenekA,B es C ugyanahhoz a kıserlethez tartozo esemenyek). Ekkor

1 AA = A,

2 AB = BA,

3 (AB)C = A(BC ) = ABC ,

4 A + A = A,

5 A + B = B + A,

6 (A + B) + C =A + (B + C ) = A + B + C ,

7 AA = ∅,

8 A + A = Ω,

9 AΩ = A,

10 A + Ω = Ω,

11 A∅ = ∅,12 A + ∅ = A,

13 A + B = A B, (de Morgan),

14 AB = A + B, (de Morgan).

Mivel az esemenyekre gondolhatunk ugy is, hogy azok elemi esemenyek

reszhalmazai, az esemenyek kozotti muveletek halmazmuveletek, ennek a

tetelnek a bizonyıtasa nem is lenne nagyon bonyolult.

Horvath Gabor Analızis 2

Page 133: Anal zis 2 - Széchenyi István Universityhorvathg/bsc/gazdmath2.pdfHa f es g gra konja, ak ar t obbsz or is, atmeszi egym ast [a;b]-n, az a-ban es b-ben all tott fugg} oleges egyenesnek,

Definıcio 39

Legyen A,B ∈ A. Ekkor B − A-val jeloljuk es ”B mınusz A”-nakazt az esemenyt, amelyik pontosan akkor kovetkezik be, habekovetkezik a B esemeny, de nem kovetkezik be az A esemeny.

Tetel 28

A definıcio alapjan B − A = BA.

Feladat 32

Igazoljuk, hogy tetszoleges A,B,C esemenyek esetenA(B − C ) = AB − AC .

Megoldas: Az ilyen ugynevezett esemenypolinomok kozotti egyenlosegekbizonyıtasanak egyik modja, hogy az elozo teteleket felhasznalva addigalakıtjuk az egyik oldalt, mıg a masik oldal nem lesz belole. Most a jobboldalt celszeru elkezdeni atalakıtani.

AB − AC = ABAC = AB(A + C ) = ABA + ABC = ∅+ ABC =

= A(BC ) = A(B − C ).

Horvath Gabor Analızis 2

Page 134: Anal zis 2 - Széchenyi István Universityhorvathg/bsc/gazdmath2.pdfHa f es g gra konja, ak ar t obbsz or is, atmeszi egym ast [a;b]-n, az a-ban es b-ben all tott fugg} oleges egyenesnek,

Definıcio 39

Legyen A,B ∈ A. Ekkor B − A-val jeloljuk es ”B mınusz A”-nakazt az esemenyt, amelyik pontosan akkor kovetkezik be, habekovetkezik a B esemeny, de nem kovetkezik be az A esemeny.

Tetel 28

A definıcio alapjan B − A = BA.

Feladat 32

Igazoljuk, hogy tetszoleges A,B,C esemenyek esetenA(B − C ) = AB − AC .Megoldas: Az ilyen ugynevezett esemenypolinomok kozotti egyenlosegekbizonyıtasanak egyik modja, hogy az elozo teteleket felhasznalva addigalakıtjuk az egyik oldalt, mıg a masik oldal nem lesz belole. Most a jobboldalt celszeru elkezdeni atalakıtani.

AB − AC = ABAC = AB(A + C ) = ABA + ABC = ∅+ ABC =

= A(BC ) = A(B − C ).

Horvath Gabor Analızis 2

Page 135: Anal zis 2 - Széchenyi István Universityhorvathg/bsc/gazdmath2.pdfHa f es g gra konja, ak ar t obbsz or is, atmeszi egym ast [a;b]-n, az a-ban es b-ben all tott fugg} oleges egyenesnek,

Veges esemenyalgebrak

Definıcio 40

Legyen A, B ∈ A. Ha az A esemeny bekovetkezesekor mindig bekovetkezik a B esemeny is, akkor azt mondjuk,hogy az A esemeny maga utan vonja a B esemenyt. Ennek jele A ⊂ B.

Tetel 29

Legyen (Ω,A) veges esemenyalgebra, A, B, C ∈ A. Ekkor

1 A ⊂ A + B,

2 ha A ⊂ B es B ⊂ A, akkor A = B,

3 AB ⊂ A,

4 ha A ⊂ C es B ⊂ C, akkor A + B ⊂ C,

5 ha A ⊂ B, akkor B = A + BA,

6 ha A ⊂ B, akkor B ⊂ A.

Definıcio 41

Ha AB = ∅, akkor az A es a B esemenyeket egymast kizaro esemenyeknek mondjuk.

Horvath Gabor Analızis 2

Page 136: Anal zis 2 - Széchenyi István Universityhorvathg/bsc/gazdmath2.pdfHa f es g gra konja, ak ar t obbsz or is, atmeszi egym ast [a;b]-n, az a-ban es b-ben all tott fugg} oleges egyenesnek,

Feladat 33

Feldobunk egy szabalyos penzermet. Irjuk fel a kıserlethez tartozo esemenyalgebrat.

Megoldas: A fej dobasat jelolje F , az ırast I . Ekkor Ω = F , I. Mivel Ω veges halmaz, A = P(Ω), azazA = ∅, F, I, F , I.

Feladat 34

Feldobunk egy szabalyos penzermet ketszer egymas utan. Irjuk fel a kıserlethez tartozo esemenyalgebrat.Megoldas: A fej dobasat jelolje F , az ırast I . Ekkor Ω elemei ket betubol allo jelsorozatok, Ω = FF , FI , II , IF.Mivel Ω most is veges halmaz A = P(Ω), azaz

A = ∅, FF, FI, II, IF, FF , FI, FF , II, FF , IF, FI , II, FI , IF, II , IF,

FF , FI , II, FF , FI , IF, FF , II , IF, FI , II , IF, FF , FI , II , IF

.

Feladat 35

Feldobunk ket egyforma szabalyos penzermet egyszerre. Irjuk fel a kıserlethez tartozo esemenyalgebrat.Megoldas: Ω elemei most ket betubol allo jelhalmazok, mert a dobasoknak nincs sorrendje, es az ermekmegkulonboztethetetlenek. Tehat Ω = F , F, F , I, I , I . Mivel Ω most is veges halmaz A = P(Ω), azaz

A = ∅, F , F, F , I, I , I, F , F, F , I, F , F, I , I, F , I, I , I,

F , F, F , I, I , I

.

Horvath Gabor Analızis 2

Page 137: Anal zis 2 - Széchenyi István Universityhorvathg/bsc/gazdmath2.pdfHa f es g gra konja, ak ar t obbsz or is, atmeszi egym ast [a;b]-n, az a-ban es b-ben all tott fugg} oleges egyenesnek,

Feladat 33

Feldobunk egy szabalyos penzermet. Irjuk fel a kıserlethez tartozo esemenyalgebrat.Megoldas: A fej dobasat jelolje F , az ırast I . Ekkor Ω = F , I. Mivel Ω veges halmaz, A = P(Ω), azazA = ∅, F, I, F , I.

Feladat 34

Feldobunk egy szabalyos penzermet ketszer egymas utan. Irjuk fel a kıserlethez tartozo esemenyalgebrat.Megoldas: A fej dobasat jelolje F , az ırast I . Ekkor Ω elemei ket betubol allo jelsorozatok, Ω = FF , FI , II , IF.Mivel Ω most is veges halmaz A = P(Ω), azaz

A = ∅, FF, FI, II, IF, FF , FI, FF , II, FF , IF, FI , II, FI , IF, II , IF,

FF , FI , II, FF , FI , IF, FF , II , IF, FI , II , IF, FF , FI , II , IF

.

Feladat 35

Feldobunk ket egyforma szabalyos penzermet egyszerre. Irjuk fel a kıserlethez tartozo esemenyalgebrat.Megoldas: Ω elemei most ket betubol allo jelhalmazok, mert a dobasoknak nincs sorrendje, es az ermekmegkulonboztethetetlenek. Tehat Ω = F , F, F , I, I , I . Mivel Ω most is veges halmaz A = P(Ω), azaz

A = ∅, F , F, F , I, I , I, F , F, F , I, F , F, I , I, F , I, I , I,

F , F, F , I, I , I

.

Horvath Gabor Analızis 2

Page 138: Anal zis 2 - Széchenyi István Universityhorvathg/bsc/gazdmath2.pdfHa f es g gra konja, ak ar t obbsz or is, atmeszi egym ast [a;b]-n, az a-ban es b-ben all tott fugg} oleges egyenesnek,

Feladat 33

Feldobunk egy szabalyos penzermet. Irjuk fel a kıserlethez tartozo esemenyalgebrat.Megoldas: A fej dobasat jelolje F , az ırast I . Ekkor Ω = F , I. Mivel Ω veges halmaz, A = P(Ω), azazA = ∅, F, I, F , I.

Feladat 34

Feldobunk egy szabalyos penzermet ketszer egymas utan. Irjuk fel a kıserlethez tartozo esemenyalgebrat.

Megoldas: A fej dobasat jelolje F , az ırast I . Ekkor Ω elemei ket betubol allo jelsorozatok, Ω = FF , FI , II , IF.Mivel Ω most is veges halmaz A = P(Ω), azaz

A = ∅, FF, FI, II, IF, FF , FI, FF , II, FF , IF, FI , II, FI , IF, II , IF,

FF , FI , II, FF , FI , IF, FF , II , IF, FI , II , IF, FF , FI , II , IF

.

Feladat 35

Feldobunk ket egyforma szabalyos penzermet egyszerre. Irjuk fel a kıserlethez tartozo esemenyalgebrat.Megoldas: Ω elemei most ket betubol allo jelhalmazok, mert a dobasoknak nincs sorrendje, es az ermekmegkulonboztethetetlenek. Tehat Ω = F , F, F , I, I , I . Mivel Ω most is veges halmaz A = P(Ω), azaz

A = ∅, F , F, F , I, I , I, F , F, F , I, F , F, I , I, F , I, I , I,

F , F, F , I, I , I

.

Horvath Gabor Analızis 2

Page 139: Anal zis 2 - Széchenyi István Universityhorvathg/bsc/gazdmath2.pdfHa f es g gra konja, ak ar t obbsz or is, atmeszi egym ast [a;b]-n, az a-ban es b-ben all tott fugg} oleges egyenesnek,

Feladat 33

Feldobunk egy szabalyos penzermet. Irjuk fel a kıserlethez tartozo esemenyalgebrat.Megoldas: A fej dobasat jelolje F , az ırast I . Ekkor Ω = F , I. Mivel Ω veges halmaz, A = P(Ω), azazA = ∅, F, I, F , I.

Feladat 34

Feldobunk egy szabalyos penzermet ketszer egymas utan. Irjuk fel a kıserlethez tartozo esemenyalgebrat.Megoldas: A fej dobasat jelolje F , az ırast I . Ekkor Ω elemei ket betubol allo jelsorozatok, Ω = FF , FI , II , IF.Mivel Ω most is veges halmaz A = P(Ω), azaz

A = ∅, FF, FI, II, IF, FF , FI, FF , II, FF , IF, FI , II, FI , IF, II , IF,

FF , FI , II, FF , FI , IF, FF , II , IF, FI , II , IF, FF , FI , II , IF

.

Feladat 35

Feldobunk ket egyforma szabalyos penzermet egyszerre. Irjuk fel a kıserlethez tartozo esemenyalgebrat.Megoldas: Ω elemei most ket betubol allo jelhalmazok, mert a dobasoknak nincs sorrendje, es az ermekmegkulonboztethetetlenek. Tehat Ω = F , F, F , I, I , I . Mivel Ω most is veges halmaz A = P(Ω), azaz

A = ∅, F , F, F , I, I , I, F , F, F , I, F , F, I , I, F , I, I , I,

F , F, F , I, I , I

.

Horvath Gabor Analızis 2

Page 140: Anal zis 2 - Széchenyi István Universityhorvathg/bsc/gazdmath2.pdfHa f es g gra konja, ak ar t obbsz or is, atmeszi egym ast [a;b]-n, az a-ban es b-ben all tott fugg} oleges egyenesnek,

Feladat 33

Feldobunk egy szabalyos penzermet. Irjuk fel a kıserlethez tartozo esemenyalgebrat.Megoldas: A fej dobasat jelolje F , az ırast I . Ekkor Ω = F , I. Mivel Ω veges halmaz, A = P(Ω), azazA = ∅, F, I, F , I.

Feladat 34

Feldobunk egy szabalyos penzermet ketszer egymas utan. Irjuk fel a kıserlethez tartozo esemenyalgebrat.Megoldas: A fej dobasat jelolje F , az ırast I . Ekkor Ω elemei ket betubol allo jelsorozatok, Ω = FF , FI , II , IF.Mivel Ω most is veges halmaz A = P(Ω), azaz

A = ∅, FF, FI, II, IF, FF , FI, FF , II, FF , IF, FI , II, FI , IF, II , IF,

FF , FI , II, FF , FI , IF, FF , II , IF, FI , II , IF, FF , FI , II , IF

.

Feladat 35

Feldobunk ket egyforma szabalyos penzermet egyszerre. Irjuk fel a kıserlethez tartozo esemenyalgebrat.

Megoldas: Ω elemei most ket betubol allo jelhalmazok, mert a dobasoknak nincs sorrendje, es az ermekmegkulonboztethetetlenek. Tehat Ω = F , F, F , I, I , I . Mivel Ω most is veges halmaz A = P(Ω), azaz

A = ∅, F , F, F , I, I , I, F , F, F , I, F , F, I , I, F , I, I , I,

F , F, F , I, I , I

.

Horvath Gabor Analızis 2

Page 141: Anal zis 2 - Széchenyi István Universityhorvathg/bsc/gazdmath2.pdfHa f es g gra konja, ak ar t obbsz or is, atmeszi egym ast [a;b]-n, az a-ban es b-ben all tott fugg} oleges egyenesnek,

Feladat 33

Feldobunk egy szabalyos penzermet. Irjuk fel a kıserlethez tartozo esemenyalgebrat.Megoldas: A fej dobasat jelolje F , az ırast I . Ekkor Ω = F , I. Mivel Ω veges halmaz, A = P(Ω), azazA = ∅, F, I, F , I.

Feladat 34

Feldobunk egy szabalyos penzermet ketszer egymas utan. Irjuk fel a kıserlethez tartozo esemenyalgebrat.Megoldas: A fej dobasat jelolje F , az ırast I . Ekkor Ω elemei ket betubol allo jelsorozatok, Ω = FF , FI , II , IF.Mivel Ω most is veges halmaz A = P(Ω), azaz

A = ∅, FF, FI, II, IF, FF , FI, FF , II, FF , IF, FI , II, FI , IF, II , IF,

FF , FI , II, FF , FI , IF, FF , II , IF, FI , II , IF, FF , FI , II , IF

.

Feladat 35

Feldobunk ket egyforma szabalyos penzermet egyszerre. Irjuk fel a kıserlethez tartozo esemenyalgebrat.Megoldas: Ω elemei most ket betubol allo jelhalmazok, mert a dobasoknak nincs sorrendje, es az ermekmegkulonboztethetetlenek. Tehat Ω = F , F, F , I, I , I . Mivel Ω most is veges halmaz A = P(Ω), azaz

A = ∅, F , F, F , I, I , I, F , F, F , I, F , F, I , I, F , I, I , I,

F , F, F , I, I , I

.

Horvath Gabor Analızis 2

Page 142: Anal zis 2 - Széchenyi István Universityhorvathg/bsc/gazdmath2.pdfHa f es g gra konja, ak ar t obbsz or is, atmeszi egym ast [a;b]-n, az a-ban es b-ben all tott fugg} oleges egyenesnek,

Feladat 36

Egy csomag magyar kartyabol kiveszunk egyszerre ket lapot. Hany elemu ekkor az Ω es az A halmaz?

Megoldas: A magyar kartyaban a lapoknak negy fele szıne lehet: piros zold, tok es makk, es minden szınben nyolclap van: hetes, nyolcas, kilences, tizes, also, felso, kiraly es asz. Ez osszesen 32 lap.Ha ezek kozul egyszerre kiveszunk kettot, akkor a lapok sorrendje nem szamıt, es ismetlodes nem lehet. A kıserletegy kimenetele tehat azonosıthato a 32 elem egy ismetles nelkuli masod osztalyu kombinaciojaval.

Tehat |Ω| = C 232 =

(322

)= 496, es ezert

|A| = 2496 = 20458691299350886687582435605172494701354012787769154934270571050

6008362275292159680204380770369009821930417757972504438076078534117837065833032974336.

Feladat 37

Egy csomag magyar kartyabol kiveszunk egymas utan ket lapot. Hany elemu ekkor az Ω es az A halmaz?Megoldas: Ha egymas utan veszunk ki ket lapot, akkor a lapok sorrendje szamıt, es ismetlodes nem lehet. Akıserlet egy kimenetele tehat azonosıthato a 32 elem egy ismetles nelkuli masod osztalyu variaciojaval.Tehat |Ω| = V 2

32 = 32 · 31 = 992, es |A| = 2992.

Feladat 38

Egy csomag magyar kartyabol kiveszunk egymas utan ket lapot, ugy, hogy az elsonek huzott lapot visszatesszuk.Hany elemu ekkor az Ω es az A halmaz?Megoldas: Ha egymas utan veszunk ki ket lapot, akkor a lapok sorrendje szamıt, es mivel visszatesszuk az elsonekhuzott lapot, ismetlodes is lehet. A kıserlet egy kimenetele tehat azonosıthato a 32 elem egy masod osztalyuismetleses variaciojaval.

Tehat |Ω| = V2,i32 = 322 = 1024, es |A| = 21024.

Horvath Gabor Analızis 2

Page 143: Anal zis 2 - Széchenyi István Universityhorvathg/bsc/gazdmath2.pdfHa f es g gra konja, ak ar t obbsz or is, atmeszi egym ast [a;b]-n, az a-ban es b-ben all tott fugg} oleges egyenesnek,

Feladat 36

Egy csomag magyar kartyabol kiveszunk egyszerre ket lapot. Hany elemu ekkor az Ω es az A halmaz?Megoldas: A magyar kartyaban a lapoknak negy fele szıne lehet: piros zold, tok es makk, es minden szınben nyolclap van: hetes, nyolcas, kilences, tizes, also, felso, kiraly es asz. Ez osszesen 32 lap.Ha ezek kozul egyszerre kiveszunk kettot, akkor a lapok sorrendje nem szamıt, es ismetlodes nem lehet. A kıserletegy kimenetele tehat azonosıthato a 32 elem egy ismetles nelkuli masod osztalyu kombinaciojaval.

Tehat |Ω| = C 232 =

(322

)= 496, es ezert

|A| = 2496 = 20458691299350886687582435605172494701354012787769154934270571050

6008362275292159680204380770369009821930417757972504438076078534117837065833032974336.

Feladat 37

Egy csomag magyar kartyabol kiveszunk egymas utan ket lapot. Hany elemu ekkor az Ω es az A halmaz?Megoldas: Ha egymas utan veszunk ki ket lapot, akkor a lapok sorrendje szamıt, es ismetlodes nem lehet. Akıserlet egy kimenetele tehat azonosıthato a 32 elem egy ismetles nelkuli masod osztalyu variaciojaval.Tehat |Ω| = V 2

32 = 32 · 31 = 992, es |A| = 2992.

Feladat 38

Egy csomag magyar kartyabol kiveszunk egymas utan ket lapot, ugy, hogy az elsonek huzott lapot visszatesszuk.Hany elemu ekkor az Ω es az A halmaz?Megoldas: Ha egymas utan veszunk ki ket lapot, akkor a lapok sorrendje szamıt, es mivel visszatesszuk az elsonekhuzott lapot, ismetlodes is lehet. A kıserlet egy kimenetele tehat azonosıthato a 32 elem egy masod osztalyuismetleses variaciojaval.

Tehat |Ω| = V2,i32 = 322 = 1024, es |A| = 21024.

Horvath Gabor Analızis 2

Page 144: Anal zis 2 - Széchenyi István Universityhorvathg/bsc/gazdmath2.pdfHa f es g gra konja, ak ar t obbsz or is, atmeszi egym ast [a;b]-n, az a-ban es b-ben all tott fugg} oleges egyenesnek,

Feladat 36

Egy csomag magyar kartyabol kiveszunk egyszerre ket lapot. Hany elemu ekkor az Ω es az A halmaz?Megoldas: A magyar kartyaban a lapoknak negy fele szıne lehet: piros zold, tok es makk, es minden szınben nyolclap van: hetes, nyolcas, kilences, tizes, also, felso, kiraly es asz. Ez osszesen 32 lap.Ha ezek kozul egyszerre kiveszunk kettot, akkor a lapok sorrendje nem szamıt, es ismetlodes nem lehet. A kıserletegy kimenetele tehat azonosıthato a 32 elem egy ismetles nelkuli masod osztalyu kombinaciojaval.

Tehat |Ω| = C 232 =

(322

)= 496, es ezert

|A| = 2496 = 20458691299350886687582435605172494701354012787769154934270571050

6008362275292159680204380770369009821930417757972504438076078534117837065833032974336.

Feladat 37

Egy csomag magyar kartyabol kiveszunk egymas utan ket lapot. Hany elemu ekkor az Ω es az A halmaz?

Megoldas: Ha egymas utan veszunk ki ket lapot, akkor a lapok sorrendje szamıt, es ismetlodes nem lehet. Akıserlet egy kimenetele tehat azonosıthato a 32 elem egy ismetles nelkuli masod osztalyu variaciojaval.Tehat |Ω| = V 2

32 = 32 · 31 = 992, es |A| = 2992.

Feladat 38

Egy csomag magyar kartyabol kiveszunk egymas utan ket lapot, ugy, hogy az elsonek huzott lapot visszatesszuk.Hany elemu ekkor az Ω es az A halmaz?Megoldas: Ha egymas utan veszunk ki ket lapot, akkor a lapok sorrendje szamıt, es mivel visszatesszuk az elsonekhuzott lapot, ismetlodes is lehet. A kıserlet egy kimenetele tehat azonosıthato a 32 elem egy masod osztalyuismetleses variaciojaval.

Tehat |Ω| = V2,i32 = 322 = 1024, es |A| = 21024.

Horvath Gabor Analızis 2

Page 145: Anal zis 2 - Széchenyi István Universityhorvathg/bsc/gazdmath2.pdfHa f es g gra konja, ak ar t obbsz or is, atmeszi egym ast [a;b]-n, az a-ban es b-ben all tott fugg} oleges egyenesnek,

Feladat 36

Egy csomag magyar kartyabol kiveszunk egyszerre ket lapot. Hany elemu ekkor az Ω es az A halmaz?Megoldas: A magyar kartyaban a lapoknak negy fele szıne lehet: piros zold, tok es makk, es minden szınben nyolclap van: hetes, nyolcas, kilences, tizes, also, felso, kiraly es asz. Ez osszesen 32 lap.Ha ezek kozul egyszerre kiveszunk kettot, akkor a lapok sorrendje nem szamıt, es ismetlodes nem lehet. A kıserletegy kimenetele tehat azonosıthato a 32 elem egy ismetles nelkuli masod osztalyu kombinaciojaval.

Tehat |Ω| = C 232 =

(322

)= 496, es ezert

|A| = 2496 = 20458691299350886687582435605172494701354012787769154934270571050

6008362275292159680204380770369009821930417757972504438076078534117837065833032974336.

Feladat 37

Egy csomag magyar kartyabol kiveszunk egymas utan ket lapot. Hany elemu ekkor az Ω es az A halmaz?Megoldas: Ha egymas utan veszunk ki ket lapot, akkor a lapok sorrendje szamıt, es ismetlodes nem lehet. Akıserlet egy kimenetele tehat azonosıthato a 32 elem egy ismetles nelkuli masod osztalyu variaciojaval.Tehat |Ω| = V 2

32 = 32 · 31 = 992, es |A| = 2992.

Feladat 38

Egy csomag magyar kartyabol kiveszunk egymas utan ket lapot, ugy, hogy az elsonek huzott lapot visszatesszuk.Hany elemu ekkor az Ω es az A halmaz?Megoldas: Ha egymas utan veszunk ki ket lapot, akkor a lapok sorrendje szamıt, es mivel visszatesszuk az elsonekhuzott lapot, ismetlodes is lehet. A kıserlet egy kimenetele tehat azonosıthato a 32 elem egy masod osztalyuismetleses variaciojaval.

Tehat |Ω| = V2,i32 = 322 = 1024, es |A| = 21024.

Horvath Gabor Analızis 2

Page 146: Anal zis 2 - Széchenyi István Universityhorvathg/bsc/gazdmath2.pdfHa f es g gra konja, ak ar t obbsz or is, atmeszi egym ast [a;b]-n, az a-ban es b-ben all tott fugg} oleges egyenesnek,

Feladat 36

Egy csomag magyar kartyabol kiveszunk egyszerre ket lapot. Hany elemu ekkor az Ω es az A halmaz?Megoldas: A magyar kartyaban a lapoknak negy fele szıne lehet: piros zold, tok es makk, es minden szınben nyolclap van: hetes, nyolcas, kilences, tizes, also, felso, kiraly es asz. Ez osszesen 32 lap.Ha ezek kozul egyszerre kiveszunk kettot, akkor a lapok sorrendje nem szamıt, es ismetlodes nem lehet. A kıserletegy kimenetele tehat azonosıthato a 32 elem egy ismetles nelkuli masod osztalyu kombinaciojaval.

Tehat |Ω| = C 232 =

(322

)= 496, es ezert

|A| = 2496 = 20458691299350886687582435605172494701354012787769154934270571050

6008362275292159680204380770369009821930417757972504438076078534117837065833032974336.

Feladat 37

Egy csomag magyar kartyabol kiveszunk egymas utan ket lapot. Hany elemu ekkor az Ω es az A halmaz?Megoldas: Ha egymas utan veszunk ki ket lapot, akkor a lapok sorrendje szamıt, es ismetlodes nem lehet. Akıserlet egy kimenetele tehat azonosıthato a 32 elem egy ismetles nelkuli masod osztalyu variaciojaval.Tehat |Ω| = V 2

32 = 32 · 31 = 992, es |A| = 2992.

Feladat 38

Egy csomag magyar kartyabol kiveszunk egymas utan ket lapot, ugy, hogy az elsonek huzott lapot visszatesszuk.Hany elemu ekkor az Ω es az A halmaz?

Megoldas: Ha egymas utan veszunk ki ket lapot, akkor a lapok sorrendje szamıt, es mivel visszatesszuk az elsonekhuzott lapot, ismetlodes is lehet. A kıserlet egy kimenetele tehat azonosıthato a 32 elem egy masod osztalyuismetleses variaciojaval.

Tehat |Ω| = V2,i32 = 322 = 1024, es |A| = 21024.

Horvath Gabor Analızis 2

Page 147: Anal zis 2 - Széchenyi István Universityhorvathg/bsc/gazdmath2.pdfHa f es g gra konja, ak ar t obbsz or is, atmeszi egym ast [a;b]-n, az a-ban es b-ben all tott fugg} oleges egyenesnek,

Feladat 36

Egy csomag magyar kartyabol kiveszunk egyszerre ket lapot. Hany elemu ekkor az Ω es az A halmaz?Megoldas: A magyar kartyaban a lapoknak negy fele szıne lehet: piros zold, tok es makk, es minden szınben nyolclap van: hetes, nyolcas, kilences, tizes, also, felso, kiraly es asz. Ez osszesen 32 lap.Ha ezek kozul egyszerre kiveszunk kettot, akkor a lapok sorrendje nem szamıt, es ismetlodes nem lehet. A kıserletegy kimenetele tehat azonosıthato a 32 elem egy ismetles nelkuli masod osztalyu kombinaciojaval.

Tehat |Ω| = C 232 =

(322

)= 496, es ezert

|A| = 2496 = 20458691299350886687582435605172494701354012787769154934270571050

6008362275292159680204380770369009821930417757972504438076078534117837065833032974336.

Feladat 37

Egy csomag magyar kartyabol kiveszunk egymas utan ket lapot. Hany elemu ekkor az Ω es az A halmaz?Megoldas: Ha egymas utan veszunk ki ket lapot, akkor a lapok sorrendje szamıt, es ismetlodes nem lehet. Akıserlet egy kimenetele tehat azonosıthato a 32 elem egy ismetles nelkuli masod osztalyu variaciojaval.Tehat |Ω| = V 2

32 = 32 · 31 = 992, es |A| = 2992.

Feladat 38

Egy csomag magyar kartyabol kiveszunk egymas utan ket lapot, ugy, hogy az elsonek huzott lapot visszatesszuk.Hany elemu ekkor az Ω es az A halmaz?Megoldas: Ha egymas utan veszunk ki ket lapot, akkor a lapok sorrendje szamıt, es mivel visszatesszuk az elsonekhuzott lapot, ismetlodes is lehet. A kıserlet egy kimenetele tehat azonosıthato a 32 elem egy masod osztalyuismetleses variaciojaval.

Tehat |Ω| = V2,i32 = 322 = 1024, es |A| = 21024.

Horvath Gabor Analızis 2

Page 148: Anal zis 2 - Széchenyi István Universityhorvathg/bsc/gazdmath2.pdfHa f es g gra konja, ak ar t obbsz or is, atmeszi egym ast [a;b]-n, az a-ban es b-ben all tott fugg} oleges egyenesnek,

Feladat 39

Tekintsuk a kovetkezo kıserletet: feldobunk egy szabalyos penzermet; ha fej, feldobjuk megegyszer, ha ırasfeldobjuk meg ketszer. Irjuk fel a kıserlethez tartozo Ω halmazt. Hany elemu most A?

Megoldas: Ω elemei most ket vagy harom betubol allo jelsorozatok. Akkor ket betus, ha elsore fejet dobtunk, esilyenkor a masodik betu lehet F es I is. Akkor harom betus, ha elsore ırast dobtunk, es ilyenkor a masodik esharmadik betubol allo ketbetus jelsorozat barmelyik lehet a negy lehetoseg kozul. TehatΩ = FF , FI , IFF , IFI , IIF , III.

Mivel |Ω| = 6, |A| = 26 = 64.

Feladat 40

Igazoljuk az elozo tetel 5. allıtasat, tehat azt, hogy A ⊂ B eseten B = A + BA.Megoldas: Azt fogjuk megmutatni, hogy B ⊂ A + BA es A + BA ⊂ B. Ebbol az elozo tetel masodik allıtasaszerint kovetkezik az egyenloseg.

1 Tegyuk fel, hogy bekovetkezik a B esemeny. Ekkor ket eset lehet:a) bekovetkezik az A esemeny is, de akkor bekovetkezik az A + BA, hiszen az elozo tetel elso allıtasaszerint A maga utan vonja minden olyan osszeg bekovetkezeset, aminek o az egyik tagja;b) nem kovetkezik be az A esemeny, de akkor bekovetkezik az A, es ıgy a BA esemeny is, ami ismet maga

utan vonja A + BA bekovetkezeset. Tehat mindket esetben B ⊂ A + BA.

2 Tegyuk fel, hogy bekovetkezik az A + BA esemeny. Ekkor vagy bekovetkezik az A esemeny, ami a feltetelszerint maga utan vonja a B-t vagy bekovetkezik a BA esemeny, de ez az elozo tetel harmadik allıtasaalapjan maga utan vonja mindket tenyezoje bekovetkezeset, amibol nekunk az a fontos, hogy bekovetkezika B esemeny. Tehat mindket esetben A + BA ⊂ B.

Horvath Gabor Analızis 2

Page 149: Anal zis 2 - Széchenyi István Universityhorvathg/bsc/gazdmath2.pdfHa f es g gra konja, ak ar t obbsz or is, atmeszi egym ast [a;b]-n, az a-ban es b-ben all tott fugg} oleges egyenesnek,

Feladat 39

Tekintsuk a kovetkezo kıserletet: feldobunk egy szabalyos penzermet; ha fej, feldobjuk megegyszer, ha ırasfeldobjuk meg ketszer. Irjuk fel a kıserlethez tartozo Ω halmazt. Hany elemu most A?Megoldas: Ω elemei most ket vagy harom betubol allo jelsorozatok. Akkor ket betus, ha elsore fejet dobtunk, esilyenkor a masodik betu lehet F es I is. Akkor harom betus, ha elsore ırast dobtunk, es ilyenkor a masodik esharmadik betubol allo ketbetus jelsorozat barmelyik lehet a negy lehetoseg kozul. TehatΩ = FF , FI , IFF , IFI , IIF , III.

Mivel |Ω| = 6, |A| = 26 = 64.

Feladat 40

Igazoljuk az elozo tetel 5. allıtasat, tehat azt, hogy A ⊂ B eseten B = A + BA.Megoldas: Azt fogjuk megmutatni, hogy B ⊂ A + BA es A + BA ⊂ B. Ebbol az elozo tetel masodik allıtasaszerint kovetkezik az egyenloseg.

1 Tegyuk fel, hogy bekovetkezik a B esemeny. Ekkor ket eset lehet:a) bekovetkezik az A esemeny is, de akkor bekovetkezik az A + BA, hiszen az elozo tetel elso allıtasaszerint A maga utan vonja minden olyan osszeg bekovetkezeset, aminek o az egyik tagja;b) nem kovetkezik be az A esemeny, de akkor bekovetkezik az A, es ıgy a BA esemeny is, ami ismet maga

utan vonja A + BA bekovetkezeset. Tehat mindket esetben B ⊂ A + BA.

2 Tegyuk fel, hogy bekovetkezik az A + BA esemeny. Ekkor vagy bekovetkezik az A esemeny, ami a feltetelszerint maga utan vonja a B-t vagy bekovetkezik a BA esemeny, de ez az elozo tetel harmadik allıtasaalapjan maga utan vonja mindket tenyezoje bekovetkezeset, amibol nekunk az a fontos, hogy bekovetkezika B esemeny. Tehat mindket esetben A + BA ⊂ B.

Horvath Gabor Analızis 2

Page 150: Anal zis 2 - Széchenyi István Universityhorvathg/bsc/gazdmath2.pdfHa f es g gra konja, ak ar t obbsz or is, atmeszi egym ast [a;b]-n, az a-ban es b-ben all tott fugg} oleges egyenesnek,

Feladat 39

Tekintsuk a kovetkezo kıserletet: feldobunk egy szabalyos penzermet; ha fej, feldobjuk megegyszer, ha ırasfeldobjuk meg ketszer. Irjuk fel a kıserlethez tartozo Ω halmazt. Hany elemu most A?Megoldas: Ω elemei most ket vagy harom betubol allo jelsorozatok. Akkor ket betus, ha elsore fejet dobtunk, esilyenkor a masodik betu lehet F es I is. Akkor harom betus, ha elsore ırast dobtunk, es ilyenkor a masodik esharmadik betubol allo ketbetus jelsorozat barmelyik lehet a negy lehetoseg kozul. TehatΩ = FF , FI , IFF , IFI , IIF , III.

Mivel |Ω| = 6, |A| = 26 = 64.

Feladat 40

Igazoljuk az elozo tetel 5. allıtasat, tehat azt, hogy A ⊂ B eseten B = A + BA.

Megoldas: Azt fogjuk megmutatni, hogy B ⊂ A + BA es A + BA ⊂ B. Ebbol az elozo tetel masodik allıtasaszerint kovetkezik az egyenloseg.

1 Tegyuk fel, hogy bekovetkezik a B esemeny. Ekkor ket eset lehet:a) bekovetkezik az A esemeny is, de akkor bekovetkezik az A + BA, hiszen az elozo tetel elso allıtasaszerint A maga utan vonja minden olyan osszeg bekovetkezeset, aminek o az egyik tagja;b) nem kovetkezik be az A esemeny, de akkor bekovetkezik az A, es ıgy a BA esemeny is, ami ismet maga

utan vonja A + BA bekovetkezeset. Tehat mindket esetben B ⊂ A + BA.

2 Tegyuk fel, hogy bekovetkezik az A + BA esemeny. Ekkor vagy bekovetkezik az A esemeny, ami a feltetelszerint maga utan vonja a B-t vagy bekovetkezik a BA esemeny, de ez az elozo tetel harmadik allıtasaalapjan maga utan vonja mindket tenyezoje bekovetkezeset, amibol nekunk az a fontos, hogy bekovetkezika B esemeny. Tehat mindket esetben A + BA ⊂ B.

Horvath Gabor Analızis 2

Page 151: Anal zis 2 - Széchenyi István Universityhorvathg/bsc/gazdmath2.pdfHa f es g gra konja, ak ar t obbsz or is, atmeszi egym ast [a;b]-n, az a-ban es b-ben all tott fugg} oleges egyenesnek,

Feladat 39

Tekintsuk a kovetkezo kıserletet: feldobunk egy szabalyos penzermet; ha fej, feldobjuk megegyszer, ha ırasfeldobjuk meg ketszer. Irjuk fel a kıserlethez tartozo Ω halmazt. Hany elemu most A?Megoldas: Ω elemei most ket vagy harom betubol allo jelsorozatok. Akkor ket betus, ha elsore fejet dobtunk, esilyenkor a masodik betu lehet F es I is. Akkor harom betus, ha elsore ırast dobtunk, es ilyenkor a masodik esharmadik betubol allo ketbetus jelsorozat barmelyik lehet a negy lehetoseg kozul. TehatΩ = FF , FI , IFF , IFI , IIF , III.

Mivel |Ω| = 6, |A| = 26 = 64.

Feladat 40

Igazoljuk az elozo tetel 5. allıtasat, tehat azt, hogy A ⊂ B eseten B = A + BA.Megoldas: Azt fogjuk megmutatni, hogy B ⊂ A + BA es A + BA ⊂ B. Ebbol az elozo tetel masodik allıtasaszerint kovetkezik az egyenloseg.

1 Tegyuk fel, hogy bekovetkezik a B esemeny. Ekkor ket eset lehet:a) bekovetkezik az A esemeny is, de akkor bekovetkezik az A + BA, hiszen az elozo tetel elso allıtasaszerint A maga utan vonja minden olyan osszeg bekovetkezeset, aminek o az egyik tagja;b) nem kovetkezik be az A esemeny, de akkor bekovetkezik az A, es ıgy a BA esemeny is, ami ismet maga

utan vonja A + BA bekovetkezeset. Tehat mindket esetben B ⊂ A + BA.

2 Tegyuk fel, hogy bekovetkezik az A + BA esemeny. Ekkor vagy bekovetkezik az A esemeny, ami a feltetelszerint maga utan vonja a B-t vagy bekovetkezik a BA esemeny, de ez az elozo tetel harmadik allıtasaalapjan maga utan vonja mindket tenyezoje bekovetkezeset, amibol nekunk az a fontos, hogy bekovetkezika B esemeny. Tehat mindket esetben A + BA ⊂ B.

Horvath Gabor Analızis 2

Page 152: Anal zis 2 - Széchenyi István Universityhorvathg/bsc/gazdmath2.pdfHa f es g gra konja, ak ar t obbsz or is, atmeszi egym ast [a;b]-n, az a-ban es b-ben all tott fugg} oleges egyenesnek,

Feladat 39

Tekintsuk a kovetkezo kıserletet: feldobunk egy szabalyos penzermet; ha fej, feldobjuk megegyszer, ha ırasfeldobjuk meg ketszer. Irjuk fel a kıserlethez tartozo Ω halmazt. Hany elemu most A?Megoldas: Ω elemei most ket vagy harom betubol allo jelsorozatok. Akkor ket betus, ha elsore fejet dobtunk, esilyenkor a masodik betu lehet F es I is. Akkor harom betus, ha elsore ırast dobtunk, es ilyenkor a masodik esharmadik betubol allo ketbetus jelsorozat barmelyik lehet a negy lehetoseg kozul. TehatΩ = FF , FI , IFF , IFI , IIF , III.

Mivel |Ω| = 6, |A| = 26 = 64.

Feladat 40

Igazoljuk az elozo tetel 5. allıtasat, tehat azt, hogy A ⊂ B eseten B = A + BA.Megoldas: Azt fogjuk megmutatni, hogy B ⊂ A + BA es A + BA ⊂ B. Ebbol az elozo tetel masodik allıtasaszerint kovetkezik az egyenloseg.

1 Tegyuk fel, hogy bekovetkezik a B esemeny. Ekkor ket eset lehet:a) bekovetkezik az A esemeny is, de akkor bekovetkezik az A + BA, hiszen az elozo tetel elso allıtasaszerint A maga utan vonja minden olyan osszeg bekovetkezeset, aminek o az egyik tagja;b) nem kovetkezik be az A esemeny, de akkor bekovetkezik az A, es ıgy a BA esemeny is, ami ismet maga

utan vonja A + BA bekovetkezeset. Tehat mindket esetben B ⊂ A + BA.

2 Tegyuk fel, hogy bekovetkezik az A + BA esemeny. Ekkor vagy bekovetkezik az A esemeny, ami a feltetelszerint maga utan vonja a B-t vagy bekovetkezik a BA esemeny, de ez az elozo tetel harmadik allıtasaalapjan maga utan vonja mindket tenyezoje bekovetkezeset, amibol nekunk az a fontos, hogy bekovetkezika B esemeny. Tehat mindket esetben A + BA ⊂ B.

Horvath Gabor Analızis 2

Page 153: Anal zis 2 - Széchenyi István Universityhorvathg/bsc/gazdmath2.pdfHa f es g gra konja, ak ar t obbsz or is, atmeszi egym ast [a;b]-n, az a-ban es b-ben all tott fugg} oleges egyenesnek,

Feladat 39

Tekintsuk a kovetkezo kıserletet: feldobunk egy szabalyos penzermet; ha fej, feldobjuk megegyszer, ha ırasfeldobjuk meg ketszer. Irjuk fel a kıserlethez tartozo Ω halmazt. Hany elemu most A?Megoldas: Ω elemei most ket vagy harom betubol allo jelsorozatok. Akkor ket betus, ha elsore fejet dobtunk, esilyenkor a masodik betu lehet F es I is. Akkor harom betus, ha elsore ırast dobtunk, es ilyenkor a masodik esharmadik betubol allo ketbetus jelsorozat barmelyik lehet a negy lehetoseg kozul. TehatΩ = FF , FI , IFF , IFI , IIF , III.

Mivel |Ω| = 6, |A| = 26 = 64.

Feladat 40

Igazoljuk az elozo tetel 5. allıtasat, tehat azt, hogy A ⊂ B eseten B = A + BA.Megoldas: Azt fogjuk megmutatni, hogy B ⊂ A + BA es A + BA ⊂ B. Ebbol az elozo tetel masodik allıtasaszerint kovetkezik az egyenloseg.

1 Tegyuk fel, hogy bekovetkezik a B esemeny. Ekkor ket eset lehet:a) bekovetkezik az A esemeny is, de akkor bekovetkezik az A + BA, hiszen az elozo tetel elso allıtasaszerint A maga utan vonja minden olyan osszeg bekovetkezeset, aminek o az egyik tagja;b) nem kovetkezik be az A esemeny, de akkor bekovetkezik az A, es ıgy a BA esemeny is, ami ismet maga

utan vonja A + BA bekovetkezeset. Tehat mindket esetben B ⊂ A + BA.

2 Tegyuk fel, hogy bekovetkezik az A + BA esemeny. Ekkor vagy bekovetkezik az A esemeny, ami a feltetelszerint maga utan vonja a B-t vagy bekovetkezik a BA esemeny, de ez az elozo tetel harmadik allıtasaalapjan maga utan vonja mindket tenyezoje bekovetkezeset, amibol nekunk az a fontos, hogy bekovetkezika B esemeny. Tehat mindket esetben A + BA ⊂ B.

Horvath Gabor Analızis 2

Page 154: Anal zis 2 - Széchenyi István Universityhorvathg/bsc/gazdmath2.pdfHa f es g gra konja, ak ar t obbsz or is, atmeszi egym ast [a;b]-n, az a-ban es b-ben all tott fugg} oleges egyenesnek,

Feladat 41

Igazoljuk, hogy tetszoleges A, B, C ∈ A esemenyekre A− (A− (B − C)) = ABC.

Megoldas: A bal oldali kifejezest fogjuk atalakıtani, egyszerusıteni, mert az tunik bonyolultabbnak. Belulrolhaladunk kifele. Eloszor is B − C = BC, ıgy

A− (B − C) = A− (BC) = ABC =︸︷︷︸de Morgan

A(B + C) = AB + AC .

Tehat

A− (A− (B − C)) = A(

AB + AC)

=︸︷︷︸de Morgan

AAB AC = A(A + B)(A + C) =

= (AA + AB)(A + C) = AB(A + C) = ABA + ABC = ABC .

Feladat 42

Igazoljuk, hogy tetszoleges A, B, C ∈ A esemenyekre (A + B)(A + C)(B + C) = AB + AC + BC.Megoldas: Felhasznaljuk, hogy az esemenyek szorzasa disztributıv az osszeadasra nezve. Ezert

(A + B)(A + C) = AA + BA + AC + BC = A + AB + AC + BC ,

ıgy

(A + B)(A + C)(B + C) = (A + AB + AC + BC)(B + C) = AB + ABB + ACB + BCB + AC + ABC + ACC + BCC =

= AB + AB + ABC + BC + AC + ABC + AC + BC = AB + AC + (BC + ABC) = AB + AC + BC .

Horvath Gabor Analızis 2

Page 155: Anal zis 2 - Széchenyi István Universityhorvathg/bsc/gazdmath2.pdfHa f es g gra konja, ak ar t obbsz or is, atmeszi egym ast [a;b]-n, az a-ban es b-ben all tott fugg} oleges egyenesnek,

Feladat 41

Igazoljuk, hogy tetszoleges A, B, C ∈ A esemenyekre A− (A− (B − C)) = ABC.Megoldas: A bal oldali kifejezest fogjuk atalakıtani, egyszerusıteni, mert az tunik bonyolultabbnak. Belulrolhaladunk kifele. Eloszor is B − C = BC, ıgy

A− (B − C) = A− (BC) = ABC =︸︷︷︸de Morgan

A(B + C) = AB + AC .

Tehat

A− (A− (B − C)) = A(

AB + AC)

=︸︷︷︸de Morgan

AAB AC = A(A + B)(A + C) =

= (AA + AB)(A + C) = AB(A + C) = ABA + ABC = ABC .

Feladat 42

Igazoljuk, hogy tetszoleges A, B, C ∈ A esemenyekre (A + B)(A + C)(B + C) = AB + AC + BC.Megoldas: Felhasznaljuk, hogy az esemenyek szorzasa disztributıv az osszeadasra nezve. Ezert

(A + B)(A + C) = AA + BA + AC + BC = A + AB + AC + BC ,

ıgy

(A + B)(A + C)(B + C) = (A + AB + AC + BC)(B + C) = AB + ABB + ACB + BCB + AC + ABC + ACC + BCC =

= AB + AB + ABC + BC + AC + ABC + AC + BC = AB + AC + (BC + ABC) = AB + AC + BC .

Horvath Gabor Analızis 2

Page 156: Anal zis 2 - Széchenyi István Universityhorvathg/bsc/gazdmath2.pdfHa f es g gra konja, ak ar t obbsz or is, atmeszi egym ast [a;b]-n, az a-ban es b-ben all tott fugg} oleges egyenesnek,

Feladat 41

Igazoljuk, hogy tetszoleges A, B, C ∈ A esemenyekre A− (A− (B − C)) = ABC.Megoldas: A bal oldali kifejezest fogjuk atalakıtani, egyszerusıteni, mert az tunik bonyolultabbnak. Belulrolhaladunk kifele. Eloszor is B − C = BC, ıgy

A− (B − C) = A− (BC) = ABC =︸︷︷︸de Morgan

A(B + C) = AB + AC .

Tehat

A− (A− (B − C)) = A(

AB + AC)

=︸︷︷︸de Morgan

AAB AC = A(A + B)(A + C) =

= (AA + AB)(A + C) = AB(A + C) = ABA + ABC = ABC .

Feladat 42

Igazoljuk, hogy tetszoleges A, B, C ∈ A esemenyekre (A + B)(A + C)(B + C) = AB + AC + BC.

Megoldas: Felhasznaljuk, hogy az esemenyek szorzasa disztributıv az osszeadasra nezve. Ezert

(A + B)(A + C) = AA + BA + AC + BC = A + AB + AC + BC ,

ıgy

(A + B)(A + C)(B + C) = (A + AB + AC + BC)(B + C) = AB + ABB + ACB + BCB + AC + ABC + ACC + BCC =

= AB + AB + ABC + BC + AC + ABC + AC + BC = AB + AC + (BC + ABC) = AB + AC + BC .

Horvath Gabor Analızis 2

Page 157: Anal zis 2 - Széchenyi István Universityhorvathg/bsc/gazdmath2.pdfHa f es g gra konja, ak ar t obbsz or is, atmeszi egym ast [a;b]-n, az a-ban es b-ben all tott fugg} oleges egyenesnek,

Feladat 41

Igazoljuk, hogy tetszoleges A, B, C ∈ A esemenyekre A− (A− (B − C)) = ABC.Megoldas: A bal oldali kifejezest fogjuk atalakıtani, egyszerusıteni, mert az tunik bonyolultabbnak. Belulrolhaladunk kifele. Eloszor is B − C = BC, ıgy

A− (B − C) = A− (BC) = ABC =︸︷︷︸de Morgan

A(B + C) = AB + AC .

Tehat

A− (A− (B − C)) = A(

AB + AC)

=︸︷︷︸de Morgan

AAB AC = A(A + B)(A + C) =

= (AA + AB)(A + C) = AB(A + C) = ABA + ABC = ABC .

Feladat 42

Igazoljuk, hogy tetszoleges A, B, C ∈ A esemenyekre (A + B)(A + C)(B + C) = AB + AC + BC.Megoldas: Felhasznaljuk, hogy az esemenyek szorzasa disztributıv az osszeadasra nezve. Ezert

(A + B)(A + C) = AA + BA + AC + BC = A + AB + AC + BC ,

ıgy

(A + B)(A + C)(B + C) = (A + AB + AC + BC)(B + C) = AB + ABB + ACB + BCB + AC + ABC + ACC + BCC =

= AB + AB + ABC + BC + AC + ABC + AC + BC = AB + AC + (BC + ABC) = AB + AC + BC .

Horvath Gabor Analızis 2

Page 158: Anal zis 2 - Széchenyi István Universityhorvathg/bsc/gazdmath2.pdfHa f es g gra konja, ak ar t obbsz or is, atmeszi egym ast [a;b]-n, az a-ban es b-ben all tott fugg} oleges egyenesnek,

Veges valoszınusegalgebrak

Tekintsunk egy (Ω,A) veges esemenyalgebrat, ami a K kıserletheztartozik. Legyen A ∈ A. Hajtsuk vegre a kıserletet n-szer.

Definıcio 42

Tegyuk fel, hogy az n kıserletet kozul k kıserlet soran bekovetkezett az Aesemeny, n − k esetben nem, tehat n − k-szor az A esemeny kovetkezettbe. Ekkor a k szamot az A esemeny gyakorisaganak, a k

n szamot az Aesemeny relatıv gyakorisaganak hıvjuk ebben a kıserletsorozatban.

Ezutan tekintsuk a kovetkezo procedurat: hajtsuk vegre K -t egyszer es

szamoljuk ki A relatıv gyakorisagat, hajtsuk vegre K -t megegyszer es

szamoljuk ki A relatıv gyakorisagat a kapott ket kıserletbol allo

kıserletsorozatban, hajtsuk vegre K -t megegyszer es szamoljuk ki A

relatıv gyakorisagat a kapott harom kıserletbol allo kıserletsorozatban, es

ıgy tovabb.

Horvath Gabor Analızis 2

Page 159: Anal zis 2 - Széchenyi István Universityhorvathg/bsc/gazdmath2.pdfHa f es g gra konja, ak ar t obbsz or is, atmeszi egym ast [a;b]-n, az a-ban es b-ben all tott fugg} oleges egyenesnek,

Veges valoszınusegalgebrak

Tapasztalati teny

Ekkor a relatıv gyakorisagok egy bizonyos szam korul ingadoznak.Minel hosszabb a kiserletsorozat elemszama annal kisebbek azingadozasok. Sot, ha ismet vegrehajtjuk ezt a procedurat, a relatıvgyakorisagok megint ingadozni fognak, es megint ugyanazon szamkorul, mint elobb.

Peldaul egy szabalyos erme husz egymas utani feldobasakor aFIIFFIFIFIFFIIFIIFIF eredmenyt kaptuk. A fej dobasanak relatıvgyakorisaga ebben a kıserletsorozatban ıgy alakult:

Horvath Gabor Analızis 2

Page 160: Anal zis 2 - Széchenyi István Universityhorvathg/bsc/gazdmath2.pdfHa f es g gra konja, ak ar t obbsz or is, atmeszi egym ast [a;b]-n, az a-ban es b-ben all tott fugg} oleges egyenesnek,

Veges valoszınusegalgebrak

Definıcio 43

Azt a szamot, ami korul a relatıv gyakorisag ingadozikvaloszınusegnek hıvjuk.

Valoszınuseg

A tapasztalat tehat azt mutatja, hogy minden K veges sokkimenetellel rendelkezo kıserlet eseten a K -hoz tartozo (Ω,A)veges esemenyalgebra minden A ∈ A esemenyehez hozza vanrendelve egy P(A) szam, az A esemeny valoszınusege, ami korul Arelatıv gyakorisaga ingadozik. A valoszınusegre tehat ugy isgondolhatunk, hogy az egy P : A −→ R fuggveny, ami az Aesemenyhez az esemeny P(A) valoszınuseget rendeli.

Vizsgaljuk meg, hogy milyen tulajdonsagokkal rendelkezik ez afuggveny.

Horvath Gabor Analızis 2

Page 161: Anal zis 2 - Széchenyi István Universityhorvathg/bsc/gazdmath2.pdfHa f es g gra konja, ak ar t obbsz or is, atmeszi egym ast [a;b]-n, az a-ban es b-ben all tott fugg} oleges egyenesnek,

Veges valoszınusegalgebrak

Ha egy n elemu kıserletsorozatban az A esemeny k-szorkovetkezett be, akkor 0 ≤ k ≤ n, ezert 0 ≤ k

n ≤ 1, amibolnyilvanvalo, hogy 0 ≤ P(A) ≤ 1 is teljesul.

A biztos esemeny relatıv gyakorisaga 1, es ıgy P(Ω) = 1 isigaz. A lehetetlen esemeny relatıv gyakorisaga 0, es ıgyP(∅) = 0 is igaz.

Ha A es B ugyanannak a kıserletnek ket egymast kizaroesemenye, azaz AB = ∅, es az A gyakorisaga kA, a Bgyakorisaga kB , akkor az A + B esemeny gyakorisagakA+B = kA + kB , relatıv gyakorisaga pedig

kA+B

n=

kA + kB

n=

kA

n+

kB

n,

ezert teljesulni kell, hogy

P(A + B) = P(A) + P(B).

Horvath Gabor Analızis 2

Page 162: Anal zis 2 - Széchenyi István Universityhorvathg/bsc/gazdmath2.pdfHa f es g gra konja, ak ar t obbsz or is, atmeszi egym ast [a;b]-n, az a-ban es b-ben all tott fugg} oleges egyenesnek,

Veges valoszınusegalgebrak

Ezek miatt a tapasztalatok miatt a kovetkezo axiomakat fogadjukel.

A valoszınusegszamıtas axiomai veges Ω eseten

Adott egy K kıserlethez tartozo (Ω,A) veges esemenyalgebra.

1 Minden A ∈ A esemenyehez hozza van rendelve egynemnegatıv P(A) szam, az A esemeny valoszınusege.

2 A biztos esemeny valoszınusege 1, azaz P(Ω) = 1.

3 Ha A es B egymast kizaro esemenyek, azaz AB = ∅, akkorP(A + B) = P(A) + P(B).

Definıcio 44

Egy olyan (Ω,A) veges esemenyalgebrat, ahol minden A ∈ Aesemenyehez hozza van rendelve egy P(A) szam, ugy, hogy ez ahozzarendeles eleget tesz az elobbi harom axiomanak, vegesvaloszınusegalgebranak hıvunk, es (Ω,A,P) fogja jelolni.

Horvath Gabor Analızis 2

Page 163: Anal zis 2 - Széchenyi István Universityhorvathg/bsc/gazdmath2.pdfHa f es g gra konja, ak ar t obbsz or is, atmeszi egym ast [a;b]-n, az a-ban es b-ben all tott fugg} oleges egyenesnek,

Az axiomak kovetkezmenyei

Az axiomakbol bebizonyıthatok a kovetkezok:

Tetel 30

1 Ha A ⊂ B, akkor P(A) ≤ P(B).

2 Minden A esemenyre 0 ≤ P(A) ≤ 1.

3 Minden A esemenyre P(A) + P(A) = 1.

4 P(∅) = 0.

5 Tetszoleges A,B esemenyekreP(A + B) = P(A) + P(B)− P(AB).

6 Ha A ⊂ B, akkor P(B − A) = P(B)− P(A).

7 Tetszoleges A,B esemenyekre P(A− B) = P(A)− P(AB).

Definıcio 45

Az A1,A2, . . . ,An esemenyek egymast paronkent kizarjak, haAi Aj = ∅, minden i 6= j eseten.

Horvath Gabor Analızis 2

Page 164: Anal zis 2 - Széchenyi István Universityhorvathg/bsc/gazdmath2.pdfHa f es g gra konja, ak ar t obbsz or is, atmeszi egym ast [a;b]-n, az a-ban es b-ben all tott fugg} oleges egyenesnek,

Teljes esemenyrendszer

Tetel 31

Ha az A1,A2, . . . ,An esemenyek egymast paronkent kizarjak, akkorP(A1 + A2 + . . .+ An) = P(A1) + P(A2) + . . .+ P(An).

Definıcio 46

Az A1,A2, . . . ,An esemenyek teljes esemenyrendszert alkotnak, ha

1 egymast paronkent kizarjak,

2 A1 + A2 + . . .+ An = Ω.

Tehat, ha A1,A2, . . . ,An teljes esemenyrendszer, akkor mindigbekovetkezik valamelyik Ai esemeny, de csak az egyik.

Tetel 32

Ha az A1,A2, . . . ,An teljes esemenyrendszer, akkor

P(A1) + P(A2) + . . .+ P(An) = 1.

Horvath Gabor Analızis 2

Page 165: Anal zis 2 - Széchenyi István Universityhorvathg/bsc/gazdmath2.pdfHa f es g gra konja, ak ar t obbsz or is, atmeszi egym ast [a;b]-n, az a-ban es b-ben all tott fugg} oleges egyenesnek,

Klasszikus veges valoszınusegalgebrak

Egy veges valoszınusegalgebraban az elemi esemenyekvaloszınusegei minden esemeny valoszınuseget meghatarozzak.Ugyanis, ha A ∈ A, akkor A ⊂ Ω, azaz A = ωi1 , ωi2 , . . . , ωik. Deekkor

A = ωi1+ ωi2+ . . .+ ωik

is igaz, es ωir ωis = ∅, ha r 6= s, tehat a fenti osszeg tagjaiegymast paronkent kizarjak, ıgy

P(A) = P(ωi1) + P(ωi2) + . . .+ P(ωik).

Mivel, ha |Ω| = n, akkorΩ = ω1, ω2, . . . , ωn = ω1+ ω2+ . . .+ ωn, esωiωj = ∅, ha i 6= j , azaz az elemi esemenyek teljesesemenyrendszert is alkotnak. A pi = P(ωi) jelolessel ezert

p1 + p2 + . . .+ pn = 1.

Horvath Gabor Analızis 2

Page 166: Anal zis 2 - Széchenyi István Universityhorvathg/bsc/gazdmath2.pdfHa f es g gra konja, ak ar t obbsz or is, atmeszi egym ast [a;b]-n, az a-ban es b-ben all tott fugg} oleges egyenesnek,

Klasszikus veges valoszınusegalgebrak

Definıcio 47

Az (Ω,A,P) veges valoszınusegalgebrat klasszikus vegesvaloszınusegalgebranak hıvjuk, ha |Ω| = n, es pi = P(ωi) = 1

n .

Ekkor minden A esemenyre

P(A) = k · 1

n=

k

n,

ahol k az A szempontjabol ”kedvezo” elemi esemenyek szama,azoke tehat, amelyek bekovetkezesekor bekovetkezik az A esemenyis, n pedig az osszes elemi esemeny szama.Ez az alpja

”a valoszınuseg = a kedvezo esetek szama osztva az osszes eset szamaval”

kijelentesnek, ami tehat, mostmar tudjuk, csak klasszikus vegesvaloszınusegalgebra eseten igaz.

Horvath Gabor Analızis 2

Page 167: Anal zis 2 - Széchenyi István Universityhorvathg/bsc/gazdmath2.pdfHa f es g gra konja, ak ar t obbsz or is, atmeszi egym ast [a;b]-n, az a-ban es b-ben all tott fugg} oleges egyenesnek,

Klasszikus veges valoszınusegalgebrak

Klasszikus veges valoszınusegalgebra eseten egy A esemenyvaloszınusegenek kiszamolasa kombinatorikus uton tortenik.Eloszor az elemi esemenyek szamat erdemes kiszamolni, azaz |Ω|-t.Ezutan az A szempontjabol kedvezo elemi esemenyek szamat. Eztgyakran hasonlo kombinatorikai meggondolasokkal lehetkiszamolni, mint |Ω|-t, altalaban egy kicsit bonyolultabb keplettel.

Azt, hogy klasszikus veges valoszınusegalgebraval van dolgunkaltalaban abbol kovetkeztetjuk ki, hogy nincs semmi okunk aztfeltetelezni, hogy az elemi esemenyek nem mind egyformanvaloszınuek.De ennel a lepesnel gondosan kell eljarni, ha ketsegek merulnek fel,akkor tovabbi meggondolasokra van szukseg. Bonyolut esetbenkısetletek elemzesevel lehet a feltetelezest elfogadni vagy elvetni.

Horvath Gabor Analızis 2

Page 168: Anal zis 2 - Széchenyi István Universityhorvathg/bsc/gazdmath2.pdfHa f es g gra konja, ak ar t obbsz or is, atmeszi egym ast [a;b]-n, az a-ban es b-ben all tott fugg} oleges egyenesnek,

Feladat 43

Egy (Ω,A) veges esemenyalgebraban P(A) = 0.6, P(B) = 0.5 esP(AB) = 0.3. Szamoljuk ki az alabbi valoszınusegeket:

1 P(A + B),

2 P(A + B).

Megoldas:

¶ Mivel az egyik de Morgan azonossag miatt A + B = AB,

P(A + B) = P(AB)

= 1− P(AB) = 0.7.

· Most A + B = AB, ıgy

P(A + B) = P(

AB)

= 1− P(AB) = 1− P(A− B) =

=︸︷︷︸T etel 30/7

1− (P(A)− P(AB)) = 1− P(A) + P(AB) = 0.7.

Horvath Gabor Analızis 2

Page 169: Anal zis 2 - Széchenyi István Universityhorvathg/bsc/gazdmath2.pdfHa f es g gra konja, ak ar t obbsz or is, atmeszi egym ast [a;b]-n, az a-ban es b-ben all tott fugg} oleges egyenesnek,

Feladat 43

Egy (Ω,A) veges esemenyalgebraban P(A) = 0.6, P(B) = 0.5 esP(AB) = 0.3. Szamoljuk ki az alabbi valoszınusegeket:

1 P(A + B),

2 P(A + B).

Megoldas:

¶ Mivel az egyik de Morgan azonossag miatt A + B = AB,

P(A + B) = P(AB)

= 1− P(AB) = 0.7.

· Most A + B = AB, ıgy

P(A + B) = P(

AB)

= 1− P(AB) = 1− P(A− B) =

=︸︷︷︸T etel 30/7

1− (P(A)− P(AB)) = 1− P(A) + P(AB) = 0.7.

Horvath Gabor Analızis 2

Page 170: Anal zis 2 - Széchenyi István Universityhorvathg/bsc/gazdmath2.pdfHa f es g gra konja, ak ar t obbsz or is, atmeszi egym ast [a;b]-n, az a-ban es b-ben all tott fugg} oleges egyenesnek,

Feladat 43

Egy (Ω,A) veges esemenyalgebraban P(A) = 0.6, P(B) = 0.5 esP(AB) = 0.3. Szamoljuk ki az alabbi valoszınusegeket:

1 P(A + B),

2 P(A + B).

Megoldas:

¶ Mivel az egyik de Morgan azonossag miatt A + B = AB,

P(A + B) = P(AB)

= 1− P(AB) = 0.7.

· Most A + B = AB, ıgy

P(A + B) = P(

AB)

= 1− P(AB) = 1− P(A− B) =

=︸︷︷︸T etel 30/7

1− (P(A)− P(AB)) = 1− P(A) + P(AB) = 0.7.

Horvath Gabor Analızis 2

Page 171: Anal zis 2 - Széchenyi István Universityhorvathg/bsc/gazdmath2.pdfHa f es g gra konja, ak ar t obbsz or is, atmeszi egym ast [a;b]-n, az a-ban es b-ben all tott fugg} oleges egyenesnek,

Feladat 44

Egy dobozban 2 piros, 3 kek es 4 zold golyo van. Kiveszunk egygolyot talalomra. Mi a valoszınusege annak, hogy kek lesz?

Megoldas: Megszamozzuk a golyokat ugy, hogy az 1 es 2 a piros,a 3, 4, 5 a kek es a 6, 7, 8, 9 a zold.Mivel barmelyik golyot kihuzhatjuk, mondhatjuk, hogyΩ = 1, 2, 3, 4, 5, 6, 7, 8, 9. Nyilvan |Ω| = 9, tehat vegesvaloszınusegalgebraval van dolgunk.Mivel semmi okunk feltetelezni, hogy barmelyik golyot nagyobbvaloszınuseggel valasztjuk, mint egy masikat, azt is feltehetjuk,hogy klasszikus veges valoszınusegalgebrarol van szo.Jelolje A azt az esemenyt, hogy a kihuzott golyo kek. Ekkor az Aszempontjabol kedvezo elemi esemenyek halmaza A = 3, 4, 5.Mivel |A| = 3,

P(A) =|A||Ω|

=3

9=

1

3.

Horvath Gabor Analızis 2

Page 172: Anal zis 2 - Széchenyi István Universityhorvathg/bsc/gazdmath2.pdfHa f es g gra konja, ak ar t obbsz or is, atmeszi egym ast [a;b]-n, az a-ban es b-ben all tott fugg} oleges egyenesnek,

Feladat 44

Egy dobozban 2 piros, 3 kek es 4 zold golyo van. Kiveszunk egygolyot talalomra. Mi a valoszınusege annak, hogy kek lesz?Megoldas: Megszamozzuk a golyokat ugy, hogy az 1 es 2 a piros,a 3, 4, 5 a kek es a 6, 7, 8, 9 a zold.Mivel barmelyik golyot kihuzhatjuk, mondhatjuk, hogyΩ = 1, 2, 3, 4, 5, 6, 7, 8, 9. Nyilvan |Ω| = 9, tehat vegesvaloszınusegalgebraval van dolgunk.Mivel semmi okunk feltetelezni, hogy barmelyik golyot nagyobbvaloszınuseggel valasztjuk, mint egy masikat, azt is feltehetjuk,hogy klasszikus veges valoszınusegalgebrarol van szo.Jelolje A azt az esemenyt, hogy a kihuzott golyo kek. Ekkor az Aszempontjabol kedvezo elemi esemenyek halmaza A = 3, 4, 5.Mivel |A| = 3,

P(A) =|A||Ω|

=3

9=

1

3.

Horvath Gabor Analızis 2

Page 173: Anal zis 2 - Széchenyi István Universityhorvathg/bsc/gazdmath2.pdfHa f es g gra konja, ak ar t obbsz or is, atmeszi egym ast [a;b]-n, az a-ban es b-ben all tott fugg} oleges egyenesnek,

Feladat 45

Egy dobozban 2 piros, 3 kek es 4 zold golyo van. Kiveszunk egyszerre ketgolyot. Mi a valoszınusege annak, hogy lesz a golyok kozott kek?

Megoldas: Most a kıserlet egy lehetseges kimenetele azonosıthato az1, 2, 3, 4, 5, 6, 7, 8, 9 halmaz egy ket elemu reszhalmazaval, hiszen akivett golyoknak nincs sorrendje, es ismetlodes nem fordulhat elo. Ezert|Ω| = C 2

9 =(

92

)= 36. Mivel ez mar eleg nagy szam, magat az Ω halmazt

nem is ırjuk fel, valoszınusegek kiszamolasakor ugyis csak az elemszamaralesz szuksegunk.Mivel semmi okunk mast feltetelezni, most is elfogadjuk, hogy klasszikusveges valoszınusegalgebrarol van szo.Jelolje B azt az esemenyt, hogy a kivett golyok kozott van kek. Ekkor Bazt jelenti, hogy a kivett golyok kozott nincs kek, azaz a kivett golyok a1, 2, 6, 7, 8, 9 halmaz egy ketelemu reszhalmazaval azonosıthatok.Ilyenbol C 2

6 =(

62

)van, tehat |B| =

(62

)= 15. Igy

P(B) =|B||Ω|

=15

36=

5

12.

Ebbol P(B) = 1− P(B) = 1− 512 = 7

12 .

Horvath Gabor Analızis 2

Page 174: Anal zis 2 - Széchenyi István Universityhorvathg/bsc/gazdmath2.pdfHa f es g gra konja, ak ar t obbsz or is, atmeszi egym ast [a;b]-n, az a-ban es b-ben all tott fugg} oleges egyenesnek,

Feladat 45

Egy dobozban 2 piros, 3 kek es 4 zold golyo van. Kiveszunk egyszerre ketgolyot. Mi a valoszınusege annak, hogy lesz a golyok kozott kek?Megoldas: Most a kıserlet egy lehetseges kimenetele azonosıthato az1, 2, 3, 4, 5, 6, 7, 8, 9 halmaz egy ket elemu reszhalmazaval, hiszen akivett golyoknak nincs sorrendje, es ismetlodes nem fordulhat elo. Ezert|Ω| = C 2

9 =(

92

)= 36. Mivel ez mar eleg nagy szam, magat az Ω halmazt

nem is ırjuk fel, valoszınusegek kiszamolasakor ugyis csak az elemszamaralesz szuksegunk.Mivel semmi okunk mast feltetelezni, most is elfogadjuk, hogy klasszikusveges valoszınusegalgebrarol van szo.Jelolje B azt az esemenyt, hogy a kivett golyok kozott van kek. Ekkor Bazt jelenti, hogy a kivett golyok kozott nincs kek, azaz a kivett golyok a1, 2, 6, 7, 8, 9 halmaz egy ketelemu reszhalmazaval azonosıthatok.Ilyenbol C 2

6 =(

62

)van, tehat |B| =

(62

)= 15. Igy

P(B) =|B||Ω|

=15

36=

5

12.

Ebbol P(B) = 1− P(B) = 1− 512 = 7

12 .

Horvath Gabor Analızis 2

Page 175: Anal zis 2 - Széchenyi István Universityhorvathg/bsc/gazdmath2.pdfHa f es g gra konja, ak ar t obbsz or is, atmeszi egym ast [a;b]-n, az a-ban es b-ben all tott fugg} oleges egyenesnek,

Feladat 46

Egy dobozban 2 piros, 3 kek es 4 zold golyo van. Kiveszunk egymas utan ket golyot. Mi a valoszınusege annak,hogy lesz a golyok kozott kek?

Megoldas: Ebben az esetben a kıserlet egy lehetseges kimenetele azonosıthato az 1, 2, 3, 4, 5, 6, 7, 8, 9 halmazegy ket elemu ismetles nelkuli variaciojaval, hiszen a kivett golyoknak most van sorrendje, de ismetlodes nemfordulhat elo. Ezert |Ω| = V 2

9 = 9 · 8 = 72. Ismet feltetelezhetjuk, hogy klasszikus veges valoszınusegalgebrarolvan szo.Jelolje C azt az esemenyt, hogy a kivett golyok kozott van kek. Ekkor harom eset lehetseges:

az elso golyo kek, de a masodik nem, ezt az esemenyt jelolje C1,

a masodik golyo kek, de az elso nem, ezt az esemenyt jelolje C2,

mindket golyo kek, ezt az esemenyt jelolje C3.

Ekkor C = C1 + C2 + C3 es a C1, C2, C3 esemenyek egymast paronkent kizarjak. Ezert

P(C) = P(C1 + C2 + C3) = P(C1) + P(C2) + P(C3).

Meghatarozzuk a C1 esemeny szempontjabol kedvezo elemi esemenyek szamat. C1 bekovetkezese eseten az elsohelyre a harom kek golyo kozul harom felekeppen huzhatunk egy golyot, a masodik helyre a hat nem kek golyokuzul hat felekeppen, tehat |C1| = 3 · 6 = 18. Tehat P(C1) = 18

72.

Hasonloan |C2| = 6 · 3 = 18, azaz P(C2) = 1872

.

|C3| nyilvan 3 · 2 = 6, vagyis P(C3) = 672

.Ezeket felhasznalva

P(C) = P(C1 + C2 + C3) = P(C1) + P(C2) + P(C3) =18

72+

18

72+

6

72=

42

72=

7

12.

Figyeljuk meg, hogy az eredmeny ugyanaz, mint az elozo feladatban.

Horvath Gabor Analızis 2

Page 176: Anal zis 2 - Széchenyi István Universityhorvathg/bsc/gazdmath2.pdfHa f es g gra konja, ak ar t obbsz or is, atmeszi egym ast [a;b]-n, az a-ban es b-ben all tott fugg} oleges egyenesnek,

Feladat 46

Egy dobozban 2 piros, 3 kek es 4 zold golyo van. Kiveszunk egymas utan ket golyot. Mi a valoszınusege annak,hogy lesz a golyok kozott kek?Megoldas: Ebben az esetben a kıserlet egy lehetseges kimenetele azonosıthato az 1, 2, 3, 4, 5, 6, 7, 8, 9 halmazegy ket elemu ismetles nelkuli variaciojaval, hiszen a kivett golyoknak most van sorrendje, de ismetlodes nemfordulhat elo. Ezert |Ω| = V 2

9 = 9 · 8 = 72. Ismet feltetelezhetjuk, hogy klasszikus veges valoszınusegalgebrarolvan szo.Jelolje C azt az esemenyt, hogy a kivett golyok kozott van kek. Ekkor harom eset lehetseges:

az elso golyo kek, de a masodik nem, ezt az esemenyt jelolje C1,

a masodik golyo kek, de az elso nem, ezt az esemenyt jelolje C2,

mindket golyo kek, ezt az esemenyt jelolje C3.

Ekkor C = C1 + C2 + C3 es a C1, C2, C3 esemenyek egymast paronkent kizarjak. Ezert

P(C) = P(C1 + C2 + C3) = P(C1) + P(C2) + P(C3).

Meghatarozzuk a C1 esemeny szempontjabol kedvezo elemi esemenyek szamat. C1 bekovetkezese eseten az elsohelyre a harom kek golyo kozul harom felekeppen huzhatunk egy golyot, a masodik helyre a hat nem kek golyokuzul hat felekeppen, tehat |C1| = 3 · 6 = 18. Tehat P(C1) = 18

72.

Hasonloan |C2| = 6 · 3 = 18, azaz P(C2) = 1872

.

|C3| nyilvan 3 · 2 = 6, vagyis P(C3) = 672

.Ezeket felhasznalva

P(C) = P(C1 + C2 + C3) = P(C1) + P(C2) + P(C3) =18

72+

18

72+

6

72=

42

72=

7

12.

Figyeljuk meg, hogy az eredmeny ugyanaz, mint az elozo feladatban.

Horvath Gabor Analızis 2

Page 177: Anal zis 2 - Széchenyi István Universityhorvathg/bsc/gazdmath2.pdfHa f es g gra konja, ak ar t obbsz or is, atmeszi egym ast [a;b]-n, az a-ban es b-ben all tott fugg} oleges egyenesnek,

Feladat 47

Egy dobozban 2 piros, 3 kek es 4 zold golyo van. Kiveszunkegymas utan harom golyot ugy, hogy a kivett golyot mindigvisszatesszuk. Mi a valoszınusege annak, hogy az elso es utolsogolyo kek lesz?

Megoldas: Ennel a kıserletnel egy lehetseges kimenetelazonosıthato az 1, 2, 3, 4, 5, 6, 7, 8, 9 halmaz egy harom elemuismetleses variaciojaval, hiszen a kivett golyoknak most vansorrendje, es ismetlodes is elofordulhat. Ezert|Ω| = V 3,i

9 = 93 = 729. Ismet feltetelezhetjuk, hogy klasszikusveges valoszınusegalgebrarol van szo.Jelolje D azt az esemenyt, hogy az elso es utolso golyo kek lesz.Ekkor az elso helyre kek golyot harom felekeppen, a masodikhelyre barmilyen golyot kilenc felekeppen, a harmadik helyre, avisszateves miatt, kek golyot ismet harom felekeppen huzhatunk.Tehat |D| = 3 · 9 · 3 = 81. Azaz

P(D) =81

729=

1

9.

Horvath Gabor Analızis 2

Page 178: Anal zis 2 - Széchenyi István Universityhorvathg/bsc/gazdmath2.pdfHa f es g gra konja, ak ar t obbsz or is, atmeszi egym ast [a;b]-n, az a-ban es b-ben all tott fugg} oleges egyenesnek,

Feladat 47

Egy dobozban 2 piros, 3 kek es 4 zold golyo van. Kiveszunkegymas utan harom golyot ugy, hogy a kivett golyot mindigvisszatesszuk. Mi a valoszınusege annak, hogy az elso es utolsogolyo kek lesz?Megoldas: Ennel a kıserletnel egy lehetseges kimenetelazonosıthato az 1, 2, 3, 4, 5, 6, 7, 8, 9 halmaz egy harom elemuismetleses variaciojaval, hiszen a kivett golyoknak most vansorrendje, es ismetlodes is elofordulhat. Ezert|Ω| = V 3,i

9 = 93 = 729. Ismet feltetelezhetjuk, hogy klasszikusveges valoszınusegalgebrarol van szo.Jelolje D azt az esemenyt, hogy az elso es utolso golyo kek lesz.Ekkor az elso helyre kek golyot harom felekeppen, a masodikhelyre barmilyen golyot kilenc felekeppen, a harmadik helyre, avisszateves miatt, kek golyot ismet harom felekeppen huzhatunk.Tehat |D| = 3 · 9 · 3 = 81. Azaz

P(D) =81

729=

1

9.

Horvath Gabor Analızis 2

Page 179: Anal zis 2 - Széchenyi István Universityhorvathg/bsc/gazdmath2.pdfHa f es g gra konja, ak ar t obbsz or is, atmeszi egym ast [a;b]-n, az a-ban es b-ben all tott fugg} oleges egyenesnek,

Feladat 48

Egy piros es egy kek kockat feldobunk egyszerre. Mi a valoszınusege annak, hogy a dobott szamok kozott leszegyes vagy lesz hatos?

Megoldas: A kıserletunk egy kimenetelet ugy adhatjuk meg, hogy eloszor leırjuk a piros kockaval dobott szamot,azutan a kek kockaval dobottat. (Fordıtva persze ugyanilyen jo lenne.) Tehat, mivel a sorrend szamıt, es

ismetlodes is lehet, Ω egy eleme azonosıthato hat elem egy masod osztalyu ismetleses variaciojaval. Igy

|Ω| = V2,i6 = 62 = 36.

Jelolje A azt az esemenyt, hogy a dobott szamok kozott lesz egyes, B azt az esemenyt, hogy lesz hatos. Ekkor azA + B esemeny valoszınuseget kell kiszamolnunk. Mivel A es B nem egymast kizaro esemenyek, hiszen dobhatunkelsore egyest, a masodikra hatost vagy fordıtva, a

P(A + B) = P(A) + P(B)− P(AB)

formulat kell hasznalnunk.Az A esemeny haromfelekeppen kovetkezhet be: ha elsore egyest dobunk, de masodikra nem, erre 5 lehetoseg van;masodikra egyest dobunk, de elsore nem, erre is 5 lehetosegunk van, illetve, ha elsore es masodikra is egyestdobunk, ami egyfelekeppen fordulhat elo. Mivel ezek az esemenyek egyszerre nem kovetkezhetnek be|A| = 5 + 5 + 1 = 11.Teljesen hasonloan |B| = 11.AB pedig ket esetben fordul elo, hiszen A = 16, 61. Ezek alapjan

P(A + B) = P(A) + P(B)− P(AB) =11

36+

11

36−

2

36=

20

36=

5

9.

Horvath Gabor Analızis 2

Page 180: Anal zis 2 - Széchenyi István Universityhorvathg/bsc/gazdmath2.pdfHa f es g gra konja, ak ar t obbsz or is, atmeszi egym ast [a;b]-n, az a-ban es b-ben all tott fugg} oleges egyenesnek,

Feladat 48

Egy piros es egy kek kockat feldobunk egyszerre. Mi a valoszınusege annak, hogy a dobott szamok kozott leszegyes vagy lesz hatos?Megoldas: A kıserletunk egy kimenetelet ugy adhatjuk meg, hogy eloszor leırjuk a piros kockaval dobott szamot,azutan a kek kockaval dobottat. (Fordıtva persze ugyanilyen jo lenne.) Tehat, mivel a sorrend szamıt, es

ismetlodes is lehet, Ω egy eleme azonosıthato hat elem egy masod osztalyu ismetleses variaciojaval. Igy

|Ω| = V2,i6 = 62 = 36.

Jelolje A azt az esemenyt, hogy a dobott szamok kozott lesz egyes, B azt az esemenyt, hogy lesz hatos. Ekkor azA + B esemeny valoszınuseget kell kiszamolnunk. Mivel A es B nem egymast kizaro esemenyek, hiszen dobhatunkelsore egyest, a masodikra hatost vagy fordıtva, a

P(A + B) = P(A) + P(B)− P(AB)

formulat kell hasznalnunk.Az A esemeny haromfelekeppen kovetkezhet be: ha elsore egyest dobunk, de masodikra nem, erre 5 lehetoseg van;masodikra egyest dobunk, de elsore nem, erre is 5 lehetosegunk van, illetve, ha elsore es masodikra is egyestdobunk, ami egyfelekeppen fordulhat elo. Mivel ezek az esemenyek egyszerre nem kovetkezhetnek be|A| = 5 + 5 + 1 = 11.Teljesen hasonloan |B| = 11.AB pedig ket esetben fordul elo, hiszen A = 16, 61. Ezek alapjan

P(A + B) = P(A) + P(B)− P(AB) =11

36+

11

36−

2

36=

20

36=

5

9.

Horvath Gabor Analızis 2

Page 181: Anal zis 2 - Széchenyi István Universityhorvathg/bsc/gazdmath2.pdfHa f es g gra konja, ak ar t obbsz or is, atmeszi egym ast [a;b]-n, az a-ban es b-ben all tott fugg} oleges egyenesnek,

Geometriai valoszınusegek

Tekintsuk a kovetkezo kıserletet: talalomra ralovunk egy kor alakuceltablara; tegyuk fel, hogy egyaltalan nem tudunk loni vagy csukottszemmel lovunk, de minden loves eltalalja a celtablat. Megkerdezhetjuk,hogy mi a valoszınusege annak, hogy a celtabla felso felet talaljuk el?Erezzuk, hogy most is egy valoszınusegszamıtasi problemaval vandolgunk, de ez a veges valoszınusegalgebra segıtsegevel nemmodellezheto, hiszen a kıserletnek vegtelen sok kimenetele van.Tehat egy uj modellre van szuksegunk.

Definıcio 48

Legyen Ω egy sıkbeli halmaz, azaz Ω ⊂ R2. Tegyuk fel, hogy a kıserletminden kimenetele Ω egy pontjanak kivalasztasat jelenti. Legyen A ⊂ Ω,es definialjuk annak a valoszınuseget, hogy a kıserlet kimenetele az Ahalmazba esik ıgy:

P(A) =t(A)

t(Ω),

ahol t(A) az A, t(Ω) az Ω halmaz terulete.

Horvath Gabor Analızis 2

Page 182: Anal zis 2 - Széchenyi István Universityhorvathg/bsc/gazdmath2.pdfHa f es g gra konja, ak ar t obbsz or is, atmeszi egym ast [a;b]-n, az a-ban es b-ben all tott fugg} oleges egyenesnek,

Geometriai valoszınusegek

Ez a modell joval elnagyoltabb, mint a veges valoszınusegalgebramodellje. Ennek az az oka, hogy, mint tudjuk, a sık nem mindenreszhalmazanak van terulete. Egyszeru Ω es A eseten azonban jolhasznalhato.

Definıcio 49

Legyen Ω egy egyenes reszhalmaza, azaz Ω ⊂ R. Tegyuk fel, hogy akıserlet minden kimenetele Ω egy pontjanak kivalasztasat jelenti. LegyenA ⊂ Ω, es definialjuk annak a valoszınuseget, hogy a kıserlet kimeneteleaz A halmazba esik ıgy:

P(A) =l(A)

l(Ω),

ahol l(A) az A, l(Ω) az Ω halmaz hossza.

Ezzel kapcsolatban ugyanaz igaz, mint a teruletre: egy egyenes nemminden reszhalmazanak van hossza, de az egyszeru halmazoknak igen.

Ha Ω terbeli halmaz lenne, hasonlo definıcioval elhetnenk, de ezzel nem

fogunk foglalkozni.

Horvath Gabor Analızis 2

Page 183: Anal zis 2 - Széchenyi István Universityhorvathg/bsc/gazdmath2.pdfHa f es g gra konja, ak ar t obbsz or is, atmeszi egym ast [a;b]-n, az a-ban es b-ben all tott fugg} oleges egyenesnek,

Feladat 49

A busz, amivel az egyetemre jovunk, reggel 7 es 8 nyolc ora kozott 15percenkent jar. Ha 7 es 8 nyolc ora kozott talalomra kimegyunk abuszmegalloba, mennyi lesz annak a valoszınusege, hogy nem kell 5percnel tobbet varnunk?

Megoldas: A buszmegalloba valo megerkezesunk idopontja azonosıthatoegy szakasz, megpedig, ha az idot percekben merjuk, egy 60 egyseghosszu szakasz egy pontjanak a kivalasztasaval. Tehat egy egydimenziosgeometria valoszınuseges feladatrol van szo.

Az abran feltuntettuk az Ω-t reprezentalo 60 egyseg hosszu szakaszt.Jeloljuk A-val azt az esemenyt, hogy nem kell 5 percnel tobbet varnunk.Az abran vastagon megjeloltuk az A szempontjabol kedvezo erkezesiidopontokat. Ezek alapjan

P(A) =l(A)

l(Ω)=

20

60=

1

3.

Horvath Gabor Analızis 2

Page 184: Anal zis 2 - Széchenyi István Universityhorvathg/bsc/gazdmath2.pdfHa f es g gra konja, ak ar t obbsz or is, atmeszi egym ast [a;b]-n, az a-ban es b-ben all tott fugg} oleges egyenesnek,

Feladat 49

A busz, amivel az egyetemre jovunk, reggel 7 es 8 nyolc ora kozott 15percenkent jar. Ha 7 es 8 nyolc ora kozott talalomra kimegyunk abuszmegalloba, mennyi lesz annak a valoszınusege, hogy nem kell 5percnel tobbet varnunk?Megoldas: A buszmegalloba valo megerkezesunk idopontja azonosıthatoegy szakasz, megpedig, ha az idot percekben merjuk, egy 60 egyseghosszu szakasz egy pontjanak a kivalasztasaval. Tehat egy egydimenziosgeometria valoszınuseges feladatrol van szo.

Az abran feltuntettuk az Ω-t reprezentalo 60 egyseg hosszu szakaszt.Jeloljuk A-val azt az esemenyt, hogy nem kell 5 percnel tobbet varnunk.Az abran vastagon megjeloltuk az A szempontjabol kedvezo erkezesiidopontokat. Ezek alapjan

P(A) =l(A)

l(Ω)=

20

60=

1

3.

Horvath Gabor Analızis 2

Page 185: Anal zis 2 - Széchenyi István Universityhorvathg/bsc/gazdmath2.pdfHa f es g gra konja, ak ar t obbsz or is, atmeszi egym ast [a;b]-n, az a-ban es b-ben all tott fugg} oleges egyenesnek,

Feladat 50

Delutan harom orara talalkozot beszeltunk meg az ismerosunkel. Mi barmikor erkezhetunk 2 : 55 es 3 : 05 kozott,de az ismerosunk hajlamos a kesesre, 3 : 00 es 3 : 10 kozott erkezik veletlenszeruen. Mi a valoszınusege annak,hogy egyikunk sem var ket percnel tobbet?

Megoldas: Merjuk ismet percekben az idot. Az, hogy a talalkozo harom orara volt megbeszelve, nyilvan nemlenyeges. Az a lenyeg, hogy mi egy 10 perc hosszu idointervallumban valamikor, az ismerosunk egy ehhez kepest 5percel eltolt, szinten 10 hosszu idointervallumban erkezik valamikor. A bal oldali abran ezt tuntettuk fel, x-el jelolvea mi, y-al az ismerosunk egy lehetseges erkezeset.

A kıserlet egy kimenetele tehat az (x, y) rendezett szampar megadasaval adhato meg, ahol 0 ≤ x ≤ 10,5 ≤ y ≤ 15. Ezek a rendezett parok azonosıthatok a kozepso abran lathato negyzet pontjaival, ebben afeladatban ez lesz az Ω. Most tehat egy ket dimenzios geometria valoszınuseges feladattal van dolgunk, avaloszınusegeket teruletek aranyaval tudjuk kiszamolni.Jelolje A azt az esemenyt, hogy egyikunk sem var ket percnel tobbet. Meghatarozzuk az A szempontjabol kedvezopontok halmazat. A bevezetett jelolesekkel A akkor kovetkezik be, ha |x − y| ≤ 2, azaz −2 ≤ x − y ≤ 2. Azelso egyenlotlensegnek a felso ferde egyenes alatti pontok tesznek eleget, a masodiknak az also feletti pontok,

mindkettonek a pontozott trapez pontjai. Ennek terulete 72

2− 32

2= 40

2. Igy

P(A) =t(A)

t(Ω)=

402

100=

1

5.

Horvath Gabor Analızis 2

Page 186: Anal zis 2 - Széchenyi István Universityhorvathg/bsc/gazdmath2.pdfHa f es g gra konja, ak ar t obbsz or is, atmeszi egym ast [a;b]-n, az a-ban es b-ben all tott fugg} oleges egyenesnek,

Feladat 50

Delutan harom orara talalkozot beszeltunk meg az ismerosunkel. Mi barmikor erkezhetunk 2 : 55 es 3 : 05 kozott,de az ismerosunk hajlamos a kesesre, 3 : 00 es 3 : 10 kozott erkezik veletlenszeruen. Mi a valoszınusege annak,hogy egyikunk sem var ket percnel tobbet?Megoldas: Merjuk ismet percekben az idot. Az, hogy a talalkozo harom orara volt megbeszelve, nyilvan nemlenyeges. Az a lenyeg, hogy mi egy 10 perc hosszu idointervallumban valamikor, az ismerosunk egy ehhez kepest 5percel eltolt, szinten 10 hosszu idointervallumban erkezik valamikor. A bal oldali abran ezt tuntettuk fel, x-el jelolvea mi, y-al az ismerosunk egy lehetseges erkezeset.

A kıserlet egy kimenetele tehat az (x, y) rendezett szampar megadasaval adhato meg, ahol 0 ≤ x ≤ 10,5 ≤ y ≤ 15. Ezek a rendezett parok azonosıthatok a kozepso abran lathato negyzet pontjaival, ebben afeladatban ez lesz az Ω. Most tehat egy ket dimenzios geometria valoszınuseges feladattal van dolgunk, avaloszınusegeket teruletek aranyaval tudjuk kiszamolni.Jelolje A azt az esemenyt, hogy egyikunk sem var ket percnel tobbet. Meghatarozzuk az A szempontjabol kedvezopontok halmazat. A bevezetett jelolesekkel A akkor kovetkezik be, ha |x − y| ≤ 2, azaz −2 ≤ x − y ≤ 2. Azelso egyenlotlensegnek a felso ferde egyenes alatti pontok tesznek eleget, a masodiknak az also feletti pontok,

mindkettonek a pontozott trapez pontjai. Ennek terulete 72

2− 32

2= 40

2. Igy

P(A) =t(A)

t(Ω)=

402

100=

1

5.

Horvath Gabor Analızis 2

Page 187: Anal zis 2 - Széchenyi István Universityhorvathg/bsc/gazdmath2.pdfHa f es g gra konja, ak ar t obbsz or is, atmeszi egym ast [a;b]-n, az a-ban es b-ben all tott fugg} oleges egyenesnek,

Kolmogorov fele valoszınusegi mezok

A veges valoszınusegalgebra modellje nem eleg altalanos, azA = P(Ω) feltetellel van a baj, azt kell gyengıteni.

Definıcio 50

Legyen Ω egy tetszoleges halmaz, A ⊂ P(Ω). A σ-algebra, ha

1 Ω ∈ A,

2 ha A ∈ A, akkor A ∈ A,

3 ha A1, A2,. . . veges vagy megszamlalhatoan vegtelen sokesemeny, akkor

⋃i

Ai ∈ A.

Belathato, hogy a fenti halmazok metszete is A-ban van, ıgy aσ-algebra lenyege, hogy abban a legfontosabb halmazmuveletekkorlatlanul elvegezhetok.

Horvath Gabor Analızis 2

Page 188: Anal zis 2 - Széchenyi István Universityhorvathg/bsc/gazdmath2.pdfHa f es g gra konja, ak ar t obbsz or is, atmeszi egym ast [a;b]-n, az a-ban es b-ben all tott fugg} oleges egyenesnek,

Kolmogorov fele valoszınusegi mezok

A valoszınusegszamıtas Kolmogorov fele axiomai

(Ω,A, P) Kolmogorov fele valoszınusegi mezo, ha

1 Ω egy halmaz, Ω elemeit elemi esemenyeknek hıvjuk,

2 A ⊂ P(Ω) σ-algebra, A elemeit esemenyeknek hıvjuk,

3 A P : A −→ R olyan fuggveny, hogy

1 P(A) ≥ 0 minden A ∈ A-ra,2 P(Ω) = 1,3 ha A1, A2,. . . veges vagy megszamlalhatoan vegtelen sok

esemeny, akkor

P

(⋃i

Ai

)=∑

i

P(Ai ).

Ezzel a modellel rengetek valoszınusegszamıtasi problema vizsgalhato, eddig meg nem merult fel komolyan, hogyszukseg lenne a tovabbi altalanosıtasara.Fontos megjegyezni, hogy minden veges valoszınusegalgebra egyben Kolmogorov fele valoszınusegi mezo is, es azeddigi tetelek valtoztatas nelkul ervenyben maradnak.Ha Ω egy olyan sıkbeli vagy egy egyenesben fekvo halmaz, aminek van terulete illetve hossza, akkor az osszesterulettel illetve hosszusaggal rendelkezo reszhalmaza σ-algebrat alkot. Ez a modell tehat a geometriavaloszınuseges feladatoknal is alkalmazhato.

Horvath Gabor Analızis 2

Page 189: Anal zis 2 - Széchenyi István Universityhorvathg/bsc/gazdmath2.pdfHa f es g gra konja, ak ar t obbsz or is, atmeszi egym ast [a;b]-n, az a-ban es b-ben all tott fugg} oleges egyenesnek,

Felteteles valoszınusegek

Adot egy (Ω,A,P) Kolmogorov fele valoszınusegi mezo, A,B ∈ A.Vegezzuk el a kıserletet N-szer.

Definıcio 51

Tegyuk fel, hogy az N kıserlet soran n-szer bekovetkezett a B esemeny,es ekozul az n kıserlet kozul k-ban a B-vel egyutt az A esemeny isbekovetkezett. Ekkor a k

n szamot az A esemeny B feltetel mellettifelteteles relatıv gyakorisaganak hıvjuk.

Mivelk

n=

kNnN

,

es itt a szamlaloban az AB esemeny relatıv gyakorisaga all, ami P(AB) korul ingadozik, a nevezoben a B esemeny

relatıv gyakorisaga, ami P(B) korul ingadozik, indokolt a kovetkezo definıcio.

Definıcio 52

A,B ∈ A, P(B) > 0. Az A esemeny B feltetel melletti feltetelesvaloszınusege

P(A|B) =P(AB)

P(B).

Horvath Gabor Analızis 2

Page 190: Anal zis 2 - Széchenyi István Universityhorvathg/bsc/gazdmath2.pdfHa f es g gra konja, ak ar t obbsz or is, atmeszi egym ast [a;b]-n, az a-ban es b-ben all tott fugg} oleges egyenesnek,

Feladat 51

Feldobunk egy piros es egy feher kockat egyszerre. Mi avaloszınusege annak, hogy ket hatost dobunk, felteve, hogy a feherkockaval hatost dobtunk?

Megoldas: Jelolje A azt az esemenyt, hogy ket hatost dobunk, Bazt, hogy a feher kockaval hatost dobunk. Ki akarjuk szamolniP(A|B)-t.Most Ω azonosıthato 6 elem osszes masodosztalyu ismetlesesvariaciojaval, ezert |Ω| = 36, es persze olyan Kolmogorov felevaloszınusegi mezorol van szo, ami egyben klasszikus vegesvaloszınusegalgebra is. Mivel

P(AB) =1

36, P(B) =

6

36,

ezert

P(A|B) =P(AB)

P(B)=

1366

36

=1

6.

Horvath Gabor Analızis 2

Page 191: Anal zis 2 - Széchenyi István Universityhorvathg/bsc/gazdmath2.pdfHa f es g gra konja, ak ar t obbsz or is, atmeszi egym ast [a;b]-n, az a-ban es b-ben all tott fugg} oleges egyenesnek,

Feladat 51

Feldobunk egy piros es egy feher kockat egyszerre. Mi avaloszınusege annak, hogy ket hatost dobunk, felteve, hogy a feherkockaval hatost dobtunk?Megoldas: Jelolje A azt az esemenyt, hogy ket hatost dobunk, Bazt, hogy a feher kockaval hatost dobunk. Ki akarjuk szamolniP(A|B)-t.Most Ω azonosıthato 6 elem osszes masodosztalyu ismetlesesvariaciojaval, ezert |Ω| = 36, es persze olyan Kolmogorov felevaloszınusegi mezorol van szo, ami egyben klasszikus vegesvaloszınusegalgebra is. Mivel

P(AB) =1

36, P(B) =

6

36,

ezert

P(A|B) =P(AB)

P(B)=

1366

36

=1

6.

Horvath Gabor Analızis 2

Page 192: Anal zis 2 - Széchenyi István Universityhorvathg/bsc/gazdmath2.pdfHa f es g gra konja, ak ar t obbsz or is, atmeszi egym ast [a;b]-n, az a-ban es b-ben all tott fugg} oleges egyenesnek,

Tetel 33

1 P(A|B) · P(B) = P(B|A) · P(A) = P(AB),

2 P(A|B) = 1− P(A|B),

3 ha AB = ∅, akkor P(A + B|C ) = P(A|C ) + P(B|C ),

4 P(Ω|B) = 1, P(∅|B) = 0.

Tetel 34 (A valoszınusegek szorzasi tetele)

Ha P(A1A2 . . .An) > 0, akkor

P(A1A2 . . .An) =

= P(A1) · P(A2|A1) · P(A3|A1A2) · . . . · P(An|A1A2 . . .An−1).

Horvath Gabor Analızis 2

Page 193: Anal zis 2 - Széchenyi István Universityhorvathg/bsc/gazdmath2.pdfHa f es g gra konja, ak ar t obbsz or is, atmeszi egym ast [a;b]-n, az a-ban es b-ben all tott fugg} oleges egyenesnek,

Feladat 52

Tegyuk fel, hogy P(A|B) = 0.7, P(A|B) = 0.3 es P(B|A) = 0.6. Mivel egyenlo P(A)?

Megoldas: Felırjuk a megadott felteteles valoszınusegeket definıciojuk szerint.

P(A|B) =P(AB)P(B)

= 0.7, azaz P(AB) = 0.7P(B),

P(A|B) =P(AB)

P(B)= 0.3, azaz P(AB) = 0.3P(B) = 0.3(1− P(B)),

P(B|A) =P(BA)P(A)

= 0.6, azaz P(AB) = 0.6P(A)

A sorok jobb szelen van harom egyenletunk, amiben azonban negy ismeretlen van.De tudjuk, hogy AB + AB = A(B + B) = A, es AB valamint AB egymast kizaro esemenyek. Ezert

P(AB) + P(AB) = P(A). Igy, ha osszeadjuk a felso es a kozepso egyenletet, illetve az also es a kozepsoegyenletet, akkor azt kapjuk, hogy

P(A) = 0.3 + 0.4P(B),

P(A) = 0.6P(A)− 0.3P(B) + 0.3.

Ez ket linearis egyenlet ket ismeretlenre, amik kozott ott van a P(A) is, amit keresunk. Megoldva az

egyenletrendszert P(A) = 2146

, P(B) = 1846

.

Horvath Gabor Analızis 2

Page 194: Anal zis 2 - Széchenyi István Universityhorvathg/bsc/gazdmath2.pdfHa f es g gra konja, ak ar t obbsz or is, atmeszi egym ast [a;b]-n, az a-ban es b-ben all tott fugg} oleges egyenesnek,

Feladat 52

Tegyuk fel, hogy P(A|B) = 0.7, P(A|B) = 0.3 es P(B|A) = 0.6. Mivel egyenlo P(A)?Megoldas: Felırjuk a megadott felteteles valoszınusegeket definıciojuk szerint.

P(A|B) =P(AB)P(B)

= 0.7, azaz P(AB) = 0.7P(B),

P(A|B) =P(AB)

P(B)= 0.3, azaz P(AB) = 0.3P(B) = 0.3(1− P(B)),

P(B|A) =P(BA)P(A)

= 0.6, azaz P(AB) = 0.6P(A)

A sorok jobb szelen van harom egyenletunk, amiben azonban negy ismeretlen van.De tudjuk, hogy AB + AB = A(B + B) = A, es AB valamint AB egymast kizaro esemenyek. Ezert

P(AB) + P(AB) = P(A). Igy, ha osszeadjuk a felso es a kozepso egyenletet, illetve az also es a kozepsoegyenletet, akkor azt kapjuk, hogy

P(A) = 0.3 + 0.4P(B),

P(A) = 0.6P(A)− 0.3P(B) + 0.3.

Ez ket linearis egyenlet ket ismeretlenre, amik kozott ott van a P(A) is, amit keresunk. Megoldva az

egyenletrendszert P(A) = 2146

, P(B) = 1846

.

Horvath Gabor Analızis 2

Page 195: Anal zis 2 - Széchenyi István Universityhorvathg/bsc/gazdmath2.pdfHa f es g gra konja, ak ar t obbsz or is, atmeszi egym ast [a;b]-n, az a-ban es b-ben all tott fugg} oleges egyenesnek,

Felteteles valoszınusegek

Tetel 35 (A teljes valoszınuseg tetele)

Ha B1,B2, . . . ,Bn teljes esemenyrendszer, es minden i-reP(Bi ) > 0, akkor tetszoleges A esemenyre

P(A) = P(A|B1) ·P(B1) + P(A|B2) ·P(B2) + . . .+ P(A|Bn) ·P(Bn).

Tetel 36 (Bayes tetele)

Ha B1,B2, . . . ,Bn teljes esemenyrendszer, P(A) > 0, es mindeni-re P(Bi ) > 0, akkor

P(Bk |A) =

=P(A|Bk ) · P(Bk )

P(A|B1) · P(B1) + P(A|B2) · P(B2) + . . .+ P(A|Bn) · P(Bn).

Horvath Gabor Analızis 2

Page 196: Anal zis 2 - Széchenyi István Universityhorvathg/bsc/gazdmath2.pdfHa f es g gra konja, ak ar t obbsz or is, atmeszi egym ast [a;b]-n, az a-ban es b-ben all tott fugg} oleges egyenesnek,

Feladat 53

Van ket dobozunk. Az elsoben 4 piros es 4 feher golyo van, a masodikban 6 piros es 3 feher. Talalomrakivalasztunk egy dobozt, es abbol kiveszunk egy golyot. A dobozok kozul azonos valoszınuseggel valasztunk. Mi avaloszınusege annak, hogy a kivett golyo piros?

Megoldas: Jelolje A azt az esemenyt, hogy a kivett golyo piros. Ha biztosan tudnank, hogy az elso dobozbolhuzunk, ennek valoszınusegere 1/2 lenne a valasz, ha azt tudnank, hogy a masodik dobozbol huzunk, 6/9 lenne avalasz. De nem tudjuk elore, hogy melyik dobozbol huzunk.A feladatban azonban szo van egy teljes esemenyrendszerrol is: jelolje B1 azt az esemenyt, hogy az elso dobozbolhuzunk, B2 azt, hogy a masodikbol. B1 es B2 nyilvan teljes esemenyrendszert alkot, es a feladat szovegebolP(B1) = P(B2) = 1/2. Ezert a teljes valoszınuseg tetele alapjan

P(A) = P(A|B1) · P(B1) + P(A|B2) · P(B2).

Itt P(A|B1) azt jelenti, hogy mi a valoszınusege annak, hogy a kivett golyo piros, felteve, hogy az elso dobozbolhuztuk. Ez persze 1/2. P(A|B2) azt jelenti, hogy mi a valoszınusege annak, hogy a kivett golyo piros, felteve,hogy a masodik dobozbol huztuk. Ez meg 6/9. Ezek alapjan tehat

P(A) = P(A|B1) · P(B1) + P(A|B2) · P(B2) =1

1

2+

6

1

2=

7

12.

Horvath Gabor Analızis 2

Page 197: Anal zis 2 - Széchenyi István Universityhorvathg/bsc/gazdmath2.pdfHa f es g gra konja, ak ar t obbsz or is, atmeszi egym ast [a;b]-n, az a-ban es b-ben all tott fugg} oleges egyenesnek,

Feladat 53

Van ket dobozunk. Az elsoben 4 piros es 4 feher golyo van, a masodikban 6 piros es 3 feher. Talalomrakivalasztunk egy dobozt, es abbol kiveszunk egy golyot. A dobozok kozul azonos valoszınuseggel valasztunk. Mi avaloszınusege annak, hogy a kivett golyo piros?Megoldas: Jelolje A azt az esemenyt, hogy a kivett golyo piros. Ha biztosan tudnank, hogy az elso dobozbolhuzunk, ennek valoszınusegere 1/2 lenne a valasz, ha azt tudnank, hogy a masodik dobozbol huzunk, 6/9 lenne avalasz. De nem tudjuk elore, hogy melyik dobozbol huzunk.A feladatban azonban szo van egy teljes esemenyrendszerrol is: jelolje B1 azt az esemenyt, hogy az elso dobozbolhuzunk, B2 azt, hogy a masodikbol. B1 es B2 nyilvan teljes esemenyrendszert alkot, es a feladat szovegebolP(B1) = P(B2) = 1/2. Ezert a teljes valoszınuseg tetele alapjan

P(A) = P(A|B1) · P(B1) + P(A|B2) · P(B2).

Itt P(A|B1) azt jelenti, hogy mi a valoszınusege annak, hogy a kivett golyo piros, felteve, hogy az elso dobozbolhuztuk. Ez persze 1/2. P(A|B2) azt jelenti, hogy mi a valoszınusege annak, hogy a kivett golyo piros, felteve,hogy a masodik dobozbol huztuk. Ez meg 6/9. Ezek alapjan tehat

P(A) = P(A|B1) · P(B1) + P(A|B2) · P(B2) =1

1

2+

6

1

2=

7

12.

Horvath Gabor Analızis 2

Page 198: Anal zis 2 - Széchenyi István Universityhorvathg/bsc/gazdmath2.pdfHa f es g gra konja, ak ar t obbsz or is, atmeszi egym ast [a;b]-n, az a-ban es b-ben all tott fugg} oleges egyenesnek,

Feladat 54

Van ket dobozunk. Az elsoben 4 piros es 4 feher golyo van, amasodikban 6 piros es 3 feher. Talalomra kivalasztunk egy dobozt, esabbol kiveszunk egy golyot. A dobozok kozul azonos valoszınuseggelvalasztunk. Ha tudjuk, hogy a kivett golyo feher lett, akkor mi avaloszınusege annak, hogy a masodik dobozbol huztuk?

Megoldas: Jelolje most A azt az esemenyt, hogy a kivett golyo feher. Afeladatban most is szo van egy teljes esemenyrendszerrol: jelolje B1 az azesemeny, hogy az elso dobozbol huzunk, B2 az, hogy a masodikbol. B1 esB2 nyilvan teljes esemenyrendszert alkot, es a feladat szovegebolP(B1) = P(B2) = 1/2.Amit ki kell szamolnunk az P(B2|A). A Bayes tetel alapjan

P(B2|A) =P(A|B2) · P(B2)

P(A|B1) · P(B1) + P(A|B2) · P(B2).

Ezert

P(B2|A) =39 ·

12

12 ·

12 + 3

9 ·12

=4

5

Horvath Gabor Analızis 2

Page 199: Anal zis 2 - Széchenyi István Universityhorvathg/bsc/gazdmath2.pdfHa f es g gra konja, ak ar t obbsz or is, atmeszi egym ast [a;b]-n, az a-ban es b-ben all tott fugg} oleges egyenesnek,

Feladat 54

Van ket dobozunk. Az elsoben 4 piros es 4 feher golyo van, amasodikban 6 piros es 3 feher. Talalomra kivalasztunk egy dobozt, esabbol kiveszunk egy golyot. A dobozok kozul azonos valoszınuseggelvalasztunk. Ha tudjuk, hogy a kivett golyo feher lett, akkor mi avaloszınusege annak, hogy a masodik dobozbol huztuk?Megoldas: Jelolje most A azt az esemenyt, hogy a kivett golyo feher. Afeladatban most is szo van egy teljes esemenyrendszerrol: jelolje B1 az azesemeny, hogy az elso dobozbol huzunk, B2 az, hogy a masodikbol. B1 esB2 nyilvan teljes esemenyrendszert alkot, es a feladat szovegebolP(B1) = P(B2) = 1/2.Amit ki kell szamolnunk az P(B2|A). A Bayes tetel alapjan

P(B2|A) =P(A|B2) · P(B2)

P(A|B1) · P(B1) + P(A|B2) · P(B2).

Ezert

P(B2|A) =39 ·

12

12 ·

12 + 3

9 ·12

=4

5

Horvath Gabor Analızis 2

Page 200: Anal zis 2 - Széchenyi István Universityhorvathg/bsc/gazdmath2.pdfHa f es g gra konja, ak ar t obbsz or is, atmeszi egym ast [a;b]-n, az a-ban es b-ben all tott fugg} oleges egyenesnek,

Esemenyek fuggetlensege

Definıcio 53

A,B ∈ A, P(A) > 0 es P(B) > 0. Az A esemeny fuggetlen a Besemenytol, ha

P(A|B) = P(A).

Tetel 37

1 Ha A fuggetlen B-tol, akkor B is fuggetlen A-tol.

2 A es B fuggetlenek pontosan akkor, ha P(AB) = P(A) · P(B).

3 Ha P(A) = 0 vagy P(A) = 1, akkor A minden B esemenytolfuggetlen.

4 A es B fuggetlenek, akkor A es B is fuggetlenek.

Az esemenyek fuggetlensegenek a definıcioja persze azt akarjamatematikai formaba onteni, hogy az egyik esemeny bekovetkezese nembefolyasolja a masik bekovetkezeset, ”fuggetlenek” egymastol. Altalabana fuggetlenseget feltetelezve hasznaljuk fel, hogy akkorP(AB) = P(A) · P(B).

Horvath Gabor Analızis 2

Page 201: Anal zis 2 - Széchenyi István Universityhorvathg/bsc/gazdmath2.pdfHa f es g gra konja, ak ar t obbsz or is, atmeszi egym ast [a;b]-n, az a-ban es b-ben all tott fugg} oleges egyenesnek,

Feladat 55

Feldobunk egy piros es egy feher kockat egyszerre. Jelolje A azt azesemenyt, hogy a piros kockaval hatost dobunk, B azt, hogy afeherrel negyest. Fuggetlenek-e ezek az esemenyek?

Megoldas: Ω elemei azonosıthatok hat elem osszes masodosztalyu ismetleses variaciojaval, |Ω| = 36. Az A esemenyszempontjanol hat kedvezo elemi esemeny van, es a Bszempontjabol is, azaz

P(A) = P(B) =6

36=

1

6.

De az AB esemeny szempontjabol nyilvan egy kedvezo kimenetelvan, ıgy

P(AB) =1

36=

1

6· 1

6= P(A) · P(B),

tehat az esemenyek fuggetlenek.

Horvath Gabor Analızis 2

Page 202: Anal zis 2 - Széchenyi István Universityhorvathg/bsc/gazdmath2.pdfHa f es g gra konja, ak ar t obbsz or is, atmeszi egym ast [a;b]-n, az a-ban es b-ben all tott fugg} oleges egyenesnek,

Feladat 55

Feldobunk egy piros es egy feher kockat egyszerre. Jelolje A azt azesemenyt, hogy a piros kockaval hatost dobunk, B azt, hogy afeherrel negyest. Fuggetlenek-e ezek az esemenyek?Megoldas: Ω elemei azonosıthatok hat elem osszes masodosztalyu ismetleses variaciojaval, |Ω| = 36. Az A esemenyszempontjanol hat kedvezo elemi esemeny van, es a Bszempontjabol is, azaz

P(A) = P(B) =6

36=

1

6.

De az AB esemeny szempontjabol nyilvan egy kedvezo kimenetelvan, ıgy

P(AB) =1

36=

1

6· 1

6= P(A) · P(B),

tehat az esemenyek fuggetlenek.

Horvath Gabor Analızis 2

Page 203: Anal zis 2 - Széchenyi István Universityhorvathg/bsc/gazdmath2.pdfHa f es g gra konja, ak ar t obbsz or is, atmeszi egym ast [a;b]-n, az a-ban es b-ben all tott fugg} oleges egyenesnek,

Feladat 56

Egy helyiseget ket uton lehet megkozelıteni. Egy-egy auto indul el a ket uton. Annak valoszınusege, hogy az elsouton hofuvas miatt elakad a gepkocsi 0.8, annak, hogy a masodikon elakad 0.65. Mi a valoszınusege annak, hogylegalabb az egyik gepkocsi celbaer?

Megoldas: Jeloljuk A-val azt az esemenyt, hogy az elso uton elindulo auto elakad, B-vel azt, hogy a masodik utonelindulo elakad. Ekkor AB azt jelenti, hogy mindket auto elakad. AB azt jelenti, hogy nem akad el mindket auto,vagyis legalabb az egyik celba er. Tehat P(AB)-t kell kiszamolnunk.

Joggal feltehetjuk, hogy A es B fuggetlenek egymastol. Igy

P(AB) = 1− P(AB) = 1− P(A)P(B) = 1− 0.52 = 0.48.

Ugy is okoskodhattunk volna, hogy A azt jelenti, hogy az elso gepkocsi nem akad el, B azt, hogy az masodikgepkocsi nem akad el. Ekkor amit ki akarunk szamolni az P(A + B). De, felhasznalva, hogy A es B is fuggetlenek,

P(A + B) = P(A) + P(B)− P(A)P(B) = 0.2 + 0.35− 0.2 · 0.35 = 0.48,

persze ugyan az, mint az elobb.

Horvath Gabor Analızis 2

Page 204: Anal zis 2 - Széchenyi István Universityhorvathg/bsc/gazdmath2.pdfHa f es g gra konja, ak ar t obbsz or is, atmeszi egym ast [a;b]-n, az a-ban es b-ben all tott fugg} oleges egyenesnek,

Feladat 56

Egy helyiseget ket uton lehet megkozelıteni. Egy-egy auto indul el a ket uton. Annak valoszınusege, hogy az elsouton hofuvas miatt elakad a gepkocsi 0.8, annak, hogy a masodikon elakad 0.65. Mi a valoszınusege annak, hogylegalabb az egyik gepkocsi celbaer?Megoldas: Jeloljuk A-val azt az esemenyt, hogy az elso uton elindulo auto elakad, B-vel azt, hogy a masodik utonelindulo elakad. Ekkor AB azt jelenti, hogy mindket auto elakad. AB azt jelenti, hogy nem akad el mindket auto,vagyis legalabb az egyik celba er. Tehat P(AB)-t kell kiszamolnunk.

Joggal feltehetjuk, hogy A es B fuggetlenek egymastol. Igy

P(AB) = 1− P(AB) = 1− P(A)P(B) = 1− 0.52 = 0.48.

Ugy is okoskodhattunk volna, hogy A azt jelenti, hogy az elso gepkocsi nem akad el, B azt, hogy az masodikgepkocsi nem akad el. Ekkor amit ki akarunk szamolni az P(A + B). De, felhasznalva, hogy A es B is fuggetlenek,

P(A + B) = P(A) + P(B)− P(A)P(B) = 0.2 + 0.35− 0.2 · 0.35 = 0.48,

persze ugyan az, mint az elobb.

Horvath Gabor Analızis 2

Page 205: Anal zis 2 - Széchenyi István Universityhorvathg/bsc/gazdmath2.pdfHa f es g gra konja, ak ar t obbsz or is, atmeszi egym ast [a;b]-n, az a-ban es b-ben all tott fugg} oleges egyenesnek,

Esemenyek fuggetlensege

Definıcio 54

Az A1, A2, . . . , An esemenyek teljesen fuggetlenek, ha mindenk = 2, . . . , n eseten

P(Ai1Ai2 , . . . ,Aik ) = P(Ai1) · P(Ai2) · . . . · P(Aik ).

Tehat az esemenyek kozul akarhany egyuttes bekovetkezesenek avaloszınusege egyenlo az esemenyek valoszınusegeinek szorzataval.

Tetel 38

Ha az A1, A2, . . . , An esemenyek teljesen fuggetlenek, akkorkozuluk akarmennyit a negaltjara lecserelve ismet teljesenfuggetlen esemenyeket kapunk.

Horvath Gabor Analızis 2

Page 206: Anal zis 2 - Széchenyi István Universityhorvathg/bsc/gazdmath2.pdfHa f es g gra konja, ak ar t obbsz or is, atmeszi egym ast [a;b]-n, az a-ban es b-ben all tott fugg} oleges egyenesnek,

Feladat 57

Ot katona kozul mindegyik 100 lovesbol atlagosan 80-szor eltalalja aceltablat. Az ot katona egyszerre ralo a celtablara. Mi a valoszınusegeannak, hogy kevesebb, mint ket talalat eri a celtablat?

Megoldas: Annak a valoszınusege, hogy egy adott katona eltalalja aceltablat 0.8. Jelolje A1 azt az esemenyt, hogy az elso katona eltalalja aceltablat, es ıgy tovabb, A5 azt az esemenyt, hogy az otodik katonaeltalalja a celtablat. Ekkor joggal feltetelezhetjuk, hogy ezek azesemenyek teljesen fuggetlenek.Kevesebb, mint ket talalat akkor eri a celtablat, ha egy sem eri vagy, hapontoson egy eri. Ha B jeloli azt az esemenyt, hogy kevesebb, mint kettalalat eri a celtablat, akkor

B = A1 A2 A3 A4 A5 + A1A2 A3 A4 A5 + A1A2A3 A4 A5 + A1 A2A3A4 A5 + A1 A2 A3A4A5 + A1 A2 A3 A4A5

Ezek az osszeadandok egymast paronkent kizarjak, es a teljesfuggetlenseg miatt

P(B) = 0.25 + 5 · 0.8 · 0.24 = 0.00672,

ahogy azt vathattuk is, eleg kicsi.

Horvath Gabor Analızis 2

Page 207: Anal zis 2 - Széchenyi István Universityhorvathg/bsc/gazdmath2.pdfHa f es g gra konja, ak ar t obbsz or is, atmeszi egym ast [a;b]-n, az a-ban es b-ben all tott fugg} oleges egyenesnek,

Feladat 57

Ot katona kozul mindegyik 100 lovesbol atlagosan 80-szor eltalalja aceltablat. Az ot katona egyszerre ralo a celtablara. Mi a valoszınusegeannak, hogy kevesebb, mint ket talalat eri a celtablat?Megoldas: Annak a valoszınusege, hogy egy adott katona eltalalja aceltablat 0.8. Jelolje A1 azt az esemenyt, hogy az elso katona eltalalja aceltablat, es ıgy tovabb, A5 azt az esemenyt, hogy az otodik katonaeltalalja a celtablat. Ekkor joggal feltetelezhetjuk, hogy ezek azesemenyek teljesen fuggetlenek.Kevesebb, mint ket talalat akkor eri a celtablat, ha egy sem eri vagy, hapontoson egy eri. Ha B jeloli azt az esemenyt, hogy kevesebb, mint kettalalat eri a celtablat, akkor

B = A1 A2 A3 A4 A5 + A1A2 A3 A4 A5 + A1A2A3 A4 A5 + A1 A2A3A4 A5 + A1 A2 A3A4A5 + A1 A2 A3 A4A5

Ezek az osszeadandok egymast paronkent kizarjak, es a teljesfuggetlenseg miatt

P(B) = 0.25 + 5 · 0.8 · 0.24 = 0.00672,

ahogy azt vathattuk is, eleg kicsi.

Horvath Gabor Analızis 2

Page 208: Anal zis 2 - Széchenyi István Universityhorvathg/bsc/gazdmath2.pdfHa f es g gra konja, ak ar t obbsz or is, atmeszi egym ast [a;b]-n, az a-ban es b-ben all tott fugg} oleges egyenesnek,

Valoszınusegi valtozok

Egy kıserlet kimenetelei gyakran matematikailag nehezen kezelhetodolgok. Peldaul kartyalapokbol allo rendezett harmasok, termekekegy adott elemu halmaza, stb. Celszeru a kısetlet kimeneteleihezszamokat rendelni, ahogy ezt mar eddig is tobbszor megtettuk. Igyolyan mennyisegeket definialtunk, amelyek veletlentol fuggo erteketvesznek fel, durvan szolva ezeket hıvjuk valoszınusegi valtozoknak.A valoszınusegi valtozo egy kıserlethez tartozo fuggveny, es aveletlentol fugg, hogy milyen erteket vesz fel.A valoszınusegi valtozoknak ket fajtaja van: diszkret es folytonos.A kovetkezokben ezekkel fogunk foglalkozni, eloszor a diszkretvaloszınusegi valtozokkal.A diszkret valoszınusegi valtozok az egyszerubbek, ilyenek lepnekfel peldaul akkor, ha az Ω halmaz veges.

Horvath Gabor Analızis 2

Page 209: Anal zis 2 - Széchenyi István Universityhorvathg/bsc/gazdmath2.pdfHa f es g gra konja, ak ar t obbsz or is, atmeszi egym ast [a;b]-n, az a-ban es b-ben all tott fugg} oleges egyenesnek,

Diszkret valoszınusegi valtozok

Definıcio 55

Adott egy (Ω,A,P) Kolmogorov fele valoszınusegi mezo. Aξ : Ω −→ R fuggveny diszkret valoszınusegi valtozo, ha az Rξertekkeszlete veges vagy megszamlalhatoan vegtelen halmaz, esminden x ∈ Rξ eseten Ax = ξ = x = ω ∈ Ω|ξ(ω) = x ∈ A.

A definıcio masodik felenek az a lenyege, hogy egy jolmeghatarozott valoszınusege van annak, hogy a ξ egy adott xerteket vesz fel, megpedig P(Ax ) = P(ξ = x). Mi afeladatokban altalaban olyan diszkret valoszınusegi valtozokkalfogunk foglalkozni, amelyek veges sok erteket vehetnek fel.

Horvath Gabor Analızis 2

Page 210: Anal zis 2 - Széchenyi István Universityhorvathg/bsc/gazdmath2.pdfHa f es g gra konja, ak ar t obbsz or is, atmeszi egym ast [a;b]-n, az a-ban es b-ben all tott fugg} oleges egyenesnek,

Feladat 58

Legyen (Ω,A, P) a kockadobast leıro Kolmogorov fele valoszınusegi mezo. Legyen ξ a dobott szam pozitıvosztoinak szama. Adjuk meg ezt a ξ-t.

Megoldas: Egy diszkret valoszınusegi valtozot az hataroz meg, hogy milyen ertekeket vesz fel, es hogy azokatmilyen valoszınuseggel veszi fel.

A dobott szam 1 2 3 4 5 6A pozitıv osztoinak szama 1 2 2 3 2 4

Vilagos, hogy ξ az 1, 2, 3 es 4 ertekeket veszi fel, rendre P(ξ = 1) = p1 = 16

, P(ξ = 2) = p2 = 36

,

P(ξ = 3) = p3 = 16

es P(ξ = 4) = p4 = 16

valoszınuseggel.Ezt is celszeru tablazatban osszefoglalni.

ξ 1 2 3 416

36

16

16

Horvath Gabor Analızis 2

Page 211: Anal zis 2 - Széchenyi István Universityhorvathg/bsc/gazdmath2.pdfHa f es g gra konja, ak ar t obbsz or is, atmeszi egym ast [a;b]-n, az a-ban es b-ben all tott fugg} oleges egyenesnek,

Feladat 58

Legyen (Ω,A, P) a kockadobast leıro Kolmogorov fele valoszınusegi mezo. Legyen ξ a dobott szam pozitıvosztoinak szama. Adjuk meg ezt a ξ-t.Megoldas: Egy diszkret valoszınusegi valtozot az hataroz meg, hogy milyen ertekeket vesz fel, es hogy azokatmilyen valoszınuseggel veszi fel.

A dobott szam 1 2 3 4 5 6A pozitıv osztoinak szama 1 2 2 3 2 4

Vilagos, hogy ξ az 1, 2, 3 es 4 ertekeket veszi fel, rendre P(ξ = 1) = p1 = 16

, P(ξ = 2) = p2 = 36

,

P(ξ = 3) = p3 = 16

es P(ξ = 4) = p4 = 16

valoszınuseggel.Ezt is celszeru tablazatban osszefoglalni.

ξ 1 2 3 416

36

16

16

Horvath Gabor Analızis 2

Page 212: Anal zis 2 - Széchenyi István Universityhorvathg/bsc/gazdmath2.pdfHa f es g gra konja, ak ar t obbsz or is, atmeszi egym ast [a;b]-n, az a-ban es b-ben all tott fugg} oleges egyenesnek,

Teljes esemenyrendszer

Definıcio 56

Az A1, A2, . . . veges vagy megszamlalhatoan sok esemeny teljes esemenyrendszert alkot, ha

1 egymast paronkent kizarjak,

2 A1 + A2 + . . . = Ω.

Tetel 39

Legyen ξ diszkret valoszınusegi valtozo, az ertekkeszlete Rξ = x1, x2, . . .. Ekkor azAxi

= Ai = ω ∈ Ω|ξ(ω) = xi esemenyek teljes esemenyrenszert alkotnak.

A pi = P(Ai ) = P(ξ = xi) = P(ξ = xi ) nemnegatıv szamokra pedig∑

ipi = 1.

Definıcio 57

Az elobbi pi szamok rendszeret valoszınusegeloszlasnak hıvjuk.

Tetel 40

Ha ξ es η diszkret valoszınusegi valtozok ugyanazon a (Ω,A, P) Kolmogorov fele valoszınusegi mezon,

f : R −→ R es g : R2 −→ R tetszoleges egy- illetve ketvaltozos fuggvenyek, akkor f (ξ) es g(ξ, η) is diszkretvaloszınusegi valtozok, ha mint fuggvenyek tovabbra is az egesz Ω-n ertelmezve vannak.

Ennek a tetelnek a kovetkezmenyekent ξ2,√ξ, 1ξ

, sin(ξ), ξ · η, ξ + η, ξη

, stb. mind diszkret valoszınusegi

valtozok, ha mint fuggvenyek tovabbra is az egesz Ω-n ertelmezve vannak.

Horvath Gabor Analızis 2

Page 213: Anal zis 2 - Széchenyi István Universityhorvathg/bsc/gazdmath2.pdfHa f es g gra konja, ak ar t obbsz or is, atmeszi egym ast [a;b]-n, az a-ban es b-ben all tott fugg} oleges egyenesnek,

Az eloszlasfuggveny

Egy ξ : Ω −→ R valoszınusegi valtozo altalaban matematikai szempontbol nehezen kezelheto fuggveny, mert az Ωegy absztrakt halmaz, az elemeivel nem nagyon lehet ”szamolni”. (Pedaul katyahuzas eseten az elemi esemenyekkartyakbol allo halmazok.) Emiatt a ξ-vel is csak korlatozottan lehet ”szamolni”, peldaul altalaban nem lehet ahatarerteket venni, derivalni, stb.Ezert minden ξ valoszınusegi valtozohoz hozzarendelunk egy Fξ : R −→ R egyvaltozos valos fuggvenyt, amely joljellemzi ξ-t, es amelyet mar az eddigi analızis tanulmanyaink minden eszkozevel vizsgalhatunk.

Definıcio 58

Legyen ξ diszkret valoszınusegi valtozo (Ω,A, P)-n, Rξ = x1, x2, . . .. A ξvaloszınusegi valtozo Fξ : R −→ Reloszlasfuggvenye az az egyvaltozos valos fuggveny, amelyre minden x ∈ R eseten

Fξ(x) = P(ξ < x) = P(ξ < x),

ahol persze ξ < x = ω ∈ Ω|ξ(ω) < x.

A diszkret valoszınusegi valtozo definıcioja garantalja, hogy ξ < x ∈ A, tehat van valoszınusege.

Tetel 41

ξ < x =∑

ixi<x

ω ∈ Ω|ξ(ω) = xi =∑

ixi<x

Ai =∑

ixi<x

ξ = xi, es mivel ezek az esemenyek egymast

paronkent kizarjak

Fξ(x) =∑

ixi<x

P(ξ = xi) =∑

ixi<x

P(ξ = xi ) =∑

ixi<x

pi .

Horvath Gabor Analızis 2

Page 214: Anal zis 2 - Széchenyi István Universityhorvathg/bsc/gazdmath2.pdfHa f es g gra konja, ak ar t obbsz or is, atmeszi egym ast [a;b]-n, az a-ban es b-ben all tott fugg} oleges egyenesnek,

Az eloszlasfuggveny

Tetel 42

A ξ diszkret valoszınusegi valtozo Fξ eloszlasfuggvenye rendelkezikaz alabbi tulajdonsagokkal:

1 0 ≤ Fξ(x) ≤ 1 minden x ∈ R eseten.

2 Fξ monoton novo fuggveny.

3 Fξ mindenhol balrol folytonos.

4 limx−→−∞

Fξ(x) = 0. limx−→∞

Fξ(x) = 1.

Egy diszkret valoszınusegi valtozo eloszlasfuggvenye egy lepcsosfuggveny. Azokon a helyeken ugrik, amilyen ertekeket avaloszınusegi valtozo felvesz. Minden ilyen helyen az ugras akkora,amekkora valoszınuseggel a szobanforgo erteket felveszi avaloszınusegi valtozo.

Horvath Gabor Analızis 2

Page 215: Anal zis 2 - Széchenyi István Universityhorvathg/bsc/gazdmath2.pdfHa f es g gra konja, ak ar t obbsz or is, atmeszi egym ast [a;b]-n, az a-ban es b-ben all tott fugg} oleges egyenesnek,

Feladat 59

Legyen (Ω,A, P) a kockadobast leıro Kolmogorov fele valoszınusegi mezo. Legyen ξ a dobott szam pozitıvosztoinak szama. Adjuk meg a ξ eloszlasfuggvenyet.

Megoldas: Mar tudjuk, hogy ξ milyen ertekeket milyen valoszınuseggel vesz fel:

ξ 1 2 3 416

36

16

16

Ebbol az eloszlasfuggveny mar meghatarozhato.A ξ ertekei feldaraboljak a valos szamegyenest. Ez lathato a fenti abran. Meghatarozzuk, hogy az egyesdarabokban milyen erteket vesz fel Fξ .

Ha x ≤ 1, akkor ξ < x = ∅, tehat Fξ(x) = 0.

Ha 1 < x ≤ 2, akkor ξ < x = ξ = 1, tehat Fξ(x) = p1 = 16

.

Ha 2 < x ≤ 3, akkor ξ < x = ξ = 1 + ξ = 2, tehat Fξ(x) = p1 + p2 = 46

.

Ha 3 < x ≤ 4, akkor ξ < x = ξ = 1 + ξ = 2 + ξ = 3, tehat Fξ(x) = p1 + p2 + p3 = 56

.

Ha 4 < x, akkor ξ < x = ξ = 1 + ξ = 2 + ξ = 3 + ξ = 4, tehatFξ(x) = p1 + p2 + p3 + p4 = 1. Vagyis

Fξ(x) =

0, ha x ≤ 1,1/6, ha 1 < x ≤ 2,4/6, ha 2 < x ≤ 3,5/6, ha 3 < x ≤ 4,1, ha 4 < x.

Az eloszlasfuggveny grafikonja a jobb oldali abran lathato.

Horvath Gabor Analızis 2

Page 216: Anal zis 2 - Széchenyi István Universityhorvathg/bsc/gazdmath2.pdfHa f es g gra konja, ak ar t obbsz or is, atmeszi egym ast [a;b]-n, az a-ban es b-ben all tott fugg} oleges egyenesnek,

Feladat 59

Legyen (Ω,A, P) a kockadobast leıro Kolmogorov fele valoszınusegi mezo. Legyen ξ a dobott szam pozitıvosztoinak szama. Adjuk meg a ξ eloszlasfuggvenyet.Megoldas: Mar tudjuk, hogy ξ milyen ertekeket milyen valoszınuseggel vesz fel:

ξ 1 2 3 416

36

16

16

Ebbol az eloszlasfuggveny mar meghatarozhato.A ξ ertekei feldaraboljak a valos szamegyenest. Ez lathato a fenti abran. Meghatarozzuk, hogy az egyesdarabokban milyen erteket vesz fel Fξ .

Ha x ≤ 1, akkor ξ < x = ∅, tehat Fξ(x) = 0.

Ha 1 < x ≤ 2, akkor ξ < x = ξ = 1, tehat Fξ(x) = p1 = 16

.

Ha 2 < x ≤ 3, akkor ξ < x = ξ = 1 + ξ = 2, tehat Fξ(x) = p1 + p2 = 46

.

Ha 3 < x ≤ 4, akkor ξ < x = ξ = 1 + ξ = 2 + ξ = 3, tehat Fξ(x) = p1 + p2 + p3 = 56

.

Ha 4 < x, akkor ξ < x = ξ = 1 + ξ = 2 + ξ = 3 + ξ = 4, tehatFξ(x) = p1 + p2 + p3 + p4 = 1. Vagyis

Fξ(x) =

0, ha x ≤ 1,1/6, ha 1 < x ≤ 2,4/6, ha 2 < x ≤ 3,5/6, ha 3 < x ≤ 4,1, ha 4 < x.

Az eloszlasfuggveny grafikonja a jobb oldali abran lathato.

Horvath Gabor Analızis 2

Page 217: Anal zis 2 - Széchenyi István Universityhorvathg/bsc/gazdmath2.pdfHa f es g gra konja, ak ar t obbsz or is, atmeszi egym ast [a;b]-n, az a-ban es b-ben all tott fugg} oleges egyenesnek,

Varhato ertek

Legyen ξ diszkret valoszınusegi valtozo, Rξ = x1, x2, . . ., pi = P(ξ = xi). Vegezzunk N megfigyelest ξertekere. Ekkor kb. Np1 esetben lesz ξ = x1, Np2 esetben lesz ξ = x2, stb. Kepezzuk az N megfigyeles sorankapott ξ ertekek atlagat. Ekkor korulbelul az

Np1x1 + Np2x2 + . . .

N=∑

i

pi xi

erteket kapjuk. Ez a motivacioja az alabbi definıcionak.

Definıcio 59

A ξ diszkret valoszınusegi valtozo varhato erteke az

M(ξ) =∑

i

pi xi ,

szam, ha ez az osszeg letezik es veges. Egyebbkent a ξ-nek nincs varhato erteke.

Ha ξ csak veges sok kulonbozo erteket vesz fel, akkor mindig letezik a varhato erteke. Ha a valoszınusegeloszlasttomegeloszlasnak fogjuk fel, azaz pi sulyt helyezunk az xi pontba, akkor a varhato ertek eppen a sulypont.

Horvath Gabor Analızis 2

Page 218: Anal zis 2 - Széchenyi István Universityhorvathg/bsc/gazdmath2.pdfHa f es g gra konja, ak ar t obbsz or is, atmeszi egym ast [a;b]-n, az a-ban es b-ben all tott fugg} oleges egyenesnek,

Varhato ertek

Tetel 43

Legyen c tetszoleges valos szam. Ha letezik az M(ξ) es M(η) varhato ertek, akkor

1 M(c) = c, (a valoszınusegi valtozo most konstans, egyedul a c erteket veszi fel 1 valoszınuseggel).

2 M(c · ξ) = c · M(ξ).

3 M(ξ + η) = M(ξ) + M(η).

Definıcio 60

A ξ, Rξ = x1, x2, . . ., pi = P(ξ = xi) es az η, Rη = y1, y2, . . ., qi = P(η = yi) diszkretvaloszınusegi valtozok fuggetlenek, ha minden i es j eseten

P(ξ = xiη = yj) = P(ξ = xi , η = yj ) = P(ξ = xi ) · P(η = yj ).

Tetel 44

Ha ξ es η fuggetlenek, akkor M(ξ · η) = M(ξ) · M(η).

Definıcio 61

A ξ1, ξ2, . . . , ξn diszkret valoszınusegi valtozok paronkent fuggetlenek, ha barmely ketto kulonbozo kozulukfuggetlen.

Tetel 45

Ha a ξ1, ξ2, . . . , ξn diszkret valoszınusegi valtozok paronkent fuggetlenek, akkor a g1(ξ1), g2(ξ2), . . . , gn(ξn)valoszınusegi valtozok is paronkent fuggetlenek tetszoleges, minden ω-ra ertelmezett g1, g2, . . . , gn fuggvenyekeseten.

Horvath Gabor Analızis 2

Page 219: Anal zis 2 - Széchenyi István Universityhorvathg/bsc/gazdmath2.pdfHa f es g gra konja, ak ar t obbsz or is, atmeszi egym ast [a;b]-n, az a-ban es b-ben all tott fugg} oleges egyenesnek,

Szoras

Ha megfigyeljuk egy valoszınusegi valtozo ertekeit, akkor azt tapasztaljuk, hogy az gyakran esik a varhato ertekkozelebe, akorul ingadozik. Vannak valoszınusegi valtozok, amelyeknel ez az ingadozas kicsi, vannak amelyeknelnagy. Az ingadozas merteket szeretnenk valahogy jellemezni. A ξ − M(ξ) mennyiseg meri azt, hogy a ξ mennyireter el a varhato erteketol. Ez is egy valoszınusegi valtozo, deM(ξ−M(ξ)) = M(ξ)−M(M(ξ)) = M(ξ)−M(ξ) = 0, azaz a varhato ertektol valo pozitıv es negatıv elteresekkiegyenlıtik egymast. Ez nem jo az ingadozas mertekere. Felmerul a |ξ − M(ξ)| mennyiseg, de az abszolut ertekmiatt ez nehezen kezelheto. Ezert az alabbi definıciot valasztjuk.

Definıcio 62

A ξ diszkret valoszınusegi valtozo szorasnegyzete a

D2(ξ) = M((ξ − M(ξ))2)

szam, ha ez letezik es veges. Egyebbkent a ξ-nek nincs szorasnegyzete.

A szorasnegyzete pozitıv negyzetgyoke a D(ξ)-vel jelolt szoras, azaz D(ξ) =√

D2(ξ).

Tetel 46

1 Ha letezik a D2(ξ), akkor D2(ξ) = M(ξ2)− (M(ξ))2.

2 Ha letezik a D2(ξ), akkor D2(aξ + b) = a2D2(ξ) tetszoleges a, b valos szamokra.

3 Ha a ξ1, ξ2, . . . , ξn diszkret valoszınusegi valtozok paronkent fuggetlenek, es minden i-re letezik a

D2(ξi ), akkor

D2(ξ1 + ξ2 + . . . + ξn) = D2(ξ1) + D2(ξ2) + . . . + D2(ξn).

Horvath Gabor Analızis 2

Page 220: Anal zis 2 - Széchenyi István Universityhorvathg/bsc/gazdmath2.pdfHa f es g gra konja, ak ar t obbsz or is, atmeszi egym ast [a;b]-n, az a-ban es b-ben all tott fugg} oleges egyenesnek,

Feladat 60

Feldobunk egy kockat. Jelolje ξ a dobott szamnal harommal kisebb szamot. Szamoljuk ki ξ varhato erteket esszorasat.

Megoldas: A bal oldali tablazat tartalmazza ξ lehetseges ertekeit, es hogy azokat milyen valoszınuseggel veszi fel :

ξ −2 −1 0 1 2 316

16

16

16

16

16

ξ2 0 1 4 916

26

26

16

Ezt felhasznalva

M(ξ) =1

6· (−2) +

1

6· (−1) +

1

6· 0 +

1

6· 1 +

1

6· 2 +

1

6· 3 =

1

2.

A szorasnegyzet kiszamolasahoz eloszor meghatarozzuk a ξ2 ertekeit, es azt, hogy azokat milyen valoszınuseggelveszi fel. ξ2 lehetseges ertekei: 0, 1, 4, 9.

p1 = P(ξ2 = 0) = P(ξ = 0) = 16

.

p2 = P(ξ2 = 1) = P(ξ = −1 + ξ = 1) = P(ξ = −1) + P(ξ = 1) = 26

.

p3 = P(ξ2 = 4) = P(ξ = −2 + ξ = 2) = P(ξ = −2) + P(ξ = 2) = 26

.

p4 = P(ξ2 = 9) = P(ξ = 3) = 16

.

ξ2 adatait a jobb oldali tablazat tartalmazza.Ezutan

M(ξ2) =1

6· 0 +

2

6· 1 +

2

6· 4 +

1

6· 9 =

19

6.

Igy

D2(ξ) = M(ξ2)− (M(ξ))2 =

(19

6

)−(

1

2

)2

=35

12,

azaz

D(ξ) =

√105

6≈ 1.7.

Horvath Gabor Analızis 2

Page 221: Anal zis 2 - Széchenyi István Universityhorvathg/bsc/gazdmath2.pdfHa f es g gra konja, ak ar t obbsz or is, atmeszi egym ast [a;b]-n, az a-ban es b-ben all tott fugg} oleges egyenesnek,

Feladat 60

Feldobunk egy kockat. Jelolje ξ a dobott szamnal harommal kisebb szamot. Szamoljuk ki ξ varhato erteket esszorasat.Megoldas: A bal oldali tablazat tartalmazza ξ lehetseges ertekeit, es hogy azokat milyen valoszınuseggel veszi fel :

ξ −2 −1 0 1 2 316

16

16

16

16

16

ξ2 0 1 4 916

26

26

16

Ezt felhasznalva

M(ξ) =1

6· (−2) +

1

6· (−1) +

1

6· 0 +

1

6· 1 +

1

6· 2 +

1

6· 3 =

1

2.

A szorasnegyzet kiszamolasahoz eloszor meghatarozzuk a ξ2 ertekeit, es azt, hogy azokat milyen valoszınuseggelveszi fel. ξ2 lehetseges ertekei: 0, 1, 4, 9.

p1 = P(ξ2 = 0) = P(ξ = 0) = 16

.

p2 = P(ξ2 = 1) = P(ξ = −1 + ξ = 1) = P(ξ = −1) + P(ξ = 1) = 26

.

p3 = P(ξ2 = 4) = P(ξ = −2 + ξ = 2) = P(ξ = −2) + P(ξ = 2) = 26

.

p4 = P(ξ2 = 9) = P(ξ = 3) = 16

.

ξ2 adatait a jobb oldali tablazat tartalmazza.Ezutan

M(ξ2) =1

6· 0 +

2

6· 1 +

2

6· 4 +

1

6· 9 =

19

6.

Igy

D2(ξ) = M(ξ2)− (M(ξ))2 =

(19

6

)−(

1

2

)2

=35

12,

azaz

D(ξ) =

√105

6≈ 1.7.

Horvath Gabor Analızis 2

Page 222: Anal zis 2 - Széchenyi István Universityhorvathg/bsc/gazdmath2.pdfHa f es g gra konja, ak ar t obbsz or is, atmeszi egym ast [a;b]-n, az a-ban es b-ben all tott fugg} oleges egyenesnek,

Feladat 61

Egy bobozban harom piros es ot feher golyo van. Egymas utan kiveszunk negy golyot. Jelolje ξ a kivett fehergolyok szamat. Szamoljuk ki annak a valoszınuseget, hogy ξ a varhato ertekenel nagyobb erteket vesz fel.

Megoldas: ξ lehetseges ertekei: 1, 2, 3, 4, hiszen nincs negy piros golyo, legalabb egy feher golyot biztos kiveszunk.Meghatarozzuk, hogy ξ milyen valoszınuseggel veszi fel az ertekeit. A kıserlethez tartozo (Ω,A, P)-ben|Ω| = 8 · 7 · 6 · 5 = 1680, hiszen a golyok huzasanak van sorrendje es ismetlodes nem lehet, tehat nyolc elemnegyed osztalyu ismetles nelkuli variaciorol van szo.

p1 = P(ξ = 1) =

(41

)·V 1

5 ·V33

1680= 120

1680= 1

14.

p2 = P(ξ = 2) =

(42

)·V 2

5 ·V23

1680= 720

1680= 6

14.

p3 = P(ξ = 3) =

(43

)·V 3

5 ·V13

1680= 720

1680= 6

14.

p4 = P(ξ = 4) =

(44

)·V 4

5 ·V03

1680= 120

1680= 1

14.

ξ 1 2 3 41

146

146

141

14

A varhato ertek most

M(ξ) =1

14· 1 +

6

14· 2 +

6

14· 3 +

1

14· 4 =

35

14.

Igy

P(ξ > M(ξ)) = P

(ξ >

35

14

)= P(ξ = 3 + ξ = 4) = P(ξ = 3) + P(ξ = 4) =

7

14=

1

2.

Horvath Gabor Analızis 2

Page 223: Anal zis 2 - Széchenyi István Universityhorvathg/bsc/gazdmath2.pdfHa f es g gra konja, ak ar t obbsz or is, atmeszi egym ast [a;b]-n, az a-ban es b-ben all tott fugg} oleges egyenesnek,

Feladat 61

Egy bobozban harom piros es ot feher golyo van. Egymas utan kiveszunk negy golyot. Jelolje ξ a kivett fehergolyok szamat. Szamoljuk ki annak a valoszınuseget, hogy ξ a varhato ertekenel nagyobb erteket vesz fel.Megoldas: ξ lehetseges ertekei: 1, 2, 3, 4, hiszen nincs negy piros golyo, legalabb egy feher golyot biztos kiveszunk.Meghatarozzuk, hogy ξ milyen valoszınuseggel veszi fel az ertekeit. A kıserlethez tartozo (Ω,A, P)-ben|Ω| = 8 · 7 · 6 · 5 = 1680, hiszen a golyok huzasanak van sorrendje es ismetlodes nem lehet, tehat nyolc elemnegyed osztalyu ismetles nelkuli variaciorol van szo.

p1 = P(ξ = 1) =

(41

)·V 1

5 ·V33

1680= 120

1680= 1

14.

p2 = P(ξ = 2) =

(42

)·V 2

5 ·V23

1680= 720

1680= 6

14.

p3 = P(ξ = 3) =

(43

)·V 3

5 ·V13

1680= 720

1680= 6

14.

p4 = P(ξ = 4) =

(44

)·V 4

5 ·V03

1680= 120

1680= 1

14.

ξ 1 2 3 41

146

146

141

14

A varhato ertek most

M(ξ) =1

14· 1 +

6

14· 2 +

6

14· 3 +

1

14· 4 =

35

14.

Igy

P(ξ > M(ξ)) = P

(ξ >

35

14

)= P(ξ = 3 + ξ = 4) = P(ξ = 3) + P(ξ = 4) =

7

14=

1

2.

Horvath Gabor Analızis 2

Page 224: Anal zis 2 - Széchenyi István Universityhorvathg/bsc/gazdmath2.pdfHa f es g gra konja, ak ar t obbsz or is, atmeszi egym ast [a;b]-n, az a-ban es b-ben all tott fugg} oleges egyenesnek,

Nevezetes valoszınusegi valtozok

A nevezetes valoszınusegi valtozok a valoszınusegszamıtas elmeleteben esgyakorlati alkalmazasaban gyakran felbukkano eloszlas tıpusok, ezertnagy a jelentoseguk.

Definıcio 63

Legyen A olyan esemenye (Ω,A,P)-nek, amelyre P(A) > 0. Az Aesemeny karekterisztikus valoszınusegi valtozoja a

ξA(ω) =

1, ha ω ∈ A,0 ha ω 6∈ A.

valoszınusegi valtozo. ξA tehat ket erteket vesz fel, 1-et P(A), 0-t P(A)valoszınuseggel.

Tetel 47

A ξA karekterisztikus valoszınusegi valtozo eseten

1 M(ξA) = P(A).

2 D2(ξA) = P(A)P(A).

Horvath Gabor Analızis 2

Page 225: Anal zis 2 - Széchenyi István Universityhorvathg/bsc/gazdmath2.pdfHa f es g gra konja, ak ar t obbsz or is, atmeszi egym ast [a;b]-n, az a-ban es b-ben all tott fugg} oleges egyenesnek,

A hipergeometriai eloszlasu valoszınusegi valtozo(visszateves nelkuli mintavetel)

Definıcio 64

Egy dobozban N termek van, amelyek kozul s darab selejtes. Kivalasztunk egyszerre n darabot kozuluk.Feltesszuk, hogy 0 ≤ n ≤ s ≤ N, es hogy n ≤ N − s. Jelolje ξ a mintaban levo selejt alkatreszek szamat. Ekkorξ N, s, n parameteru hipergeometria eloszlasu valoszınusegi valtozo. Ennek a ξ-nek a lethetseges ertekei:0, 1, . . . , n, es a k erteket

P(ξ = k) =

(sk

)(N−sn−k

)(

Nn

)valoszınuseggel veszi fel. A p = s

Nszam annak a valoszınusege, hogy egy termek selejtes.

Tetel 48

Az N, s, n parameteru hipergeometria eloszlasu valoszınusegi valtozo eseten

1 M(ξ) = sN· n = pn.

2 D2(ξ) = n sN

(1− s

N

)N−nN−1

= np(1− p) N−nN−1

.

Horvath Gabor Analızis 2

Page 226: Anal zis 2 - Széchenyi István Universityhorvathg/bsc/gazdmath2.pdfHa f es g gra konja, ak ar t obbsz or is, atmeszi egym ast [a;b]-n, az a-ban es b-ben all tott fugg} oleges egyenesnek,

A binomialis eloszlasu valoszınusegi valtozo (visszatevesesmintavetel)

Definıcio 65

Egy dobozban N termek van, amelyek kozul s darab selejtes. Feltesszuk, hogy 0 ≤ s ≤ N. Kivalasztunk egymasutan n darabot kozuluk ugy, hogy a kivalasztott alkatreszt mindig visszatesszuk. Jelolje ξ a mintaban levo selejtalkatreszek szamat. Ekkor ξ n, p = s

Nparameteru binomialis eloszlasu valoszınusegi valtozo. Ennek a ξ-nek a

lethetseges ertekei: 0, 1, . . . , n, es a k erteket

P(ξ = k) =

(nk

)sk (N − s)n−k

Nn=

(n

k

)(s

N

)k (1−

s

N

)n−k=

(n

k

)pk (1− p)n−k

valoszınuseggel veszi fel. A p = sN

szam annak a valoszınusege, hogy egy termek selejtes.

Tetel 49

Ha egy A esemeny bekovetkezesre n megfigyelest vegzunk, ugy, hogy a kıserletek egymastol fuggetlenek, akkor az,hogy hanyszor kovetkezik be az A esemeny, binomialis eloszlasu valoszınusegi valtozo n, p = P(A) parameterrel.

Tetel 50

Az n, sN

parameteru binomialis eloszlasu valoszınusegi valtozo eseten

1 M(ξ) = sN· n = pn.

2 D2(ξ) = n sN

(1− s

N

)= np(1− p).

Horvath Gabor Analızis 2

Page 227: Anal zis 2 - Széchenyi István Universityhorvathg/bsc/gazdmath2.pdfHa f es g gra konja, ak ar t obbsz or is, atmeszi egym ast [a;b]-n, az a-ban es b-ben all tott fugg} oleges egyenesnek,

A Poisson eloszlasu valoszınusegi valtozo

Definıcio 66

Legyen λ pozitıv valos szam. A ξ λ parameteru Poisson eloszlasu valoszınusegi valtozo, ha lehetseges ertekei anemnegatıv egesz szamok, es

P(ξ = k) =λk

k!e−λ.

Ez a valoszınusegi valtozo tehat az eddigiekkel ellentetben vegtelen sok kulonbozo erteket vehet fel.Poisson eloszlasunak tekintheto egy ismert valoszınusegu esemeny adott idointervallum alatti bekovetkezesenek aszama, ha az ujabb bekovetkezes fuggetlen az elozotol. Peldaul, egy telefonkozpntban egy ora alatt befuto hıvasokszama (λ az egy ora alatt atlagosan befuto hıvasok szama), egy postahivatalba egy nap beerkezo levelek szama (λaz egy nap alatt atlagosan beerkezo levelek szama). De Poisson eloszlasunak tekintheto egy konyv egy oldalara esosajtohibak szama is (λ az atlagosan egy oldalra eso sajtohibak szama), egy szovogep adott, rogzıtett meretu,szovetdarabra eso szovesi hibainak szama (λ az atlagosan egy darabra eso szovesi hibak szama).

Tetel 51

Legyen ξ λ parameteru Poisson eloszlasu valoszınusegi valtozo. Ekkor

1 M(ξ) = λ.

2 D2(ξ) = λ.

Horvath Gabor Analızis 2

Page 228: Anal zis 2 - Széchenyi István Universityhorvathg/bsc/gazdmath2.pdfHa f es g gra konja, ak ar t obbsz or is, atmeszi egym ast [a;b]-n, az a-ban es b-ben all tott fugg} oleges egyenesnek,

Feladat 62

Egy bobozban ot piros es nyolc feher golyo van. Kiveszunk egyszerre ot golyot. Mi a valoszınusege annak, hogyparatlan szamu piros golyot veszunk ki.

Megoldas: Jelolje ξ kivett golyok kozott levo piros golyok szamat. ξ lehetseges ertekei: 0, 1, 2, 3, 4, 5. Ez a ξhipergeometriai eloszlasu valoszınusegi valtozo, most N = 13, s = 5, es mivel ot golyot veszunk ki, n = 5.

p1 = P(ξ = 1) =

(51

)·(

84

)(

135

) = 3501287

.

p3 = P(ξ = 3) =

(53

)·(

82

)(

135

) = 2801287

.

p5 = P(ξ = 5) =

(55

)·(

80

)(

135

) = 11287

.

Ezert, ha A jeloli azt az esemenyt, hogy paratlan szamu piros golyot huzunk, akkor

P(A) = P(ξ = 1 + ξ = 3 + ξ = 5) = P(ξ = 1) + P(ξ = 3) + P(ξ = 5) =631

1287.

Horvath Gabor Analızis 2

Page 229: Anal zis 2 - Széchenyi István Universityhorvathg/bsc/gazdmath2.pdfHa f es g gra konja, ak ar t obbsz or is, atmeszi egym ast [a;b]-n, az a-ban es b-ben all tott fugg} oleges egyenesnek,

Feladat 62

Egy bobozban ot piros es nyolc feher golyo van. Kiveszunk egyszerre ot golyot. Mi a valoszınusege annak, hogyparatlan szamu piros golyot veszunk ki.Megoldas: Jelolje ξ kivett golyok kozott levo piros golyok szamat. ξ lehetseges ertekei: 0, 1, 2, 3, 4, 5. Ez a ξhipergeometriai eloszlasu valoszınusegi valtozo, most N = 13, s = 5, es mivel ot golyot veszunk ki, n = 5.

p1 = P(ξ = 1) =

(51

)·(

84

)(

135

) = 3501287

.

p3 = P(ξ = 3) =

(53

)·(

82

)(

135

) = 2801287

.

p5 = P(ξ = 5) =

(55

)·(

80

)(

135

) = 11287

.

Ezert, ha A jeloli azt az esemenyt, hogy paratlan szamu piros golyot huzunk, akkor

P(A) = P(ξ = 1 + ξ = 3 + ξ = 5) = P(ξ = 1) + P(ξ = 3) + P(ξ = 5) =631

1287.

Horvath Gabor Analızis 2

Page 230: Anal zis 2 - Széchenyi István Universityhorvathg/bsc/gazdmath2.pdfHa f es g gra konja, ak ar t obbsz or is, atmeszi egym ast [a;b]-n, az a-ban es b-ben all tott fugg} oleges egyenesnek,

Feladat 63

Feldobunk egy dobokockat hetszer egymas utan. Jelolje ξ a hatos dobasok szamat. Mennyi ξ varhato erteke?Mennyi annak a valoszınusege, hogy a ξ varhato ertekenel kevesebb hatost dobunk?

Megoldas: Feltehetjuk, hogy a kockadobasok egymastol fuggetlenek, ezert ξ 7, p = 16

parameteru binomialiseloszlasu valoszınusegi valtozo.Igy M(ξ) = 7 · 1

6= 7

6≈ 1.167.

Jelolje A azt az esemenyt, hogy a ξ varhato ertekenel kevesebb hatost dobunk.

P(A) = P(ξ = 0 + ξ = 1 = P(ξ = 0) + P(ξ = 1) =

=

(7

0

)(1

6

)0 (1−

1

6

)7

+

(7

1

)(1

6

)1 (1−

1

6

)6

=15625

23328≈ 0.67.

Horvath Gabor Analızis 2

Page 231: Anal zis 2 - Széchenyi István Universityhorvathg/bsc/gazdmath2.pdfHa f es g gra konja, ak ar t obbsz or is, atmeszi egym ast [a;b]-n, az a-ban es b-ben all tott fugg} oleges egyenesnek,

Feladat 63

Feldobunk egy dobokockat hetszer egymas utan. Jelolje ξ a hatos dobasok szamat. Mennyi ξ varhato erteke?Mennyi annak a valoszınusege, hogy a ξ varhato ertekenel kevesebb hatost dobunk?Megoldas: Feltehetjuk, hogy a kockadobasok egymastol fuggetlenek, ezert ξ 7, p = 1

6parameteru binomialis

eloszlasu valoszınusegi valtozo.Igy M(ξ) = 7 · 1

6= 7

6≈ 1.167.

Jelolje A azt az esemenyt, hogy a ξ varhato ertekenel kevesebb hatost dobunk.

P(A) = P(ξ = 0 + ξ = 1 = P(ξ = 0) + P(ξ = 1) =

=

(7

0

)(1

6

)0 (1−

1

6

)7

+

(7

1

)(1

6

)1 (1−

1

6

)6

=15625

23328≈ 0.67.

Horvath Gabor Analızis 2

Page 232: Anal zis 2 - Széchenyi István Universityhorvathg/bsc/gazdmath2.pdfHa f es g gra konja, ak ar t obbsz or is, atmeszi egym ast [a;b]-n, az a-ban es b-ben all tott fugg} oleges egyenesnek,

Feladat 64

Egy gyumolcsosben az egyes fakon talalhato gyumolcsok szama Poisson eloszlast kovet husz varhato ertekkel.Talalomra kivalasztunk egy fat. Mi a valoszınusege annak, hogy azon legalabb hat gyumolcs lesz?

Megoldas: Jelolje ξ kivalasztott fan levo gyumolcsok szamat. ξ 20 parameteru Poisson eloszlasu valoszınusegivaltozo. (Valojaban ez nem lehet igaz, de eleg jo kozelıtesnek tekintheto.)

Jelolje A azt az esemenyt, hogy a kivalasztott fan legalabb hat gyumolcs van. Ekkor A azt jelenti, hogy akivalasztott fan legfeljebb ot gyumolcs van.

P(A) = P(ξ = 0 + ξ = 1 + ξ = 2 + ξ = 3 + ξ = 4 + ξ = 5) =

= P(ξ = 0) + P(ξ = 1) + P(ξ = 2) + P(ξ = 3) + P(ξ = 4) + P(ξ = 5) =

=200

0!e−20 +

201

1!e−20 +

202

2!e−20 +

203

3!e−20 +

204

4!e−20 +

205

5!e−20 =

104663

3e−20 ≈

≈ 0.00007190884052.

Tehat P(A) = 1− 0.00007190884052 = 0.9999280912.

Horvath Gabor Analızis 2

Page 233: Anal zis 2 - Széchenyi István Universityhorvathg/bsc/gazdmath2.pdfHa f es g gra konja, ak ar t obbsz or is, atmeszi egym ast [a;b]-n, az a-ban es b-ben all tott fugg} oleges egyenesnek,

Feladat 64

Egy gyumolcsosben az egyes fakon talalhato gyumolcsok szama Poisson eloszlast kovet husz varhato ertekkel.Talalomra kivalasztunk egy fat. Mi a valoszınusege annak, hogy azon legalabb hat gyumolcs lesz?Megoldas: Jelolje ξ kivalasztott fan levo gyumolcsok szamat. ξ 20 parameteru Poisson eloszlasu valoszınusegivaltozo. (Valojaban ez nem lehet igaz, de eleg jo kozelıtesnek tekintheto.)

Jelolje A azt az esemenyt, hogy a kivalasztott fan legalabb hat gyumolcs van. Ekkor A azt jelenti, hogy akivalasztott fan legfeljebb ot gyumolcs van.

P(A) = P(ξ = 0 + ξ = 1 + ξ = 2 + ξ = 3 + ξ = 4 + ξ = 5) =

= P(ξ = 0) + P(ξ = 1) + P(ξ = 2) + P(ξ = 3) + P(ξ = 4) + P(ξ = 5) =

=200

0!e−20 +

201

1!e−20 +

202

2!e−20 +

203

3!e−20 +

204

4!e−20 +

205

5!e−20 =

104663

3e−20 ≈

≈ 0.00007190884052.

Tehat P(A) = 1− 0.00007190884052 = 0.9999280912.

Horvath Gabor Analızis 2

Page 234: Anal zis 2 - Széchenyi István Universityhorvathg/bsc/gazdmath2.pdfHa f es g gra konja, ak ar t obbsz or is, atmeszi egym ast [a;b]-n, az a-ban es b-ben all tott fugg} oleges egyenesnek,

Folytonos valoszınusegi valtozok

Definıcio 67

Adott egy (Ω,A,P) Kolmogorov fele valoszınusegi mezo. Aξ : Ω −→ R fuggveny folytonos valoszınusegi valtozo, ha az Rξertekkeszlete tartalmaz intervallumot, es minden x ∈ Rξ esetenAx = ξ < x = ω ∈ Ω|ξ(ω) < x ∈ A.

Ez a folytonossag mast jelent, mint valos fuggvenyek korebendefinialt folytonossag. Folytonos valoszınusegi valtozot csak olyanΩ-n lehet definialni, amelynek a szamossaga vegtelen, de nemmegszamlalhatoan vegtelen.Mostantol ha csak egyszeruen valoszınusegi valtozorol beszelunk,akkor amit mondunk az egyarant igaz diszkret es folytonosvaloszınusegi valtozokra is.

Horvath Gabor Analızis 2

Page 235: Anal zis 2 - Széchenyi István Universityhorvathg/bsc/gazdmath2.pdfHa f es g gra konja, ak ar t obbsz or is, atmeszi egym ast [a;b]-n, az a-ban es b-ben all tott fugg} oleges egyenesnek,

Az eloszlasfuggveny

Definıcio 68

Legyen ξ valoszınusegi valtozo (Ω,A,P)-n. A ξvaloszınusegi valtozoFξ : R −→ R eloszlasfuggvenye az az egyvaltozos valos fuggveny, amelyreminden x ∈ R eseten

Fξ(x) = P(ξ < x) = P(ξ < x),

ahol persze ξ < x = ω ∈ Ω|ξ(ω) < x.

Tetel 52

A ξ valoszınusegi valtozo Fξ eloszlasfuggvenye rendelkezik az alabbitulajdonsagokkal:

1 0 ≤ Fξ(x) ≤ 1 minden x ∈ R eseten.

2 Fξ monoton novo fuggveny.

3 Fξ mindenhol balrol folytonos.

4 limx→−∞

Fξ(x) = 0, limx→∞

Fξ(x) = 1.

Horvath Gabor Analızis 2

Page 236: Anal zis 2 - Széchenyi István Universityhorvathg/bsc/gazdmath2.pdfHa f es g gra konja, ak ar t obbsz or is, atmeszi egym ast [a;b]-n, az a-ban es b-ben all tott fugg} oleges egyenesnek,

A surusegfuggveny

Tetel 53

1 Ha Fξ a ξ valoszınusegi valtozo eloszlasfuggvenye, a, b ∈ R es a < b, akkor

P(a ≤ ξ < b) = Fξ(b)− Fξ(a).

2 Ha Fξ folytonos az a helyen, akkor P(ξ = a) = 0.

3 P(ξ = a) = limx→b+

Fξ(x)− limx→a−

Fξ(x) = limx→b+

Fξ(x)− Fξ(a).

4 Ha Fξ folytonos az a es a b helyen, akkor

P(a ≤ ξ < b) = P(a ≤ ξ ≤ b) = P(a < ξ ≤ b) = P(a < ξ < b) = Fξ(b)− Fξ(a).

Eloszlasfuggvenye minden valoszınusegi valtozonak van. Most egy ujabb fuggvenyt rendelunk hozza bizonyosvaloszınusegi valtozokhoz, ami a kesobbiekben nagyon hasznos lesz.

Definıcio 69

Legyen a ξ valoszınusegi valtozo eloszlasfuggvenye Fξ . Ha letezik olyan f : R −→ R fuggveny, hogy mindenx ∈ R eseten

Fξ(x) =

x∫−∞

f (t) dt,

akkor az f fuggvenyt a ξ surusegfuggvenyenek hıvjuk.

Surusegfuggvenye nem minden valoszınusegi valtozonak letezik. A diszkret valoszınusegi valtozoknak nincssurusegfuggvenyuk. Azt is meg lehet engedni, hogy a surusegfuggveny megszamlalhato sok x-re ne is legyenertelmezve.

Horvath Gabor Analızis 2

Page 237: Anal zis 2 - Széchenyi István Universityhorvathg/bsc/gazdmath2.pdfHa f es g gra konja, ak ar t obbsz or is, atmeszi egym ast [a;b]-n, az a-ban es b-ben all tott fugg} oleges egyenesnek,

A surusegfuggveny

Tetel 54

1 Ha letezik surusegfuggveny, akkor f (x) = F ′(x) legfeljebbmegszamlalhato sok x kivetelevel.

2 f (x) ≥ 0, azaz a surusegfuggveny nem negatıv.

3

∞∫−∞

f (x) dx = 1.

4 P(a ≤ ξ < b) = P(a ≤ ξ ≤ b) = P(a < ξ ≤ b) = P(a < ξ < b) =b∫a

f (x) dx.

5 P(ξ < b) = P(ξ ≤ b) =b∫−∞

f (x) dx, es P(a ≤ ξ) = P(a < ξ) =∞∫a

f (x) dx.

Tetel 55

Legyen A veges vagy vegtelen intervallumok tetszoleges diszjunkt unioja.Ekkor

P(ξ ∈ A) =

∫A

f (x) dx .

Horvath Gabor Analızis 2

Page 238: Anal zis 2 - Széchenyi István Universityhorvathg/bsc/gazdmath2.pdfHa f es g gra konja, ak ar t obbsz or is, atmeszi egym ast [a;b]-n, az a-ban es b-ben all tott fugg} oleges egyenesnek,

A varhato ertek

Definıcio 70

Legyen a ξ valoszınusegi valtozo surusegfuggvenye f (x). Ekkor a ξ M(ξ) varhato erteke

M(ξ) =

∞∫−∞

x · f (x) dx,

ha ez az improprius integral letezik es veges. Egyebkent ξ-nek nincs varhato erteke.

Diszkret valoszınusegi valtozonak nincs surusegfuggvenye, ezert abban az esetben a definıcio a regi.

Tetel 56

Ha letezik az M(ξ) varhato ertek, akkor

M(ξ) =

∞∫0

(1− Fξ(x)) dx −0∫

−∞

Fξ(x) dx.

Tetel 57

Ha g : R −→ R folytonos fuggveny, es letezik az M(g(ξ)) varhato ertek, akkor letezik az M(ξ) varhato ertek ises, ha f a ξ valoszınusegi valtozo surusegfuggvenye, akkor

M(g(ξ)) =

∞∫−∞

g(x) · f (x) dx.

Horvath Gabor Analızis 2

Page 239: Anal zis 2 - Széchenyi István Universityhorvathg/bsc/gazdmath2.pdfHa f es g gra konja, ak ar t obbsz or is, atmeszi egym ast [a;b]-n, az a-ban es b-ben all tott fugg} oleges egyenesnek,

A szoras

Definıcio 71

Legyen ξ valoszınusegi valtozo (Ω,A,P)-n. A ξ valoszınusegivaltozo szorasnegyzete

D2(ξ) = M((ξ −M(ξ))2),

ha ez az improprius integral letezik es veges. Egyebkent ξ-neknincs szoresnegyzete. ξ szorasa D(ξ) =

√D2(ξ).

Tetel 58

1 Ha letezik M(ξ) es M(ξ2), akkor D2(ξ) = M(ξ2)− (M(ξ))2.

2 Ha ξ1, ξ2, . . . , ξn paronkent fuggetlenek, c1, c2, . . . , cn ∈ R,akkor

D2(c1ξ1+c2ξ2+. . .+cnξn) = c21 D2(ξ1)+c2

2 D2(ξ2)+. . .+c2n D2(ξn).

Horvath Gabor Analızis 2

Page 240: Anal zis 2 - Széchenyi István Universityhorvathg/bsc/gazdmath2.pdfHa f es g gra konja, ak ar t obbsz or is, atmeszi egym ast [a;b]-n, az a-ban es b-ben all tott fugg} oleges egyenesnek,

Feladat 65

Tekintsuk az

f (x) =

x, ha 0 < x ≤ 1,2− x, ha 1 < x < 2,0, maskor.

Igazoljuk, hogy ez surusegfuggveny. Hatarozzuk meg az eloszlasfuggvenyt es szamoljuk ki a varhato erteket.

Megoldas: Eloszor is felrajzoljuk f grafikonjat, ez lathato alabbi abran.

Egy fuggveny akkor lehet egy ξ valoszınusegi valtozo surusegfuggvenye, ha nem negatıv, es a fuggvenygorbe alattiterulet 1. Az abrarol latszik, hogy f nem negatıv. Mivel f grafikonja egyenes szakaszokbol all, a gorbe alatti teruletkiszamolasahoz nem kell integralszamıtast hasznalnunk, vilagos, hogy az abran lathato haromszog terulete 1. Tehatf surusegfuggveny.A grafikon harom torespontja negy reszre bontja a szamegyenest. Meghatarozzuk az egyes reszekbe eso x esetenFξ(x) erteket.

Ha x ≤ 0, akkor Fξ(x) =x∫−∞

f (t) dt =x∫−∞

0 dt = 0.

Ha 0 < x ≤ 1, akkor Fξ(x) =x∫−∞

f (t) dt =0∫−∞

f (t) dt +x∫0

f (t) dt = 0 +x∫0

t dt =

[t2

2

]x

0= x2

2.

Ha 1 < x ≤ 2, akkor Fξ(x) =x∫−∞

f (t) dt =0∫−∞

f (t) dt +1∫

0f (t) dt +

x∫1

f (t) dt =

0 + 12

+x∫1

(2− t) dt = 12

+

[2t − t2

2

]x

1= 2x − x2

2− 1.

Horvath Gabor Analızis 2

Page 241: Anal zis 2 - Széchenyi István Universityhorvathg/bsc/gazdmath2.pdfHa f es g gra konja, ak ar t obbsz or is, atmeszi egym ast [a;b]-n, az a-ban es b-ben all tott fugg} oleges egyenesnek,

Feladat 65

Tekintsuk az

f (x) =

x, ha 0 < x ≤ 1,2− x, ha 1 < x < 2,0, maskor.

Igazoljuk, hogy ez surusegfuggveny. Hatarozzuk meg az eloszlasfuggvenyt es szamoljuk ki a varhato erteket.Megoldas: Eloszor is felrajzoljuk f grafikonjat, ez lathato alabbi abran.

Egy fuggveny akkor lehet egy ξ valoszınusegi valtozo surusegfuggvenye, ha nem negatıv, es a fuggvenygorbe alattiterulet 1. Az abrarol latszik, hogy f nem negatıv. Mivel f grafikonja egyenes szakaszokbol all, a gorbe alatti teruletkiszamolasahoz nem kell integralszamıtast hasznalnunk, vilagos, hogy az abran lathato haromszog terulete 1. Tehatf surusegfuggveny.A grafikon harom torespontja negy reszre bontja a szamegyenest. Meghatarozzuk az egyes reszekbe eso x esetenFξ(x) erteket.

Ha x ≤ 0, akkor Fξ(x) =x∫−∞

f (t) dt =x∫−∞

0 dt = 0.

Ha 0 < x ≤ 1, akkor Fξ(x) =x∫−∞

f (t) dt =0∫−∞

f (t) dt +x∫0

f (t) dt = 0 +x∫0

t dt =

[t2

2

]x

0= x2

2.

Ha 1 < x ≤ 2, akkor Fξ(x) =x∫−∞

f (t) dt =0∫−∞

f (t) dt +1∫

0f (t) dt +

x∫1

f (t) dt =

0 + 12

+x∫1

(2− t) dt = 12

+

[2t − t2

2

]x

1= 2x − x2

2− 1.

Horvath Gabor Analızis 2

Page 242: Anal zis 2 - Széchenyi István Universityhorvathg/bsc/gazdmath2.pdfHa f es g gra konja, ak ar t obbsz or is, atmeszi egym ast [a;b]-n, az a-ban es b-ben all tott fugg} oleges egyenesnek,

Ha 2 < x , akkor Fξ(x) =x∫−∞

f (t) dt = 1 +x∫2

f (t) dt = 1 +x∫2

0 dt = 1.

Tehat

Fξ(x) =

0, ha x ≤ 0,x2

2, ha 0 < x ≤ 1,

2x − x2

2− 1, ha 1 < x ≤ 2,

1, ha 2 < x.

Fξ grafikonja lathato a kovetkezo abran.

Ezek utan a varhato ertek

M(ξ) =

∞∫−∞

x · f (x) dx =

0∫−∞

x · f (x) dx +

1∫0

x · f (x) dx +

2∫1

x · f (x) dx +

∞∫2

x · f (x) dx =

= 0 +

1∫0

x · x dx +

2∫1

x · (2− x) dx + 0 =

[x3

3

]1

0

+

[x2 −

x3

3

]2

1

=1

3+

(4−

8

3

)−(

1−1

3

)= 1.

Horvath Gabor Analızis 2

Page 243: Anal zis 2 - Széchenyi István Universityhorvathg/bsc/gazdmath2.pdfHa f es g gra konja, ak ar t obbsz or is, atmeszi egym ast [a;b]-n, az a-ban es b-ben all tott fugg} oleges egyenesnek,

Feladat 66

Egy ξ valoszınusegi valtozo eloszlasfuggvenye

Fξ(x) =

0, ha x ≤ 0,

2x − x2, ha 0 < x ≤ 1,1, ha 1 < x.

Hatarozzuk meg ξ surusegfuggvenyet. Szamoljuk ki a P(ξ ≥ 1/3) valoszınuseget es ξ szorasat.

Megoldas: Az eloszlasfuggveny grafikonja lathato bal oldali abran.

A grafikonrol latszik, hogy ez a fuggveny egyedul a nullaban nem derivalhato. Mivel Fξ(x) szakaszonkent definialt,a derivaltjat is szakaszonkent hatarozzuk meg.

Ha x < 0, akkor F ′ξ(x) = f (x) = 0.

Ha 0 < x < 1, akkor F ′ξ(x) = f (x) = 2− 2x.

Ha 1 < x, akkor F ′ξ(x) = f (x) = 0.

Tehat

f (x) =

0, ha x < 0,2− 2x, ha 0 < x < 1,0, ha 1 < x.

A 0 es az 1 helyeken nem definialjuk a surusegfuggvenyt. (Ha barhogy definialnank, peldaul nullanak, az is jolenne.) A surusegfuggveny grafikonja lathato jobb oldali abran.

Horvath Gabor Analızis 2

Page 244: Anal zis 2 - Széchenyi István Universityhorvathg/bsc/gazdmath2.pdfHa f es g gra konja, ak ar t obbsz or is, atmeszi egym ast [a;b]-n, az a-ban es b-ben all tott fugg} oleges egyenesnek,

Feladat 66

Egy ξ valoszınusegi valtozo eloszlasfuggvenye

Fξ(x) =

0, ha x ≤ 0,

2x − x2, ha 0 < x ≤ 1,1, ha 1 < x.

Hatarozzuk meg ξ surusegfuggvenyet. Szamoljuk ki a P(ξ ≥ 1/3) valoszınuseget es ξ szorasat.Megoldas: Az eloszlasfuggveny grafikonja lathato bal oldali abran.

A grafikonrol latszik, hogy ez a fuggveny egyedul a nullaban nem derivalhato. Mivel Fξ(x) szakaszonkent definialt,a derivaltjat is szakaszonkent hatarozzuk meg.

Ha x < 0, akkor F ′ξ(x) = f (x) = 0.

Ha 0 < x < 1, akkor F ′ξ(x) = f (x) = 2− 2x.

Ha 1 < x, akkor F ′ξ(x) = f (x) = 0.

Tehat

f (x) =

0, ha x < 0,2− 2x, ha 0 < x < 1,0, ha 1 < x.

A 0 es az 1 helyeken nem definialjuk a surusegfuggvenyt. (Ha barhogy definialnank, peldaul nullanak, az is jolenne.) A surusegfuggveny grafikonja lathato jobb oldali abran.

Horvath Gabor Analızis 2

Page 245: Anal zis 2 - Széchenyi István Universityhorvathg/bsc/gazdmath2.pdfHa f es g gra konja, ak ar t obbsz or is, atmeszi egym ast [a;b]-n, az a-ban es b-ben all tott fugg} oleges egyenesnek,

Egy intervallumba eses valoszınusege a surusegfuggveny intervallum feletti integralja, tehat

P(ξ ≥ 1/3) =

∞∫1/3

f (x) dx =

1∫1/3

(2− 2x) dx =[

2x − x2]1

1/3=

4

9.

ξ szorasanak meghatarozasahoz eloszor kiszamoljuk ξ varhato erteket.

M(ξ) =

∞∫−∞

x · f (x) dx =

1∫0

x(2− 2x) dx =

[x2 −

2x3

3

]1

0

=1

3.

Mivel a D2(ξ) = M(ξ2)− (M(ξ))2 kepletet akarjuk hasznalni, szuksegunk van M(ξ2) ertekere. De avaloszınusegi valtozo fuggvenyenek varhato ertekere vonatkozo keplet alapjan

M(ξ2) =

∞∫−∞

x2 · f (x) dx =

1∫0

x2(2− 2x) dx =

[2x3

3−

2x4

4

]1

0

=1

6.

Igy D2(ξ) = 16− 1

9= 1

18, azaz D(ξ) = 1

3√

2.

Horvath Gabor Analızis 2

Page 246: Anal zis 2 - Széchenyi István Universityhorvathg/bsc/gazdmath2.pdfHa f es g gra konja, ak ar t obbsz or is, atmeszi egym ast [a;b]-n, az a-ban es b-ben all tott fugg} oleges egyenesnek,

Nevezetes folytonos valoszınusegi valtozok

Definıcio 72

A ξ valoszınusegi valtozo egyenletes eloszlasu az (a,b) intervallumon, ha surusegfuggvenye

f (x) =

0, ha −∞ < x < a,1

b−a, ha a < x < b,

0, ha b < x <∞.

Tetel 59

Ha ξ egyenletes eloszlasu az (a, b) intervallumon, akkor

1 Fξ(x) =

0, ha x ≤ a,

x−ab−a

, ha a < x ≤ b,

1, ha b < x.

2 M(ξ) = a+b2

.

3 D2(ξ) =(b−a)2

12.

Egyenletes eloszlas eseten annak a valoszınusege, hogy a ξ az (a, b) intervallum egy adott hosszusagureszintervallumara esik fuggetlen a reszintervallum pozıciojatol, csak a hosszatol fugg, (ha teljes egeszeben(a, b)-ben van). Ezt teteleztuk fel egydimenzios geometria valoszınuseges feladatokban, mikor azt mondtuk, hogytalalomra valasztunk egy pontot egy intervallumbol.

Horvath Gabor Analızis 2

Page 247: Anal zis 2 - Széchenyi István Universityhorvathg/bsc/gazdmath2.pdfHa f es g gra konja, ak ar t obbsz or is, atmeszi egym ast [a;b]-n, az a-ban es b-ben all tott fugg} oleges egyenesnek,

Nevezetes folytonos valoszınusegi valtozok

Definıcio 73

Legyen λ > 0 valos szam. A ξ valoszınusegi valtozo λ parameteru exponencialis eloszlasu valoszınusegi valtozo, hasurusegfuggvenye

f (x) =

0, ha x < 0,

λe−λx , ha 0 < x.

Tetel 60

Ha ξ λ parameteru exponencialis eloszlasu valoszınusegi valtozo, akkor

1 Fξ(x) =

0, ha x ≤ 0,

1− e−λx , ha 0 < x.

2 M(ξ) = 1λ

.

3 D2(ξ) = 1λ2 .

Exponencialis eloszlasu valoszınusegi valtozo lep fel varakozasi idok megfigyelesekor: ha tudjuk, hogy egy adottidointervallum alatt atlagosan hanyszor kovetkezik be egy esemeny, (1/λ-szor), akkor ket bekovetkezes kozotteltelt ido λ parameteru exponencialis eloszlasu.

Horvath Gabor Analızis 2

Page 248: Anal zis 2 - Széchenyi István Universityhorvathg/bsc/gazdmath2.pdfHa f es g gra konja, ak ar t obbsz or is, atmeszi egym ast [a;b]-n, az a-ban es b-ben all tott fugg} oleges egyenesnek,

A normalis eloszlasu valoszınusegi valtozo

Definıcio 74

Adott egy (Ω,A,P) Kolmogorov fele valoszınusegi mezo. Aξ : Ω −→ R valoszınusegi valtozo (m, σ) parameteru normaliseloszlasu valoszınusegi valtozo, ha m ∈ R, σ > 0, es ξsurusegfuggvenye

f (x) =1

σ√

2πe−

(x−m)2

2σ2 .

Ekkor ξ eloszlasfuggvenye

F (x) =1

σ√

x∫−∞

e−(t−m)2

2σ2 dt.

Horvath Gabor Analızis 2

Page 249: Anal zis 2 - Széchenyi István Universityhorvathg/bsc/gazdmath2.pdfHa f es g gra konja, ak ar t obbsz or is, atmeszi egym ast [a;b]-n, az a-ban es b-ben all tott fugg} oleges egyenesnek,

A standard normalis eloszlasu valoszınusegi valtozo

Definıcio 75

A (0, 1) parameteru normalis eloszlasu valoszınusegi valtozot standard normalis eloszlasu valoszınusegi valtozonakhıvjuk. Ennek surusegfuggvenye tehat

ϕ(x) =1√

2πe− x2

2 ,

eloszlasfuggvenye

Φ(x) =1√

x∫−∞

e− t2

2 dt.

Horvath Gabor Analızis 2

Page 250: Anal zis 2 - Széchenyi István Universityhorvathg/bsc/gazdmath2.pdfHa f es g gra konja, ak ar t obbsz or is, atmeszi egym ast [a;b]-n, az a-ban es b-ben all tott fugg} oleges egyenesnek,

A standard normalis eloszlasu valoszınusegi valtozo

Tetel 61

A ϕ fuggveny rendelkezik az alabbi tulajdonsagokkal:

1 ϕ paros fuggveny.

2 limx→−∞

ϕ(x) = limx→∞

ϕ(x) = 0.

3 ϕ-nek a nullaban globalis maximuma van, amelynek erteke1√2π

.

4 ϕ-nek −1-ben es 1-ben inflexios pontja van.

Tetel 62

Ha ξ (m, σ) parameteru normalis eloszlasu valoszınusegi valtozo,akkor

1 M(ξ) = m.

2 D2(ξ) = σ2.

A standard normalis eloszlasu valoszınusegi valtozonak tehat nulla a varhato erteke es egy a szorasa.

Horvath Gabor Analızis 2

Page 251: Anal zis 2 - Széchenyi István Universityhorvathg/bsc/gazdmath2.pdfHa f es g gra konja, ak ar t obbsz or is, atmeszi egym ast [a;b]-n, az a-ban es b-ben all tott fugg} oleges egyenesnek,

A standardizalt

Definıcio 76

A ξ valoszınusegi valtozo ξ∗ standardizaltja a

ξ∗ =ξ −M(ξ)

D(ξ)

valoszınusegi valtozo, ha van varhato ertek es szoras, egyebbkentnincs standardizalt.

Tetel 63

Ha ξ (m, σ) parameteru normalis eloszlasu valoszınusegi valtozo,akkor standardizaltja (0, 1) parameteru standard normalis eloszlasuvaloszınusegi valtozo.

Horvath Gabor Analızis 2

Page 252: Anal zis 2 - Széchenyi István Universityhorvathg/bsc/gazdmath2.pdfHa f es g gra konja, ak ar t obbsz or is, atmeszi egym ast [a;b]-n, az a-ban es b-ben all tott fugg} oleges egyenesnek,

Tetel 64

Ha az ξ (m, σ) parameteru normalis eloszlasu valoszınusegi valtozoeloszlasfuggvenye F , akkor

1 F (x) = Φ(

x−mσ

).

2 f (x) = F ′(x) = 1σϕ(

x−mσ

).

Tetel 65

Φ(−x) = 1− Φ(x).

Horvath Gabor Analızis 2

Page 253: Anal zis 2 - Széchenyi István Universityhorvathg/bsc/gazdmath2.pdfHa f es g gra konja, ak ar t obbsz or is, atmeszi egym ast [a;b]-n, az a-ban es b-ben all tott fugg} oleges egyenesnek,

Sem az (m, σ) parameteru, sem a standard normalis eloszlasuvaloszınusegi valtozo eloszlasfuggvenyeben szereplo integral nemszamolhato ki az elemi fuggvenyekkel, az intergandusoknak nincselemi fuggvenyekkel kifejezheto primitıv fuggvenyuk.Emiatt ezeknek az eloszlasfuggvenyeknek tablazatokban kell tarolniaz ertekeit.Az elozo ket tetel miatt eleg csak a standard normalis eloszlasuvaloszınusegi valtozora tablazatot keszıteni, es azt is csak pozitıv xertekekre.Ezt a tablazatot tartalmazza a standard normalis eloszlas.doc fajl.Kerem, hogy ezt mindenki nyomtassa ki es montantol az orakrahozza magaval.Ezt a tablazatot a vizsgakra is mindenki hozza magaval!

Horvath Gabor Analızis 2

Page 254: Anal zis 2 - Széchenyi István Universityhorvathg/bsc/gazdmath2.pdfHa f es g gra konja, ak ar t obbsz or is, atmeszi egym ast [a;b]-n, az a-ban es b-ben all tott fugg} oleges egyenesnek,

Tetel 66

Ha az ξ (m, σ) parameteru normalis eloszlasu valoszınusegivaltozo, akkor

P(|ξ −m| < kσ) = 2Φ(k)− 1.

Ebbol k = 2 es k = 3 eseten

P(|ξ −m| < 2σ) = 2Φ(2)− 1 ≈ 0.954,

P(|ξ −m| < 3σ) = 2Φ(3)− 1 ≈ 0.997.

Ezeket hıvjak 2σ es 3σ szabalynak.

Horvath Gabor Analızis 2

Page 255: Anal zis 2 - Széchenyi István Universityhorvathg/bsc/gazdmath2.pdfHa f es g gra konja, ak ar t obbsz or is, atmeszi egym ast [a;b]-n, az a-ban es b-ben all tott fugg} oleges egyenesnek,

Feladat 67

Legyen ξ egyenletes eloszlasu a [−1, 2] intervallumon. Hatarozzuk meg η = 2ξ surusegfuggvenyet.

Megoldas: A surusegfuggvenyt az eloszlasfuggvenybol tudjuk meghatarozni, derivalassal. Ezert eloszormeghatarozzuk η G eloszlasfuggvenyet.Eloszor is η a [−2, 4] intervallumbol veszi fel az ertekeit. Tehat, ha x ≤ −2, akkor G(x) = 0, ha x > 4, akkorG(x) = 1. Legyen ezutan −2 < x ≤ 4. Ekkor

G(x) = P(η < x) = P(2ξ < x) = P(ξ <x

2).

Jeloljuk ξ eloszlasfuggveny F -el. Tudjuk, hogy

F (x) =

0, ha x ≤ −1,

x+13, ha −1 < x ≤ 2,

1, ha 2 < x.

Ha −2 < x ≤ 4, akkor −1 < x/2 ≤ 2, es ıgy G(x) = P(ξ < x2

) = F (x/2) =x2

+1

3= x+2

6. Tehat

G(x) =

0, ha x ≤ −2,

x+26, ha −2 < x ≤ 4,

1, ha 4 < x.

Azaz η egyenletes eloszlasu a [−2, 4] intervallumon. Innen a g surusegfuggvenye

g(x) =

0, ha x < −2,16, ha −2 < x < 4,

0, ha 4 < x.

Horvath Gabor Analızis 2

Page 256: Anal zis 2 - Széchenyi István Universityhorvathg/bsc/gazdmath2.pdfHa f es g gra konja, ak ar t obbsz or is, atmeszi egym ast [a;b]-n, az a-ban es b-ben all tott fugg} oleges egyenesnek,

Feladat 67

Legyen ξ egyenletes eloszlasu a [−1, 2] intervallumon. Hatarozzuk meg η = 2ξ surusegfuggvenyet.Megoldas: A surusegfuggvenyt az eloszlasfuggvenybol tudjuk meghatarozni, derivalassal. Ezert eloszormeghatarozzuk η G eloszlasfuggvenyet.Eloszor is η a [−2, 4] intervallumbol veszi fel az ertekeit. Tehat, ha x ≤ −2, akkor G(x) = 0, ha x > 4, akkorG(x) = 1. Legyen ezutan −2 < x ≤ 4. Ekkor

G(x) = P(η < x) = P(2ξ < x) = P(ξ <x

2).

Jeloljuk ξ eloszlasfuggveny F -el. Tudjuk, hogy

F (x) =

0, ha x ≤ −1,

x+13, ha −1 < x ≤ 2,

1, ha 2 < x.

Ha −2 < x ≤ 4, akkor −1 < x/2 ≤ 2, es ıgy G(x) = P(ξ < x2

) = F (x/2) =x2

+1

3= x+2

6. Tehat

G(x) =

0, ha x ≤ −2,

x+26, ha −2 < x ≤ 4,

1, ha 4 < x.

Azaz η egyenletes eloszlasu a [−2, 4] intervallumon. Innen a g surusegfuggvenye

g(x) =

0, ha x < −2,16, ha −2 < x < 4,

0, ha 4 < x.

Horvath Gabor Analızis 2

Page 257: Anal zis 2 - Széchenyi István Universityhorvathg/bsc/gazdmath2.pdfHa f es g gra konja, ak ar t obbsz or is, atmeszi egym ast [a;b]-n, az a-ban es b-ben all tott fugg} oleges egyenesnek,

Feladat 68

Egy fontos telefonhıvasra varunk. A hıvas egy 8 oraval kezdodo ismeretlen hosszusagu idointervallumban fut beegyenletes eloszlas szerint. Tudjuk, hogy a varakozasi idonk varhato erteke 2 ora. Mi a valoszınusege annak, hogy ahıvas 11 ora elott megtortenik?

Megoldas: Jeloljuk ξ-vel a telefonhıvas idopontjat, F -el ξ eloszlasfuggvenyet. Tudjuk, hogy ξ egyenletes eloszlasua [8, b] intervallumon, de nem ismerjuk b-t.

F (x) =

0, ha x ≤ 8,

x−8b−8

, ha 8 < x ≤ b,

1, ha b < x.

G(x) =

0, ha x ≤ 0,x

b−8, ha 0 < x ≤ b − 8,

1, ha b − 8 < x.

A varakozasi idonk η = ξ − 8. Ez az η egyenletes eloszlasu a [0, b − 8] intervallumon. Valoban, ha G jeloli azeloszlasfuggvenyet, akkor x ≤ 0 eseten G(x) = P(η < x) = P(ξ − 8 < x) = P(ξ < x + 8) = F (x + 8) = 0,hiszen ilyenkor x + 8 ≤ 8; 0 < x ≤ b − 8 esetenG(x) = P(η < x) = P(ξ− 8 < x) = P(ξ < x + 8) = F (x + 8) = x

b−8, hiszen most x + 8 a 8 es a b koze esik;

vegul b − 8 < x eseten G(x) = P(η < x) = P(ξ − 8 < x) = P(ξ < x + 8) = F (x + 8) = 1, ugyanis mostb < x + 8.

Ez alapjan M(η) = 2 =0+(b−8)

2, amibol b = 12. Tehat ξ egyenletes eloszlasu a [8, 12] intervallumon. Ezert, ha

f jeloli a surusegfuggvenyet, akkor

f (x) =

0, ha x < 8,14, ha 8 < x < 12,

0, ha 12 < x.

Mostmar

P(ξ < 11) =

11∫−∞

f (x) dx =

11∫8

1

4dx =

[x

4

]11

8=

3

4.

Horvath Gabor Analızis 2

Page 258: Anal zis 2 - Széchenyi István Universityhorvathg/bsc/gazdmath2.pdfHa f es g gra konja, ak ar t obbsz or is, atmeszi egym ast [a;b]-n, az a-ban es b-ben all tott fugg} oleges egyenesnek,

Feladat 68

Egy fontos telefonhıvasra varunk. A hıvas egy 8 oraval kezdodo ismeretlen hosszusagu idointervallumban fut beegyenletes eloszlas szerint. Tudjuk, hogy a varakozasi idonk varhato erteke 2 ora. Mi a valoszınusege annak, hogy ahıvas 11 ora elott megtortenik?Megoldas: Jeloljuk ξ-vel a telefonhıvas idopontjat, F -el ξ eloszlasfuggvenyet. Tudjuk, hogy ξ egyenletes eloszlasua [8, b] intervallumon, de nem ismerjuk b-t.

F (x) =

0, ha x ≤ 8,

x−8b−8

, ha 8 < x ≤ b,

1, ha b < x.

G(x) =

0, ha x ≤ 0,x

b−8, ha 0 < x ≤ b − 8,

1, ha b − 8 < x.

A varakozasi idonk η = ξ − 8. Ez az η egyenletes eloszlasu a [0, b − 8] intervallumon. Valoban, ha G jeloli azeloszlasfuggvenyet, akkor x ≤ 0 eseten G(x) = P(η < x) = P(ξ − 8 < x) = P(ξ < x + 8) = F (x + 8) = 0,hiszen ilyenkor x + 8 ≤ 8; 0 < x ≤ b − 8 esetenG(x) = P(η < x) = P(ξ− 8 < x) = P(ξ < x + 8) = F (x + 8) = x

b−8, hiszen most x + 8 a 8 es a b koze esik;

vegul b − 8 < x eseten G(x) = P(η < x) = P(ξ − 8 < x) = P(ξ < x + 8) = F (x + 8) = 1, ugyanis mostb < x + 8.

Ez alapjan M(η) = 2 =0+(b−8)

2, amibol b = 12. Tehat ξ egyenletes eloszlasu a [8, 12] intervallumon. Ezert, ha

f jeloli a surusegfuggvenyet, akkor

f (x) =

0, ha x < 8,14, ha 8 < x < 12,

0, ha 12 < x.

Mostmar

P(ξ < 11) =

11∫−∞

f (x) dx =

11∫8

1

4dx =

[x

4

]11

8=

3

4.

Horvath Gabor Analızis 2

Page 259: Anal zis 2 - Széchenyi István Universityhorvathg/bsc/gazdmath2.pdfHa f es g gra konja, ak ar t obbsz or is, atmeszi egym ast [a;b]-n, az a-ban es b-ben all tott fugg} oleges egyenesnek,

Feladat 69

Egy szamıtogep hibamentes mukodesenek idotartama evekben merve λ = 1 parameteru exponencialis eloszlasuvaloszınusegi valtozo. Mennyi annak a valoszınusege, hogy a gep egy evnel tovabb mukodik? Tegyuk fel, hogy agep mar harom eve hibatlanul mukodik. Mennyi annak a valoszınusege, hogy meg legalabb egy evig mukodik?

Megoldas: Jelolje ξ a szamıtogep hibamentes mukodesenek idotartamat. Tudjuk, hogy ξ f surusegfuggvenye es Feloszlasfuggvenye

f (x) =

0, ha x < 0,

e−x , ha 0 < x,F (x) =

0, ha x ≤ 0,

1− e−x , ha 0 < x.

Amit ki kell szamolnunk az P(ξ > 1). Mivel F folytonos az 1 helyen

P(ξ ≤ 1) = P(ξ < 1) = F (1) = 1− e−1 = 1−1

e.

Tehat P(ξ > 1) = 1− P(ξ ≤ 1) = 1− (1− 1e

) = 1e≈ 0.3678794412.

A masodik kerdes egy felteteles valoszınuseg: P(ξ > 4|ξ > 3).

P(ξ > 4|ξ > 3) =P(ξ > 4ξ > 3)

P(ξ > 3)=

P(ξ > 4)

P(ξ > 3)=

1e4

1e3

=1

e≈ 0.3678794412.

Az exponencialis eloszlast emiatt a tulajdonsag miatt orokifju eloszlasnak is hıvjak.

Horvath Gabor Analızis 2

Page 260: Anal zis 2 - Széchenyi István Universityhorvathg/bsc/gazdmath2.pdfHa f es g gra konja, ak ar t obbsz or is, atmeszi egym ast [a;b]-n, az a-ban es b-ben all tott fugg} oleges egyenesnek,

Feladat 69

Egy szamıtogep hibamentes mukodesenek idotartama evekben merve λ = 1 parameteru exponencialis eloszlasuvaloszınusegi valtozo. Mennyi annak a valoszınusege, hogy a gep egy evnel tovabb mukodik? Tegyuk fel, hogy agep mar harom eve hibatlanul mukodik. Mennyi annak a valoszınusege, hogy meg legalabb egy evig mukodik?Megoldas: Jelolje ξ a szamıtogep hibamentes mukodesenek idotartamat. Tudjuk, hogy ξ f surusegfuggvenye es Feloszlasfuggvenye

f (x) =

0, ha x < 0,

e−x , ha 0 < x,F (x) =

0, ha x ≤ 0,

1− e−x , ha 0 < x.

Amit ki kell szamolnunk az P(ξ > 1). Mivel F folytonos az 1 helyen

P(ξ ≤ 1) = P(ξ < 1) = F (1) = 1− e−1 = 1−1

e.

Tehat P(ξ > 1) = 1− P(ξ ≤ 1) = 1− (1− 1e

) = 1e≈ 0.3678794412.

A masodik kerdes egy felteteles valoszınuseg: P(ξ > 4|ξ > 3).

P(ξ > 4|ξ > 3) =P(ξ > 4ξ > 3)

P(ξ > 3)=

P(ξ > 4)

P(ξ > 3)=

1e4

1e3

=1

e≈ 0.3678794412.

Az exponencialis eloszlast emiatt a tulajdonsag miatt orokifju eloszlasnak is hıvjak.

Horvath Gabor Analızis 2

Page 261: Anal zis 2 - Széchenyi István Universityhorvathg/bsc/gazdmath2.pdfHa f es g gra konja, ak ar t obbsz or is, atmeszi egym ast [a;b]-n, az a-ban es b-ben all tott fugg} oleges egyenesnek,

Feladat 70

Legyen ξ (3, 2) parameteru normalis eloszlasu valoszınusegivaltozo. Szamıtsuk ki annak a valoszınuseget, hogy ξ 2-nel nemnagyobb erteket vesz fel.

Megoldas: Mivel minden normalis eloszlasu valoszınusegi valtozoeloszlasfuggvenye mindenutt folytonos, P(ξ ≤ 2) = P(ξ < 2).Standardizaljuk a ξ-t, mivel tablazatunk csak Φ-re van.

P(ξ < 2) = P

(ξ − 3

2<

2− 3

2

)= P

(ξ∗ < −1

2

)=

= Φ

(−1

2

)= 1− Φ

(1

2

)≈ 1− 0.6915 = 0.3085.

Horvath Gabor Analızis 2

Page 262: Anal zis 2 - Széchenyi István Universityhorvathg/bsc/gazdmath2.pdfHa f es g gra konja, ak ar t obbsz or is, atmeszi egym ast [a;b]-n, az a-ban es b-ben all tott fugg} oleges egyenesnek,

Feladat 70

Legyen ξ (3, 2) parameteru normalis eloszlasu valoszınusegivaltozo. Szamıtsuk ki annak a valoszınuseget, hogy ξ 2-nel nemnagyobb erteket vesz fel.Megoldas: Mivel minden normalis eloszlasu valoszınusegi valtozoeloszlasfuggvenye mindenutt folytonos, P(ξ ≤ 2) = P(ξ < 2).Standardizaljuk a ξ-t, mivel tablazatunk csak Φ-re van.

P(ξ < 2) = P

(ξ − 3

2<

2− 3

2

)= P

(ξ∗ < −1

2

)=

= Φ

(−1

2

)= 1− Φ

(1

2

)≈ 1− 0.6915 = 0.3085.

Horvath Gabor Analızis 2

Page 263: Anal zis 2 - Széchenyi István Universityhorvathg/bsc/gazdmath2.pdfHa f es g gra konja, ak ar t obbsz or is, atmeszi egym ast [a;b]-n, az a-ban es b-ben all tott fugg} oleges egyenesnek,

Feladat 71

Legyen ξ (2, 3) parameteru normalis eloszlasu valoszınusegivaltozo. Szamıtsuk ki annak a valoszınuseget, hogy ξ −1 es 5kozotti erteket vesz fel.

Megoldas: Tudjuk, hogy P(−1 ≤ ξ < 5) = P(−1 < ξ < 5) =P(−1 ≤ ξ < 5) = P(−1 ≤ ξ ≤ 5), ezert most mindegy, hogypontosan mit is jelent az, hogy ξ −1 es 5 kozotti erteket vesz fel.

P(−1 ≤ ξ < 5) =

= P

(−1− 2

3≤ ξ −m

σ<

5− 2

3

)= P

(−1 ≤ ξ −m

σ< 1

)=

= P(−1 ≤ ξ∗ < 1) = Φ(1)− Φ(−1) =

= Φ(1)− (1− Φ(1)) = 2Φ(1)− 1 ≈ 2 · 0.8413− 1 = 0.6826.

Horvath Gabor Analızis 2

Page 264: Anal zis 2 - Széchenyi István Universityhorvathg/bsc/gazdmath2.pdfHa f es g gra konja, ak ar t obbsz or is, atmeszi egym ast [a;b]-n, az a-ban es b-ben all tott fugg} oleges egyenesnek,

Feladat 71

Legyen ξ (2, 3) parameteru normalis eloszlasu valoszınusegivaltozo. Szamıtsuk ki annak a valoszınuseget, hogy ξ −1 es 5kozotti erteket vesz fel.Megoldas: Tudjuk, hogy P(−1 ≤ ξ < 5) = P(−1 < ξ < 5) =P(−1 ≤ ξ < 5) = P(−1 ≤ ξ ≤ 5), ezert most mindegy, hogypontosan mit is jelent az, hogy ξ −1 es 5 kozotti erteket vesz fel.

P(−1 ≤ ξ < 5) =

= P

(−1− 2

3≤ ξ −m

σ<

5− 2

3

)= P

(−1 ≤ ξ −m

σ< 1

)=

= P(−1 ≤ ξ∗ < 1) = Φ(1)− Φ(−1) =

= Φ(1)− (1− Φ(1)) = 2Φ(1)− 1 ≈ 2 · 0.8413− 1 = 0.6826.

Horvath Gabor Analızis 2

Page 265: Anal zis 2 - Széchenyi István Universityhorvathg/bsc/gazdmath2.pdfHa f es g gra konja, ak ar t obbsz or is, atmeszi egym ast [a;b]-n, az a-ban es b-ben all tott fugg} oleges egyenesnek,

Feladat 72

Ha ξ olyan normalis eloszlasu valoszınusegi valtozo, amelyre P(ξ < −1.44) = 0.33 es P(ξ > 0.53) = 0.063,akkor mennyi az m es a σ erteke?

Megoldas:

P(ξ < −1.44) = P

(ξ − m

σ<−1.44− m

σ

)=

= P

(ξ∗<−1.44− m

σ

)= Φ

(−1.44− m

σ

)= 0.33.

De a tablazatban csak 0.5-nel nagyobb Φ ertekek vannak, es Φ(0) = 0.5. Ezert Φ a 0.33-at valamilyen negatıv yertekre veszi fel. Ekkor −y-ban Φ az 1− 0.33 = 0.67 erteket veszi fel. Ebbol −y = 0.44, azaz y = −0.44.

Tehat −1.44−mσ

= −0.44, kaptunk egy egyenletet a ket ismeretlenunkre. Felhasznalva a masik valoszınuseget is

0.063 = P(ξ > 0.53) = 1− P(ξ ≤ 0.53) = 1− P(ξ < 0.53) = 1− P

(ξ − m

σ<

0.53− m

σ

)=

= 1− Φ

(0.53− m

σ

),

azaz Φ(

0.53−mσ

)= 0.937, amibol 0.53−m

σ= 1.53, ez a masik egyenlet a ket ismeretlenunkre.

A ket egyenletbol m = −1 es σ = 1 adodik, tehat a (−1, 1) parameteru normalis eloszlasu valoszınusegivaltozorol szolt a feladat.

Horvath Gabor Analızis 2

Page 266: Anal zis 2 - Széchenyi István Universityhorvathg/bsc/gazdmath2.pdfHa f es g gra konja, ak ar t obbsz or is, atmeszi egym ast [a;b]-n, az a-ban es b-ben all tott fugg} oleges egyenesnek,

Feladat 72

Ha ξ olyan normalis eloszlasu valoszınusegi valtozo, amelyre P(ξ < −1.44) = 0.33 es P(ξ > 0.53) = 0.063,akkor mennyi az m es a σ erteke?Megoldas:

P(ξ < −1.44) = P

(ξ − m

σ<−1.44− m

σ

)=

= P

(ξ∗<−1.44− m

σ

)= Φ

(−1.44− m

σ

)= 0.33.

De a tablazatban csak 0.5-nel nagyobb Φ ertekek vannak, es Φ(0) = 0.5. Ezert Φ a 0.33-at valamilyen negatıv yertekre veszi fel. Ekkor −y-ban Φ az 1− 0.33 = 0.67 erteket veszi fel. Ebbol −y = 0.44, azaz y = −0.44.

Tehat −1.44−mσ

= −0.44, kaptunk egy egyenletet a ket ismeretlenunkre. Felhasznalva a masik valoszınuseget is

0.063 = P(ξ > 0.53) = 1− P(ξ ≤ 0.53) = 1− P(ξ < 0.53) = 1− P

(ξ − m

σ<

0.53− m

σ

)=

= 1− Φ

(0.53− m

σ

),

azaz Φ(

0.53−mσ

)= 0.937, amibol 0.53−m

σ= 1.53, ez a masik egyenlet a ket ismeretlenunkre.

A ket egyenletbol m = −1 es σ = 1 adodik, tehat a (−1, 1) parameteru normalis eloszlasu valoszınusegivaltozorol szolt a feladat.

Horvath Gabor Analızis 2

Page 267: Anal zis 2 - Széchenyi István Universityhorvathg/bsc/gazdmath2.pdfHa f es g gra konja, ak ar t obbsz or is, atmeszi egym ast [a;b]-n, az a-ban es b-ben all tott fugg} oleges egyenesnek,

Nevezetes valoszınusegszamıtasi tetelek

Tetel 67 (Markov egyenlotlenseg)

Ha ξ nemnegatıv ertekeket felvevo valoszınusegi valtozo, a pozitıvvalos szam, akkor

P(ξ ≥ a) ≤ M(ξ)

a,

ha letezik az M(ξ) varhato ertek.

Ekkor az is igaz, hogy

P(ξ < a) ≥ 1− M(ξ)

a.

Horvath Gabor Analızis 2

Page 268: Anal zis 2 - Széchenyi István Universityhorvathg/bsc/gazdmath2.pdfHa f es g gra konja, ak ar t obbsz or is, atmeszi egym ast [a;b]-n, az a-ban es b-ben all tott fugg} oleges egyenesnek,

Nevezetes valoszınusegszamıtasi tetelek

Tetel 68 (Csebisev egyenlotlenseg)

Ha ξ-nek van varhato erteke es szorasa, b pozitıv valos szam, akkor

P(|ξ −M(ξ)| ≥ bD(ξ)) ≤ 1

b2.

Ekkor az is igaz, hogy

P(|ξ −M(ξ)| < bD(ξ)) ≥ 1− 1

b2.

Ezt pedig

P(M(ξ)− bD(ξ) < ξ < M(ξ) + bD(ξ)) ≥ 1− 1

b2

alakban is felırhatjuk. A Csebisev egyenlotlenseggel tehat pusztana varhato ertek es a szoras ismereteben becslest kaphatunk egyintervallumba eses valoszınusegerol.

Horvath Gabor Analızis 2

Page 269: Anal zis 2 - Széchenyi István Universityhorvathg/bsc/gazdmath2.pdfHa f es g gra konja, ak ar t obbsz or is, atmeszi egym ast [a;b]-n, az a-ban es b-ben all tott fugg} oleges egyenesnek,

Nevezetes valoszınusegszamıtasi tetelek

Tetel 69 (A nagy szamok torvenyenek Bernoulli alakja)

Legyen A egy kıserlet egyik esemenye, P(A) = p. Ismeteljuk meg akıserletet n-szer fuggetlenul. Jelolje ξn az A esemeny gyakorisagata kıserletsorozatban. Ekkor tetszoleges kicsi pozitıv ε es δszamokhoz letezik csak ε-tol es δ-tol fuggo N pozitıv egesz szamugy, hogy n ≥ N eseten

P

(∣∣∣∣ξn

n− p

∣∣∣∣ < ε

)≥ 1− δ.

Horvath Gabor Analızis 2

Page 270: Anal zis 2 - Széchenyi István Universityhorvathg/bsc/gazdmath2.pdfHa f es g gra konja, ak ar t obbsz or is, atmeszi egym ast [a;b]-n, az a-ban es b-ben all tott fugg} oleges egyenesnek,

Nevezetes valoszınusegszamıtasi tetelek

Definıcio 77

A ξ1, ξ2, . . . , ξn valoszınusegi valtozok teljesen fuggetlenek, hateszoleges x1, x2, . . . , xn ertekek eseten aξ1 < x1,ξ2 < x2,. . . ,ξn < xn esemenyek teljesen fuggetlenek.

Tetel 70 (A centralis hatareloszlas tetel)

Legyenek ξ1, ξ2, . . . , ξn azonos eloszlasu teljesen fuggetlenvaloszınusegi valtozok, es tegyuk fel, hogy letezik az M(ξi ) = mvarhato ertek es a D(ξi ) = σ szoras. Legyen

ηn =ξ1 + ξ2 + . . .+ ξn − nm

σ√

n,

es jelolje Fn az ηn eloszlasfuggvenyet. Ekkor minden x ∈ R eseten

limn→∞

Fn(x) = Φ(x).

Horvath Gabor Analızis 2

Page 271: Anal zis 2 - Széchenyi István Universityhorvathg/bsc/gazdmath2.pdfHa f es g gra konja, ak ar t obbsz or is, atmeszi egym ast [a;b]-n, az a-ban es b-ben all tott fugg} oleges egyenesnek,

Feladat 73

Legyen ξ λ = 2 parameteru Poisson eloszlasu valoszınusegi valtozo.Adjunk becslest a P(ξ > 39) valoszınusegre.

Megoldas: Tudjuk, hogy ξ nemnegatıv egesz ertekeket vesz fel, esM(ξ) = 2. Tovabba ξ > 39 = ξ ≥ 40. Igy a Markovegyenlotlenseg szerint

P(ξ > 39) = P(ξ ≥ 40) ≤ M(ξ)

40=

2

40= 0.05.

A P(ξ > 39) valoszınuseget egy harminckilenc tagu osszegkiszamolasaval pontosan is ki tudnank szamolni, de ez kalkulatorralfaradsagos lenne. A pontos ertek egyebkent 3 · 10−10, ami valobankisebb 0.05-nel, a Markov egyenlotlensegbol kapott becsleslathatoan igen durva, de jobb, mint a semmi.

Horvath Gabor Analızis 2

Page 272: Anal zis 2 - Széchenyi István Universityhorvathg/bsc/gazdmath2.pdfHa f es g gra konja, ak ar t obbsz or is, atmeszi egym ast [a;b]-n, az a-ban es b-ben all tott fugg} oleges egyenesnek,

Feladat 73

Legyen ξ λ = 2 parameteru Poisson eloszlasu valoszınusegi valtozo.Adjunk becslest a P(ξ > 39) valoszınusegre.Megoldas: Tudjuk, hogy ξ nemnegatıv egesz ertekeket vesz fel, esM(ξ) = 2. Tovabba ξ > 39 = ξ ≥ 40. Igy a Markovegyenlotlenseg szerint

P(ξ > 39) = P(ξ ≥ 40) ≤ M(ξ)

40=

2

40= 0.05.

A P(ξ > 39) valoszınuseget egy harminckilenc tagu osszegkiszamolasaval pontosan is ki tudnank szamolni, de ez kalkulatorralfaradsagos lenne. A pontos ertek egyebkent 3 · 10−10, ami valobankisebb 0.05-nel, a Markov egyenlotlensegbol kapott becsleslathatoan igen durva, de jobb, mint a semmi.

Horvath Gabor Analızis 2

Page 273: Anal zis 2 - Széchenyi István Universityhorvathg/bsc/gazdmath2.pdfHa f es g gra konja, ak ar t obbsz or is, atmeszi egym ast [a;b]-n, az a-ban es b-ben all tott fugg} oleges egyenesnek,

Feladat 74

Legyen ξ olyan valoszınusegi valtozo, amelyre M(ξ) = 4, D(ξ) = 3. Adjunk becslest a P(−2 < ξ < 10)valoszınusegre.

Megoldas: Alakıtsuk at az egyenlotlenseget ugy, hogy minden oldalbol vonjuk ki a varhato erteket.

P(−2 < ξ < 10) = P(−2− 4 < ξ − 4 < 10− 4) =

= P(−6 < ξ − M(ξ) < 6).

Mivel az egyenlotlenseglanc ket szelen allo szam abszolut erteke egyenlo, ezek felırhatok a szoras tobbszoroseikentugy, hogy a ket szorzo egymas mınusz egyszerese lesz. Most a szoras 3, ıgy azt kapjuk, hogy

P(−2 < ξ < 10) = P(−2 · 3 < ξ − M(ξ) < 2 · 3) =

= P(−2 · D(ξ) < ξ − M(ξ) < 2 · D(ξ)).

Az utolso egyenlotlenseg ugy is felırhato, hogy P(|ξ − M(ξ)| < 2 · D(ξ)), es ezek az eredeti egyenlotlensegekvivalens atalakıtasai voltak, ezert a Csebisev egyenlotlenseg felhasznalasaval

P(−2 < ξ < 10) = P(|ξ − M(ξ)| < 2 · D(ξ)) ≥ 1−1

22=

3

4.

Horvath Gabor Analızis 2

Page 274: Anal zis 2 - Széchenyi István Universityhorvathg/bsc/gazdmath2.pdfHa f es g gra konja, ak ar t obbsz or is, atmeszi egym ast [a;b]-n, az a-ban es b-ben all tott fugg} oleges egyenesnek,

Feladat 74

Legyen ξ olyan valoszınusegi valtozo, amelyre M(ξ) = 4, D(ξ) = 3. Adjunk becslest a P(−2 < ξ < 10)valoszınusegre.Megoldas: Alakıtsuk at az egyenlotlenseget ugy, hogy minden oldalbol vonjuk ki a varhato erteket.

P(−2 < ξ < 10) = P(−2− 4 < ξ − 4 < 10− 4) =

= P(−6 < ξ − M(ξ) < 6).

Mivel az egyenlotlenseglanc ket szelen allo szam abszolut erteke egyenlo, ezek felırhatok a szoras tobbszoroseikentugy, hogy a ket szorzo egymas mınusz egyszerese lesz. Most a szoras 3, ıgy azt kapjuk, hogy

P(−2 < ξ < 10) = P(−2 · 3 < ξ − M(ξ) < 2 · 3) =

= P(−2 · D(ξ) < ξ − M(ξ) < 2 · D(ξ)).

Az utolso egyenlotlenseg ugy is felırhato, hogy P(|ξ − M(ξ)| < 2 · D(ξ)), es ezek az eredeti egyenlotlensegekvivalens atalakıtasai voltak, ezert a Csebisev egyenlotlenseg felhasznalasaval

P(−2 < ξ < 10) = P(|ξ − M(ξ)| < 2 · D(ξ)) ≥ 1−1

22=

3

4.

Horvath Gabor Analızis 2

Page 275: Anal zis 2 - Széchenyi István Universityhorvathg/bsc/gazdmath2.pdfHa f es g gra konja, ak ar t obbsz or is, atmeszi egym ast [a;b]-n, az a-ban es b-ben all tott fugg} oleges egyenesnek,

Feladat 75

Legyen ξ olyan valoszınusegi valtozo, amelyre M(ξ) = 5, D(ξ) = 2. Adjunk becslest a P(2 < ξ < 9)valoszınusegre.

Megoldas: Most is alakıtsuk at az egyenlotlenseget ugy, hogy minden oldalbol vonjuk ki a varhato erteket.

P(2 < ξ < 9) = P(2− 5 < ξ − 5 < 9− 5) =

= P(−3 < ξ − M(ξ) < 4).

Az egyenlotlenseglanc ket szelen allo szam abszolut erteke most nem egyenlo. Ilyenkor szukıthetjuk ugy azintervallumot, hogy a ket szelere az eelozo szamok kozul a kisebb abszolut erteku es annak mınusz egyszerese kerul.Mivel egy bovebb halmazba eses valoszınusege nagyobb, mint egy szukebb halmazba esese

P(−3 < ξ − M(ξ) < 4) ≥ P(−3 < ξ − M(ξ) < 3).

Ha a ket szelen allo szam abszolut erteke mar egyenlo, felırhatok a szoras tobbszoroseikent ugy, hogy a ket szorzoegymas mınusz egyszerese lesz. Most a szoras 2, aminek a 3 a 3/2-szerese, ıgy azt kapjuk, hogy

P(−3 < ξ − M(ξ) < 3) = P(−3/2 · 2 < ξ − M(ξ) < 3/2 · 2) =

= P(−3/2 · D(ξ) < ξ − M(ξ) < 3/2 · D(ξ)).

Az utolso egyenlotlenseg ugy is felırhato, hogy P(|ξ − M(ξ)| < 3/2 · D(ξ)), ezek az eredeti egyenlotlensegekvivalens atalakıtasai voltak, ezert a Csebisev egyenlotlenseg felhasznalasaval

P(2 < ξ < 9) ≥ P(−3 < ξ − M(ξ) < 3) = P(|ξ − M(ξ)| < 3/2 · D(ξ)) ≥ 1−1

(3/2)2=

5

9.

Horvath Gabor Analızis 2

Page 276: Anal zis 2 - Széchenyi István Universityhorvathg/bsc/gazdmath2.pdfHa f es g gra konja, ak ar t obbsz or is, atmeszi egym ast [a;b]-n, az a-ban es b-ben all tott fugg} oleges egyenesnek,

Feladat 75

Legyen ξ olyan valoszınusegi valtozo, amelyre M(ξ) = 5, D(ξ) = 2. Adjunk becslest a P(2 < ξ < 9)valoszınusegre.Megoldas: Most is alakıtsuk at az egyenlotlenseget ugy, hogy minden oldalbol vonjuk ki a varhato erteket.

P(2 < ξ < 9) = P(2− 5 < ξ − 5 < 9− 5) =

= P(−3 < ξ − M(ξ) < 4).

Az egyenlotlenseglanc ket szelen allo szam abszolut erteke most nem egyenlo. Ilyenkor szukıthetjuk ugy azintervallumot, hogy a ket szelere az eelozo szamok kozul a kisebb abszolut erteku es annak mınusz egyszerese kerul.Mivel egy bovebb halmazba eses valoszınusege nagyobb, mint egy szukebb halmazba esese

P(−3 < ξ − M(ξ) < 4) ≥ P(−3 < ξ − M(ξ) < 3).

Ha a ket szelen allo szam abszolut erteke mar egyenlo, felırhatok a szoras tobbszoroseikent ugy, hogy a ket szorzoegymas mınusz egyszerese lesz. Most a szoras 2, aminek a 3 a 3/2-szerese, ıgy azt kapjuk, hogy

P(−3 < ξ − M(ξ) < 3) = P(−3/2 · 2 < ξ − M(ξ) < 3/2 · 2) =

= P(−3/2 · D(ξ) < ξ − M(ξ) < 3/2 · D(ξ)).

Az utolso egyenlotlenseg ugy is felırhato, hogy P(|ξ − M(ξ)| < 3/2 · D(ξ)), ezek az eredeti egyenlotlensegekvivalens atalakıtasai voltak, ezert a Csebisev egyenlotlenseg felhasznalasaval

P(2 < ξ < 9) ≥ P(−3 < ξ − M(ξ) < 3) = P(|ξ − M(ξ)| < 3/2 · D(ξ)) ≥ 1−1

(3/2)2=

5

9.

Horvath Gabor Analızis 2